Download as docx, pdf, or txt
Download as docx, pdf, or txt
You are on page 1of 54

INSURANCE LAW CASE DIGESTS Held: Yes.

The test to determine if a contract is an


insurance contract or not, depends on the nature of
I. INTRODUCTION the promise, the act required to be performed, and
the exact nature of the agreement in the light of the
1. G.R. No. 154514. July 28, 2005
occurrence, contingency, or circumstances under
WHITE GOLD MARINE SERVICES, INC., which the performance becomes requisite. It is not
Petitioners vs. by what it is called.
PIONEER INSURANCE AND SURETY
CORPORATION AND THE STEAMSHIP MUTUAL
Basically, an insurance contract is a contract of
UNDERWRITING ASSOCIATION (BERMUDA)
indemnity. In it, one undertakes for a consideration
LTD., Respondents.
to indemnify another against loss, damage or
This petition for review assails the Decision1 dated liability arising from an unknown or contingent
July 30, 2002 of the Court of Appeals in CA-G.R. event.
SP No. 60144, affirming the Decision2 dated May
In particular, a marine insurance undertakes to
3, 2000 of the Insurance Commission in I.C. Adm.
indemnify the assured against marine losses, such
Case No. RD-277. Both decisions held that there
as the losses incident to a marine adventure.
was no violation of the Insurance Code and the
Section 99 of the Insurance Code enumerates the
respondents do not need license as insurer and
coverage of marine insurance.
insurance agent/broker.
Relatedly, a mutual insurance company is a
The facts are undisputed.
cooperative enterprise where the members are
Facts: White Gold Marine Services, Inc. (White both the insurer and insured. In it, the members all
Gold) procured a protection and indemnity contribute, by a system of premiums or
coverage for its vessels from The Steamship assessments, to the creation of a fund from which
Mutual Underwriting Association (Bermuda) Limited all losses and liabilities are paid, and where the
(Steamship Mutual) through Pioneer Insurance and profits are divided among themselves, in proportion
Surety Corporation (Pioneer). Subsequently, White to their interest. Additionally, mutual insurance
Gold was issued a Certificate of Entry and associations, or clubs, provide three types of
Acceptance. Pioneer also issued receipts coverage, namely, protection and indemnity, war
evidencing payments for the coverage. When risks, and defense costs.
White Gold failed to fully pay its accounts,
A P & I Club is a form of insurance against third
Steamship Mutual refused to renew the coverage.
party liability, where the third party is anyone other
Steamship Mutual thereafter filed a case against
than the P & I Club and the members. By definition
White Gold for collection of sum of money to
then, Steamship Mutual as a P & I Club is a mutual
recover the latter’s unpaid balance. White Gold on
insurance association engaged in the marine
the other hand, filed a complaint before the
insurance business.
Insurance Commission claiming that Steamship
Mutual violated Sections 186[4] and 187[5] of the
Insurance Code, while Pioneer violated Sections
299 300 and 301 in relation to Sections 302 and Since a contract of insurance involves public
303, thereof. The Insurance Commission dismissed interest, regulation by the State is necessary. Thus,
the complaint. It said that there was no need for no insurer or insurance company is allowed to
Steamship Mutual to secure a license because it engage in the insurance business without a license
was not engaged in the insurance business. It or a certificate of authority from the Insurance
explained that Steamship Mutual was a Protection Commission.
and Indemnity Club (P & I Club). Likewise, Pioneer
need not obtain another license as insurance agent
and/or a broker for Steamship Mutual because Yes. Pioneer is the resident agent of Steamship
Steamship Mutual was not engaged in the Mutual as evidenced by the certificate of
insurance business. Moreover, Pioneer was registration[22] issued by the Insurance
already licensed, hence, a separate license solely Commission. It has been licensed to do or transact
as agent/broker of Steamship Mutual was already insurance business by virtue of the certificate of
superfluous. authority issued by the same agency. However, a
Certification from the Commission states that
Pioneer does not have a separate license to be an
Issues: Whether or not the contract entered into by agent/broker of Steamship Mutual.
the parties is an insurance contract.
Whether or not Pioneer is required to obtain Although Pioneer is already licensed as an
a separate license as an insurance agent. insurance company, it needs a separate license to
act as insurance agent for Steamship Mutual.
Section 299 of the Insurance Code clearly states:
1
Verendia to give notice to Fidelity of other contracts
of insurance was waived by Fidelity as shown by its
No person shall act as an insurance agent or as an conduct in attempting to settle the claim of
insurance broker in the solicitation or procurement Verendia.
of applications for insurance, or receive for services
in obtaining insurance, any commission or other
compensation from any insurance company doing
business in the Philippines or any agent thereof, the issues Fidelity raises therein are: (a) whether or
without first procuring a license so to act from the not the contract of lease submitted by Verendia to
Commissioner, which must be renewed annually support his claim on the fire insurance policy
on the first day of January, or within six months constitutes a false declaration which would forfeit
thereafter. his benefits under Section 13 of the policy and (b)
whether or not, in submitting the subrogation
receipt in evidence, Fidelity had in effect agreed to
settle Verendia's claim in the amount stated in said
2. G.R. No. 75605 January 22, 1993 receipt
RAFAEL (REX) VERENDIA, petitioner,
vs. COURT OF APPEALS and FIDELITY &
SURETY CO. OF THE ISSUE: W/N there was false declaration which
PHILIPPINES, respondents. would forfeit his benefits under Section 13 of the
policy
G.R. No. 76399 January 22, 1993
FIDELITY & SURETY CO. OF THE PHILIPPINES,
INC., petitioner,
vs. HELD: YES.
RAFAEL VERENDIA and THE COURT OF
 Section 13 thereof which is expressed in
APPEALS, respondents.
terms that are clear and unambiguous, that
FACTS: all benefits under the policy shall be forfeited
"If the claim be in any respect fraudulent, or
 Rafael (Rex) Verendia's residential building if any false declaration be made or used in
was insured with Fidelity and Surety support thereof, or if any fraudulent means
Insurance Company, Country Bankers or devises are used by the Insured or
Insurance and Development anyone acting in his behalf to obtain any
Insurance with Monte de Piedad & Savings benefit under the policy"
Bank as beneficiary
 Robert Garcia then executed an affidavit
 December 28, 1980 early morning: the before the National Intelligence and Security
building was completely destroyed by fire Authority (NISA) to the effect that he was not
 Fidelity refused the claim stating that there the lessee of Verendia's house and that his
was a misrepresentation since the lessee signature on the contract of lease was a
was not Roberto Garcia but Marcelo Garcia  complete forgery.

 trial court: favored Fidelity  Worse yet, by presenting a false lease


contract, Verendia, reprehensibly
 CA: reversed disregarded the principle that insurance
On May 24, 1983, the trial court rendered a contracts are uberrimae fidae and demand
decision, per Judge Rodolfo A. Ortiz, ruling in favor the most abundant good faith.
of Fidelity. In sustaining the defenses set up by
Fidelity, the trial court ruled that Paragraph 3 of the
policy was also violated by Verendia in that the In view of the conflicting findings of the trial court
insured failed to inform Fidelity of his other and the appellate court on important issues in
insurance coverages with Country Bankers these consolidated cases and it appearing that the
Insurance and Development Insurance. appellate court judgment is based on a
misapprehension of facts, this Court shall review
the evidence on record.
Verendia appealed to the then Intermediate
Appellate Court and in a decision promulgated on
March 31, 1986, (CA-G.R. No. CV No. 02895, The contract of lease upon which Verendia relies to
Coquia, Zosa, Bartolome, and Ejercito (P), the support his claim for insurance benefits, was
appellate court reversed for the following reasons: entered into between him and one Robert Garcia,
(a) there was no misrepresentation concerning the married to Helen Cawinian, on June 25, 1980 (Exh.
lease for the contract was signed by Marcelo "1"), a couple of days after the effectivity of the
Garcia in the name of Roberto Garcia; and (b) insurance policy. When the rented residential
Paragraph 3 of the policy contract requiring building was razed to the ground on December 28,
2
1980, it appears that Robert Garcia (or Roberto contract to support his claim under Fire Insurance
Garcia) was still within the premises. However, Policy No. F-18876, the terms of the policy should
according to the investigation report prepared by be strictly construed against the insured. Verendia
Pat. Eleuterio M. Buenviaje of the Antipolo police, failed to live by the terms of the policy, specifically
the building appeared to have "no occupant" and Section 13 thereof which is expressed in terms that
that Mr. Roberto Garcia was "renting on the are clear and unambiguous, that all benefits under
otherside (sic) portion of said compound" the policy shall be forfeited

Robert Garcia disappeared after the fire. It was There is also no reason to conclude that by
only on October 9, 1981 that an adjuster was able submitting the subrogation receipt as evidence in
to locate him. Robert Garcia then executed an court, Fidelity bound itself to a "mutual agreement"
affidavit before the National Intelligence and to settle Verendia's claims in consideration of the
Security Authority (NISA) to the effect that he was amount of P142,685.77. While the said receipt
not the lessee of Verendia's house and that his appears to have been a filled-up form of Fidelity, no
signature on the contract of lease was a complete representative of Fidelity had signed it. It is even
forgery. Thus, on the strength of these facts, the incomplete as the blank spaces for a witness and
adjuster submitted a report dated December 4, his address are not filled up. More significantly, the
1981 recommending the denial of Verendia's claim. same receipt states that Verendia had received the
aforesaid amount.

Ironically, during the trial, Verendia admitted that it


was not Robert Garcia who signed the lease WHEREFORE, the petition in G.R. No. 75605 is
contract. According to Verendia, it was signed by DISMISSED. The petition in G.R. No. 76399 is
Marcelo Garcia, cousin of Robert, who had been GRANTED and the decision of the then
paying the rentals all the while. Verendia, however, Intermediate Appellate Court under review is
failed to explain why Marcelo had to sign his REVERSED and SET ASIDE and that of the trial
cousin's name when he in fact was paying for the court is hereby REINSTATED and UPHELD.
rent and why he (Verendia) himself, the lessor,
allowed such a ruse. Fidelity's conclusions on
these proven facts appear, therefore, to have
sufficient bases; Verendia concocted the lease
contract to deflect responsibility for the fire towards 3. G.R. No. 112360               July 18, 2000
an alleged "lessee", inflated the value of the RIZAL SURETY & INSURANCE
property by the alleged monthly rental of P6,500 COMPANY, petitioner,
when in fact, the Provincial Assessor of Rizal had vs. COURT OF APPEALS and TRANSWORLD
assessed the property's fair market value to be only KNITTING MILLS, INC., respondents
P40,300.00, insured the same property with two
other insurance companies for a total coverage of
around P900,000, and created a dead-end for the
Facts:
adjuster by the disappearance of Robert Garcia.
Rizal Surety issued a 1 million peso
fire insurance policy with Transworld. This
Basically a contract of indemnity, an insurance was increased to 1.5 million. A four span building
contract is the law between the parties (Pacific was part of the policy. A fire broke out
Banking Corporation vs. Court of Appeals 168 and gutted the building, together with a two storey
SCRA 1 [1988]). Its terms and conditions constitute building behind it were gaming machines were
the measure of the insurer's liability and stored. The company filed its claims but to no avail.
compliance therewith is a condition precedent to Hence, it brought a suit in court. It aimed to make
the insured's right to recovery from the insurer Rizal pay for almost 3 million including legal
(Oriental Assurance Corporation vs. Court of interest and damages. Rizal claimed that the policy
Appeals, 200 SCRA 459 [1991], citing Perla only covered damage on the four span building and
Compania de Seguros, Inc. vs. Court of Appeals, not the two storey building. The trial court ruled in
185 SCRA 741 [1991]). As it is also a contract of Transworld’s favor and ordered Rizal to pay actual
adhesion, an insurance contract should be liberally damages only. The court of appeals increased the
construed in favor of the insured and strictly damages. The insurance company filed a MFR.
against the insurer company which usually The CA answered by modifying the imposition of
prepares it (Western Guaranty Corporation vs. interest. Not satisfied, the insurance company
Court of Appeals, 187 SCRA 652 [1980]). petitioned to the Supreme Court.
Issue:
Considering, however, the foregoing discussion WON Rizal Surety is liable for loss of the two-
pointing to the fact that Verendia used a false lease storey building considering that the
3
fire insurance policy sued upon covered only the This generally means that the policy didn’t limit its
contents of the four-span building. coverage to what was stored in the four-span
building.
As to questions of fact, both the trial court and the
petitioner Rizal Surety & Insurance Company found Court of Appeals found that the so called "annex "
its way to this Court via the present Petition, was not an annex building but an integral part of
contending that: the four-span building described in the policy
I.....SAID DECISION (ANNEX A) ERRED IN and consequently, the machines and spare parts
ASSUMING THAT THE ANNEX BUILDING stored were covered by the fire insurance.
WHERE THE BULK OF THE BURNED A report said: "Two-storey building constructed of
PROPERTIES WERE STORED, WAS INCLUDED partly timber and partly concrete
IN THE COVERAGE OF THE INSURANCE hollow blocks under g.i. roof which is adjoining
POLICY ISSUED BY RIZAL SURETY TO and intercommunicating with the repair of the
TRANSWORLD. first right span of the lofty storey building and
II.....SAID DECISION AND RESOLUTION thence by property fence wall."
(ANNEXES A AND B) ERRED IN NOT "Art.1377. The interpretation of obscure words or
CONSIDERING THE PICTURES (EXHS. 3 TO 7- stipulations in a contract shall not favor the party
C-RIZAL SURETY), TAKEN IMMEDIATELY who caused the obscurity"
AFTER THE FIRE, WHICH CLEARLY SHOW
THAT THE PREMISES OCCUPIED BY Landicho v GSIS- the 'terms in an insurance policy,
TRANSWORLD, WHERE THE INSURED which are ambiguous, equivocal, or uncertain are
PROPERTIES WERE LOCATED, SUSTAINED to be construed strictly and most strongly against
PARTIAL DAMAGE ONLY. the insurer, and liberally in favor of the insured so
as to effect the dominant purpose of indemnity or
III. SAID DECISION (ANNEX A) ERRED IN NOT payment to the insured’
HOLDING THAT TRANSWORLD HAD ACTED IN
PALPABLE BAD FAITH AND WITH MALICE IN The issue of whether or not Transworld has an
FILING ITS CLEARLY UNFOUNDED CIVIL insurable interest in the fun and amusement
ACTION, AND IN NOT ORDERING machines and spare parts, which entitles it to be
TRANSWORLD TO PAY TO RIZAL SURETY indemnified for the loss thereof, had been settled in
MORAL AND PUNITIVE DAMAGES (ART. 2205, another SC case
CIVIL CODE), PLUS ATTORNEY'S FEES AND
EXPENSES OF LITIGATION (ART. 2208 PARS. 4
and 11, CIVIL CODE).11 The issue of whether or not Transworld has an
The Petition is not impressed with merit. insurable interest in the fun and amusement
machines and spare parts, which entitles it to be
It is petitioner's submission that the fire insurance indemnified for the loss thereof, had been settled in
policy litigated upon protected only the contents of G.R. No. L-111118, entitled New India Assurance
the main building (four-span),12 and did not include Company, Ltd., vs. Court of Appeals, where the
those stored in the two-storey annex building. On appeal of New India from the decision of the Court
the other hand, the private respondent theorized of Appeals under review, was denied with finality
that the so called "annex" was not an annex but by this Court on February 2, 1994.
was actually an integral part of the four-span
building13 and therefore, the goods and items stored So also, considering that the two-storey building
therein were covered by the same fire insurance aforementioned was already existing when subject
policy. fire insurance policy contract was entered into on
January 12, 1981, having been constructed
Held: Yes. Petition dismissed. sometime in 1978,18 petitioner should have
specifically excluded the said two-storey building
Ratio: from the coverage of the fire insurance if minded to
The policy had clauses on the building coverage exclude the same but if did not, and instead, went
that read: on to provide that such fire insurance policy covers
the products, raw materials and supplies stored
"contained and/or stored during the currency of this within the premises of respondent Transworld
Policy in the premises occupied by them forming which was an integral part of the four-span building
part of the buildings situated within own occupied by Transworld, knowing fully well the
Compound" existence of such building adjoining and
"First, said properties must be contained and/or intercommunicating with the right section of the
stored in the areas occupied by Transworld and four-span building.
second, said areas must form part of the building
described in the policy xxx"
The controversy at bar is on all fours with the
aforecited case. Considering that private
4
respondent's insurable interest in, and agreement. However, petitioner denied her claim
compensability for the loss of subject fun and saying that the Health Care Agreement was void
amusement machines and spare parts, had been on the ground that there was a concealment
adjudicated, settled and sustained by the Court of regarding Ernani’s medical history.
Appeals in CA-G.R. CV NO. 28779, and by this
Court in G.R. No. L-111118, in a Resolution, dated
February 2, 1994, the same can no longer be After his discharge, Ernani was brought again at
relitigated and passed upon in the present case. the Chinese General Hospital where he died. Julita
Ineluctably, the petitioner, Rizal Surety Insurance then filed an action for damages against
Company, is bound by the ruling of the Court of Philamcare including its President Dr. Benito
Appeals and of this Court that the private Reverente. RTC ruled in favour of Julita, and this
respondent has an insurable interest in the was affirmed by the CA except that it deleted
aforesaid fun and amusement machines and spare awards for damages and absolved Dr. Reverente.
parts; and should be indemnified for the loss of the
same.
ISSUE:

So also, the Court of Appeals correctly adjudged Whether or not the agreement is a contract of
petitioner liable for the amount of P470,328.67, it indemnity.
being the total loss and damage suffered by
Transworld for which petitioner Rizal Insurance is
liable.26 RULING:
All things studiedly considered and viewed in Yes. The health care agreement was in the nature
proper perspective, the Court is of the irresistible of non-life insurance, which is primarily a contract
conclusion, and so finds, that the Court of Appeals of indemnity. The insurable interest of respondent’s
erred not in holding the petitioner, Rizal Surety husband in obtaining the health care agreement
Insurance Company, liable for the destruction and was his own health. Once the member incurs
loss of the insured buildings and articles of the hospital, medical or any other expense arising from
private respondent. sickness, injury or other stipulated contingent, the
health care provider must pay for the same to the
WHEREFORE, the Decision, dated July 15, 1993, extent agreed upon under the contract.
and the Resolution, dated October 22, 1993, of the
Court of Appeals in CA-G.R. CV NO. 28779 are Elements of an Insurance Contract
AFFIRMED in toto
Section 2 (1) of the Insurance Code defines a
contract of insurance as an agreement whereby
one undertakes for a consideration to indemnify
4. Philamcare Health Systems, Inc. vs. Court of another against loss, damage or liability arising
Appeals and Julita Trinos from an unknown or contingent event. An
G.R. No. 125678, March 18, 2002 insurance contract exists where the following
elements concur:
(a) The insured has an insurable interest;
FACTS:
(b) The insured is subject to a risk of loss by the
Respondent Julita Trinos’ deceased husband, happening of the designated peril;
Ernani Trinos applied for a health care coverage
with petitioner Philamcare Health Systems, Inc. In (c) The insurer assumes the risk;
the standard application form, he answered NO to (d) Such assumption of risk is part of a general
the following question: Have you or any of your scheme to distribute actual losses among a large
family members ever consulted or been treated for group of persons bearing a similar risk; and
high blood pressure, heart trouble, diabetes,
cancer, liver disease, asthma or peptic ulcer? (e) In consideration of the insurer’s promise, the
insured pays a premium.
WHEREFORE, in view of the foregoing, the petition
The application was approved and extended for 13 is DENIED. The assailed decision of the Court of
months, until June 1, 1990. Appeals dated December 14, 1995 is AFFIRMED.

During the period of the coverage, Ernani suffered 5. G.R. No. 115278 May 23, 1995
a heart attack resulting in confinement for a month
at the Manila Medical Center (MMC). While her FORTUNE INSURANCE AND SURETY CO.,
husband was in the hospital, respondent tried to INC., petitioner,
claim the benefits under the health care
5
vs. COURT OF APPEALS and PRODUCERS own employees, thus an excluded loss under the
BANK OF THE PHILIPPINES, respondents. general exceptions in the policy.

DAVIDE, JR., J.: Producers filed a case against Fortune for the


recovery of the insurance proceeds.
The fundamental legal issue raised in this petition
for review on certiorari is whether the petitioner is
liable under the Money, Security, and Payroll
Robbery policy it issued to the private respondent The trial court ruled in favor of Producers Bank.
or whether recovery thereunder is precluded under The CA affirmed. The lower courts found that both
the general exceptions clause thereof. Both the trial Magalong and Atiga were not employees of the
court and the Court of Appeals held that there Bank. The SC reversed.
should be recovery. The petitioner contends
otherwise.
Issues resolved —
This case began with the filing with the Regional
Trial Court (RTC) of Makati, Metro Manila, by Was Fortune Insurance liable under the Money,
private respondent Producers Bank of the Security, and Payroll Robbery policy it issued to
Philippines (hereinafter Producers) against Producers Bank or was the recovery thereunder is
petitioner Fortune Insurance and Surety Co., Inc. precluded under the general exceptions clause
(hereinafter Fortune) of a complaint for recovery of thereof?
the sum of P725,000.00 under the policy issued by HELD – RECOVERY PRECLUDED. FORTUNE
Fortune. The sum was allegedly lost during a NOT LIABLE.
robbery of Producer's armored vehicle while it was
in transit to transfer the money from its Pasay City On the applicable law. The Court said that theft or
Branch to its head office in Makati. The case was robbery insurance policy, as with the case at bar, is
docketed as Civil Case No. 1817 and assigned to a form of casualty insurance governed by Sec. 174
Branch 146 thereof. of the Insurance Code. The Court also noted that
the Code contains no provisions specifically
applicable to casualty insurance. Thus, this shall be
Rule Synopsis governed by the general provisions applicable to all
types of insurance.
Manpower supplied by agencies may be
considered employees or authorized
representatives of the employer for purposes of On the usual exceptions in robbery and theft
construing exceptions to a robbery and theft policy policies. The numerous restrictions in the robbery
exempting the insurer from liability for any loss and theft policies were intended to reduce the
caused by any dishonest, fraudulent or criminal act moral hazard. It was noted that in these types of
of its employees or authorized representatives, insurance, the opportunity to defraud the insurer is
among others. so great (e.g. through conspiracy, etc.). Usually,
losses occasioned on the acts of the persons under
the insured’s service and employment, are
Case Summary excepted risks. The purpose of the exception is to
Producers Bank had a Money, Security, and guard against liability should the theft be committed
Payroll Robbery policy with Fortune Insurance and by one having unrestricted access to the property.
Surety Co., Inc. The policy states that the insurer “Service” and “employment” in this case are to be
shall not be liable for: “any loss caused by any understood in their ordinary sense.
dishonest, fraudulent or criminal act of the insured
or any officer, employee, partner, director, trustee
or authorized representative of the Insured whether On whether Magalong and Atiga were Producers’
acting alone or in conjunction with others.” employees, given that they were merely supplied
Producers then allegedly lost a sum of P725k by agencies. The Court held that Magalong and
during a robbery of its armored vehicle while it was Atiga may be considered employees of Producers.
in transit to transfer the money from Pasay to This is under the assumption that the contract of
Makati. The vehicle was driven by one Magalong Producers with the providers of concerned
(assigned by PRC Management Systems). The manpower were “labor-only.” And under the Labor
security guard, Atiga, was assigned by Unicorn Code, employees under labor-only contract are
Security Services, Inc. The driver and the security considered employees of the party employing them
guard, along with others, were charged with and not of the party who supplied them to the
highway robbery. The criminal case was still employer, the former merely acting as an agent for
pending as of writing of this decision. Producers the latter.
filed a claim Fortune. Fortune denied on the ground
that the robbery was due to the acts of Producers’
6
However, the Court noted that there was still lack PUNO, J.:
of evidence as to the real nature of the contract
between producers and its suppliers of manpower, Before the Court is the Petition for Certiorariunder
since the parties merely entered into stipulation Rule 45 of the Revised Rules of Court by petitioner
without submitting additional evidence other than GULF RESORTS, INC., against respondent
the insurance policy. Nevertheless, the Court held PHILIPPINE CHARTER INSURANCE
that even if Magalong and Atiga were not to be CORPORATION. Petitioner assails the appellate
considered employees of Producers, then may still court decision1 which dismissed its two appeals
be deemed its authorized representatives for and affirmed the judgment of the trial court.
purposes of transferring the money in question. For
the said particular task, the subject employees FACTS:
acted as agents of Producers.
 Gulf Resorts, Inc at Agoo, La Union was
insured with American Home Assurance
Company which includes loss or damage to
Petition granted. Judgment on appeal reversed and
shock to any of the property insured by this
set aside.
Policy occasioned by or through or in
…from fulltext consequence of earthquake 
But even granting for the sake of argument that  July 16, 1990: an earthquake struck Central
these contracts were not "labor-only" contracts, and Luzon and Northern Luzon so the properties
PRC Management Systems and Unicorn Security and 2 swimming pools in its Agoo Playa
Services were truly independent contractors, we Resort were damaged
are satisfied that Magalong and Atiga were, in
 August 23, 1990: Gulf's claim was denied on
respect of the transfer of Producer's money from its
the ground that its insurance policy only
Pasay City branch to its head office in Makati, its
afforded earthquake shock coverage to the
"authorized representatives" who served as such
two swimming pools of the resort
with its teller Maribeth Alampay. Howsoever
viewed, Producers entrusted the three with the  Petitioner contends that pursuant to
specific duty to safely transfer the money to its this rider, no qualifications were
head office, with Alampay to be responsible for its placed on the scope of the
custody in transit; Magalong to drive the armored earthquake shock coverage.  Thus,
vehicle which would carry the money; and Atiga to the policy extended earthquake shock
provide the needed security for the money, the coverage to all of the insured
vehicle, and his two other companions. In short, for properties.
these particular tasks, the three acted as agents of
Producers. A "representative" is defined as one  RTC: Favored American Home
who represents or stands in the place of another; - endorsement rider means that only the two
one who represents others or another in a special swimming pools were insured against
capacity, as an agent, and is interchangeable with earthquake shock 
"agent." 23  CA: affirmed RTC
ISSUE: W/N Gulf can claim for its properties aside
In view of the foregoing, Fortune is exempt from from the 2 swimming pools
liability under the general exceptions clause of the
insurance policy.
HELD: YES. Affirmed.
 It is basic that all the provisions of the
WHEREFORE , the instant petition is hereby insurance policy should be examined and
GRANTED. The decision of the Court of Appeals in interpreted in consonance with each other.
CA-G.R. CV No. 32946 dated 3 May 1994 as well
as that of Branch 146 of the Regional Trial Court of  All its parts are reflective of the true
Makati in Civil Case No. 1817 are REVERSED and intent of the parties.
SET ASIDE. The complaint in Civil Case No. 1817
Insurance Code
is DISMISSED.
Section 2(1)

6. G.R. NO. 156167 : May 16, 2005] contract of insurance as an agreement whereby
one undertakes for a consideration to indemnify
GULF RESORTS, INC., Petitioner, v. PHILIPPINE another against loss, damage or liability arising
CHARTER INSURANCE from an unknown or contingent event
CORPORATION, Respondent.
DECISION

7
 An insurance premium is the consideration Petition to review the decision * of the Court of
paid an insurer for undertaking to indemnify Appeals, in CA-G.R. No. SP-08642, dated 21
the insured against a specified peril. March 1979, ordering petitioner Manila Mahogany
Manufacturing Corporation to pay private
 In the subject policy, no premium respondent Zenith Insurance Corporation the sum
payments were made with regard to of Five Thousand Pesos (P5,000.00) with 6%
earthquake shock coverage, except annual interest from 18 January 1973, attorney's
on the two swimming pools.  fees in the sum of five hundred pesos (P500.00),
Respondent, in compliance with the condition set and costs of suit, and the resolution of the same
by the petitioner, copied AIU Policy No. 206- Court, dated 8 February 1980, denying petitioner's
4568061-9 in drafting its Insurance Policy No. motion for reconsideration of it's decision.
31944. It is true that there was variance in some Facts:
terms, specifically in the replacement cost
endorsement, but the principal provisions of the Manila Mahogany Manufacturing insured its
policy remained essentially similar to AHAC-AIU's Mercedez Benz 4-door sedan with private
policy. Consequently, we cannot apply the "fine respondent, Zenith Insurance Corporation. In
print" or "contract of adhesion" rule in this case as between the period covered by the insurance, the
the parties' intent to limit the coverage of the policy insured vehicle was bumped and damaged by a
to the two swimming pools only is not ambiguous. truck owned by San Miguel Corporation. Zenith
then paid Manila Mahogany P5,000 for the
damaged caused. The general manager of Manila
FROMFULLTEXT: Mahogany executed a Release of Claim,
subrogating Zenith all its right to action against San
 there is no ambiguity in the terms of the contract Miguel. When Zenith demanded reimbursement
and its riders. Petitioner cannot rely on the general from San Miguel thru Insurance Adjusters, Inc., it
rule that insurance contracts are contracts of was refused to be reimbursed in the amount it paid
adhesion which should be liberally construed in to Manila Mahogany. It now demanded
favor of the insured and strictly against the insurer reimbursement from the assured for P4,500, the
company which usually prepares it. A contract of amount paid San Miguel. However, the insured
adhesion is one wherein a party, usually a refused to pay. Thus, Zenith filed a suit.
corporation, prepares the stipulations in the
contract, while the other party merely affixes his
signature or his "adhesion" thereto. Through the Issue: W/N the insurance company is entitled to
years, the courts have held that in these type of recover from the assured the amount of insurance
contracts, the parties do not bargain on equal money paid on account of right of subrogation
footing, the weaker party's participation being
reduced to the alternative to take it or leave it.
Thus, these contracts are viewed as traps for the
weaker party whom the courts of justice must Held/Ratio: Yes, the insurer Zenith Insurance may
protect.32 Consequently, any ambiguity therein is recover the insurance money in the amount of
resolved against the insurer, or construed liberally P5,000 paid to the assured, Manila Mahogany. As
in favor of the insured.33 held in Phil Air Lines v. Heald Lumber Co., if a
property is insured and the owner receives the
We cannot apply the general rule on contracts of indemnity from the insurer, it is provided in Art.
adhesion to the case at bar. Petitioner cannot claim 2207 of the New Civil Code that the insurer is
it did not know the provisions of the policy. From deemed subrogated to the rights of the insured
the inception of the policy, petitioner had required against the wrongdoer, and if the amount paid by
the respondent to copy verbatim the provisions and the insurer does not fully cover the loss, then the
terms of its latest insurance policy from AHAC-AIU. aggrieved party is the one entitled to recover the
The testimony of Mr. Leopoldo Mantohac, a direct deficiency. Although petitioner’s right to file
participant in securing the insurance policy of deficiency claim against San Miguel is with legal
petitioner, is reflective of petitioner's knowledge. basis, without prejudice to the insurer’s right of
subrogation, nevertheless when it (Manila
IN VIEW WHEREOF, the judgment of the Court of Mahogany) executed another release claim
Appeals is affirmed. The Petition for Certiorari is discharging San Miguel from all actions arising out
dismissed. of the consequence of the accident after the insurer
had paid the proceeds of the policy, the insurer is
entitled to recover from the insured the amount of
7. Manila Mahogany Manufacturing insurance money paid.
Corporation v. CA and Zenith
Insurance Corporation (1987) WHEREFORE, premises considered, the petition is
DENIED. The judgment appealed from is hereby
AFFIRMED with costs against petitioner.

8
Company (AHAC). The following day,
BURLINGTON turned over the custody of said
8. G.R. No. 150094 August 18, cargoes to FEDEX which transported the same to
2004 Manila.
FEDERAL EXPRESS CORPORATION,
petitioner, vs. AMERICAN HOME ASSURANCE The first shipment, consisting of 92 cartons arrived
COMPANY and PHILAM INSURANCE in Manila on January 29, 1994, the second,
COMPANY, INC., respondents. consisting of 17 cartons, came in two (2) days later,
or on January 31, 1994 which was immediately
stored at Cargohaus warehouse.
Basic is the requirement that before suing to
recover loss of or damage to transported goods, Prior to the arrival of the cargoes, Federal Express
the plaintiff must give the carrier notice of the loss informed GETC Cargo International Corporation,
or damage, within the period prescribed by the the customs broker hired by the consignee to
Warsaw Convention and/or the airway bill. facilitate the release of its cargoes from the Bureau
of Customs, of the impending arrival of its client’s
cargoes.
The Case
On February 10, 1994, DARIO C. DIONEDA
Before us is a Petition for Review1 under Rule 45 (DIONEDA), found out, while he was about to
of the Rules of Court, challenging the June 4, 2001 cause the release of the said cargoes, that the
Decision2 and the September 21, 2001 same were stored only in a room with two (2) air
Resolution3 of the Court of Appeals (CA) in CA-GR conditioners running, to cool the place instead of a
CV No. 58208. The assailed Decision disposed as refrigerator.
follows:
Thereafter, DIONEDA, upon instructions from
"WHEREFORE, premises considered, the present GETC, did not proceed with the withdrawal of the
appeal is hereby DISMISSED for lack of merit. The vaccines and instead, samples of the same were
appealed Decision of Branch 149 of the Regional taken and brought to the Bureau of Animal Industry
Trial Court of Makati City in Civil Case No. 95- of the Department of Agriculture in the Philippines
1219, entitled 'American Home Assurance Co. and by SMITHKLINE for examination.
PHILAM Insurance Co., Inc. v. FEDERAL
EXPRESS CORPORATION and/or CARGOHAUS, As a consequence of the foregoing result of the
INC. (formerly U-WAREHOUSE, INC.),' is hereby veterinary biologics test, SMITHKLINE abandoned
AFFIRMED and REITERATED. the shipment and, declaring total loss for the
"Costs against the [petitioner and Cargohaus, unusable shipment, filed a claim with AHAC
Inc.]." The assailed Resolution denied petitioner's through its representative in the Philippines, the
Motion for Reconsideration. Philam Insurance Co., Inc. (PHILAM) which
recompensed SMITHKLINE for the whole insured
amount
Digest: Federal Express Corporation vs.
American Home Assurance Company [GR $39,339.00. Thereafter, respondents filed an action
150094, 18 August 2004] PARTIES INVOLVED: for damages against the petitioner imputing
Petitioner is Federal Express Corporation negligence on either or both of them in the handling
Respondent is American Home Assurance of the cargo.
Company & Philam Insurance Company Inc. ISSUE: W/N there is legal subrogation on the part
of AHAC/PHILAM (YES)
HOW THE CASE STARTED:
HELD: It should be noted that The Certificate
On January 26, 1994, SMITHKLINE Beecham specifies that loss of or damage to the insured
(SMITHKLINE) of Nebraska, USA delivered to cargo is “payable to order x x x upon surrender of
Burlington Air Express, an agent of FEDEX, a this Certificate.” Such wording conveys the right of
shipment of 109 cartons of veterinary biologicals collecting on any such damage or loss, as fully as if
for delivery to consignee SMITHKLINE and French the property were covered by a special policy in the
Overseas Company in Makati City, Metro Manila. name of the holder itself. At the back of the
Certificate appears the signature of the
The shipment was covered by Burlington Airway representative of Burlington. This document has
Bill No. 11263825 with the words, “REFRIGERATE thus been duly indorsed in blank and is deemed a
WHEN NOT IN TRANSIT” and “PERISHABLE” bearer instrument. Since the Certificate was in the
stamp marked on its face. possession of SMITHKLINE, the latter had the right
of collecting or of being indemnified for loss of or
BURLINGTON insured the cargoes in the amount damage to the insured shipment, as fully as if the
of $39,339.00 with American Home Assurance property were covered by a special policy in the
9
name of the holder. Hence, being the holder of the No medical examination shall be required for
Certificate and having an insurable interest in the amounts of insurance up to P50,000.00. However,
goods, SMITHKLINE was the proper payee of the a declaration of good health shall be required for all
insurance proceeds. Upon payment to the Lot Purchasers as part of the application. The
consignee of an indemnity for the loss of or Company reserves the right to require further
damage to the insured goods, the insurer's evidence of insurability satisfactory to the Company
entitlement to subrogation pro tanto -- being of the in respect of the following:
highest equity -- equips it with a cause of action in
case of a contractual breach or negligence. 1. Any amount of insurance in excess of
"Further, the insurer's subrogatory right to sue for P50,000.00.
recovery under the bill of lading in case of loss of or 2. Any lot purchaser who is more than 55 years of
damage to the cargo is jurisprudentially upheld." In age.
the exercise of its subrogatory right, an insurer may
proceed against an erring carrier. To all intents and LIFE INSURANCE BENEFIT.
purposes, it stands in the place and in substitution The Life Insurance coverage of any Lot Purchaser
of the consignee. A fortiori, both the insurer and the at any time shall be the amount of the unpaid
consignee are bound by the contractual balance of his loan (including arrears up to but not
stipulations... under the bill of lading. exceeding 2 months) as reported by the Assured to
the Company or the sum of P100,000.00,
whichever is smaller. Such benefit shall be paid to
9. G.R. NO. 166245 : April 9, 2008] the Assured if the Lot Purchaser dies while insured
ETERNAL GARDENS MEMORIAL PARK under the Policy.
CORPORATION, Petitioner, v. THE PHILIPPINE EFFECTIVE DATE OF BENEFIT.
AMERICAN LIFE INSURANCE
COMPANY, Respondent. The insurance of any eligible Lot Purchaser shall
be effective on the date he contracts a loan with
The Case the Assured. However, there shall be no insurance
Central to this Petition for Review if the application of the Lot Purchaser is not
on Certiorari under Rule 45 which seeks to reverse approved by the Company.
and set aside the November 26, 2004 Decision 1 of Eternal was required under the policy to submit to
the Court of Appeals (CA) in CA-G.R. CV No. Philamlife a list of all new lot purchasers, together
57810 is the query: May the inaction of the insurer with a copy of the application of each purchaser,
on the insurance application be considered as and the amounts of the respective unpaid balances
approval of the application? of all insured lot purchasers. In relation to the
instant petition, Eternal complied by submitting a
letter dated December 29, 1982, containing a list of
On December 10, 1980, respondent Philippine insurable balances of its lot buyers for October
American Life Insurance Company (Philamlife) 1982. One of those included in the list as “new
entered into an agreement denominated as business” was a certain John Chuang. His balance
Creditor Group Life Policy No. P-19202 with of payments was PhP100,000. On August 2, 1984,
petitioner Eternal Gardens Memorial Park Chuang died. Eternal sent a letter dated August 20,
Corporation (Eternal). Under the policy, the clients 1984 to Philamlife, which served as an insurance
of Eternal who purchased burial lots from it on claim for Chuang’s death. Attached to the claim
installment basis would be insured by Philamlife. were the following documents: (1) Chuang’s
The amount of insurance coverage depended upon Certificate of Death; (2) Identification Certificate
the existing balance of the purchased burial lots. stating that Chuang is a naturalized Filipino Citizen;
The policy was to be effective for a period of one (3) Certificate of Claimant; (4) Certificate of
year, renewable on a yearly basis. Attending Physician; and (5) Assured’s Certificate.
In reply, Philamlife wrote Eternal a letter on
November 12, 1984, requiring Eternal to submit the
The relevant provisions of the policy are: following documents relative to its insurance claim
ELIGIBILITY. for Chuang’s death: (1) Certificate of Claimant (with
form attached); (2) Assured’s Certificate (with form
Any Lot Purchaser of the Assured who is at least attached); (3) Application for Insurance
18 but not more than 65 years of age, is indebted accomplished and signed by the insured, Chuang,
to the Assured for the unpaid balance of his loan while still living; and (4) Statement of Account
with the Assured, and is accepted for Life showing the unpaid balance of Chuang before his
Insurance coverage by the Company on its death. Eternal transmitted the required documents
effective date is eligible for insurance under the through a letter dated November 14, 1984, which
Policy. was received by Philamlife on November 15, 1984.
EVIDENCE OF INSURABILITY. After more than a year, Philamlife had not
furnished Eternal with any reply to the latter’s
10
insurance claim. This prompted Eternal to demand against the insurer and liberally in favor of the
from Philamlife the payment of the claim for insured, especially to avoid forfeiture.
PhP100,000 on April 25, 1986.
Clearly, the vague contractual provision, in Creditor
Group Life Policy No. P-1920 dated December 10,
1980, must be construed in favor of the insured
ISSUE: WON THE INSURER IS LIABLE.  and in favor of the effectivity of the insurance
contract.

HELD: YES. As earlier stated, Philamlife and On the other hand, the seemingly conflicting
Eternal entered into an agreement denominated as provisions must be harmonized to mean that upon
Creditor Group Life Policy No. P-1920 dated a party’s purchase of a memorial lot on installment
December 10, 1980. In the policy, it is provided from Eternal, an insurance contract covering the lot
that: purchaser is created and the same is effective,
valid, and binding until terminated by Philamlife by
EFFECTIVE DATE OF BENEFIT. disapproving the insurance application. The second
The insurance of any eligible Lot Purchaser shall sentence of Creditor Group Life Policy No. P-1920
be effective on the date he contracts a loan with on the Effective Date of Benefit is in the nature of a
the Assured. However, there shall be no insurance resolutory condition which would lead to the
if the application of the Lot Purchaser is not cessation of the insurance contract. Moreover, the
approved by the Company. mere inaction of the insurer on the insurance
application must not work to prejudice the insured;
An examination of the above provision would show it cannot be interpreted as a termination of the
ambiguity between its two sentences. The first insurance contract. The termination of the
sentence appears to state that the insurance insurance contract by the insurer must be explicit
coverage of the clients of Eternal already became and unambiguous.
effective upon contracting a loan with Eternal while
the second sentence appears to require Philamlife As a final note, to characterize the insurer and the
to approve the insurance contract before the same insured as contracting parties on equal footing is
can become effective. inaccurate at best. Insurance contracts are wholly
prepared by the insurer with vast amounts of
It must be remembered that an insurance contract experience in the industry purposefully used to its
is a contract of adhesion which must be construed advantage. More often than not, insurance
liberally in favor of the insured and strictly against contracts are contracts of adhesion containing
the insurer in order to safeguard the latter’s technical terms and conditions of the industry,
interest. Thus, in Malayan Insurance Corporation confusing if at all understandable to laypersons,
v. Court of Appeals, this Court held that: that are imposed on those who wish to avail of
insurance. As such, insurance contracts are
Indemnity and liability insurance policies are
imbued with public interest that must be considered
construed in accordance with the general rule of
whenever the rights and obligations of the insurer
resolving any ambiguity therein in favor of the
and the insured are to be delineated. Hence, in
insured, where the contract or policy is prepared by
order to protect the interest of insurance applicants,
the insurer. A contract of insurance, being a
insurance companies must be obligated to act with
contract of adhesion, par excellence, any ambiguity
haste upon insurance applications, to either deny
therein should be resolved against the insurer; in
or approve the same, or otherwise be bound to
other words, it should be construed liberally in favor
honor the application as a valid, binding, and
of the insured and strictly against the insurer.
effective insurance contract.
Limitations of liability should be regarded with
extreme jealousy and must be construed in such a
way as to preclude the insurer from noncompliance
with its obligations. (Emphasis supplied.) TECcHA II. CONTRACT OF INSURANCE

In the more recent case of Philamcare Health


Systems, Inc. v. Court of Appeals, we reiterated the 1. ENRIQUEZ VS. SUN LIFE INSURANCE
above ruling, stating that: CANADA, 1920
When the terms of insurance contract contain G.R. No. L-15895 November 29, 1920
limitations on liability, courts should construe them
in such a way as to preclude the insurer from non- RAFAEL ENRIQUEZ, as administrator of the
compliance with his obligation. Being a contract of estate of the late Joaquin Ma. Herrer, plaintiff-
adhesion, the terms of an insurance contract are to appellant,
be construed strictly against the party which vs. SUN LIFE ASSURANCE COMPANY OF
prepared the contract, the insurer. By reason of the CANADA, defendant-appellee.
exclusive control of the insurance company over
the terms and phraseology of the insurance
contract, ambiguity must be strictly interpreted
11
This is an action brought by the plaintiff ad  Rafael Enriquez, as administrator of the
administrator of the estate of the late Joaquin Ma. estate of the late Joaquin Ma. Herrer filed to
Herrer to recover from the defendant life insurance recover from Sun Life Assurance Company
company the sum of pesos 6,000 paid by the of Canada through its office in Manila for a
deceased for a life annuity. The trial court gave life annuity
judgment for the defendant. Plaintiff appeals.
 RTC: favored Sun Life Insurance
The undisputed facts are these: On September 24,
1917, Joaquin Herrer made application to the Sun ISSUE: W/N Mr. Herrera received notice of
Life Assurance Company of Canada through its acceptance of his application thereby perfecting his
office in Manila for a life annuity. Two days later he life annuity
paid the sum of P6,000 to the manager of the
company's Manila office and was given a receipt HELD: NO. Judgment is reversed, and the
reading as follows: Enriquez shall have and recover from the Sun Life
the sum of P6,000 with legal interest from
November 20, 1918, until paid, without special
PROVISIONAL RECEIPT Pesos 6,000 Lessons finding as to costs in either instance. So ordered.
Applicable: Perfection (Insurance)
Art. 1319. Consent is manifested by the meeting of
FACTS: the offer and the acceptance upon the thing and
the cause which are to constitute the contract. The
 September 24, 1917: Joaquin Herrer made offer must be certain and the acceptance absolute.
application to the Sun Life Assurance A qualified acceptance constitutes a counter-offer.
Company of Canada through its office in Acceptance made by letter or telegram does not
Manila for a life annuity bind the offerer except from the time it came to his
 2 days later: he paid P6,000 to the manager knowledge. The contract, in such a case, is
of the company's Manila office and was presumed to have been entered into in the place
given a receipt where the offer was made.

 according to the provisional receipt, 3 things  not perfected because it has not been
had to be accomplished by the insurance proved satisfactorily that the acceptance of
company before there was a contract:  the application ever came to the knowledge
of the applicant.
 (1) There had to be a medical
examination of the applicant; -check SC: We hold that the contract for a life annuity in
the case at bar was not perfected because it has
 (2) there had to be approval of the not been proved satisfactorily that the acceptance
application by the head office of the of the application ever came to the knowledge of
company; and - check the applicant.
 (3) this approval had in some way to Judgment is reversed, and the plaintiff shall have
be communicated by the company to and recover from the defendant the sum of P6,000
the applicant - ? with legal interest from November 20, 1918, until
 November 26, 1917: The head office at paid, without special finding as to costs in either
Montreal, Canada gave notice of instance.
acceptance by cable to Manila but this was
not mailed
 December 4, 1917: policy was issued at
Montreal 2. DEVELOPMENT BANK VS. CA, 1994

 December 18, 1917: attorney Aurelio A. G.R. No. L-109937 March 21, 1994
Torres wrote to the Manila office of the DEVELOPMENT BANK OF THE
company stating that Herrer desired to PHILIPPINES, petitioner,
withdraw his application vs. COURT OF APPEALS and the ESTATE OF
 December 19, 1917: local office replied to THE LATE JUAN B. DANS, represented by
Mr. Torres, stating that the policy had been CANDIDA G. DANS, and the DBP MORTGAGE
issued, and called attention to the REDEMPTION INSURANCE POOL, respondents.
notification of November 26, 1917
 December 21, 1917 morning: received by This is a petition for review on certiorari under Rule
Mr. Torres 45 of the Revised Rules of Court to reverse and set
 December 20, 1917: Mr. Herrer died aside the decision of the Court of Appeals in CA-
G.R CV No. 26434 and its resolution denying
reconsideration thereof.
12
We affirm the decision of the Court of Appeals with deducting its agent's commission and service fee.
modification. The liability of an agent who exceeds the scope of
his authority depends upon whether the third
FACTS: person is aware of the limits of the agent's powers.
In May 1987, Juan B. Dans, together with his wife There is no showing that Dans knew of the
Candida, his son and daughter-in-law, applied for a limitation on DBP's authority to solicit applications
loan of P500,000.00 with the DBP. As the principal for MRI. If the third person dealing with an agent is
mortgagor, Dans, then 76 years of age, was unaware of the limits of the authority conferred by
advised by DBP to obtain a mortgage redemption the principal on the agent and he (third person) has
insurance (MRI) A loan, in the reduced amount of been deceived by the non-disclosure thereof by the
P300,000, was approved by the DBP and Dans agent, then the latter is liable for damages to him.
accomplished and submitted the application for the Last paragraph from fulltext:
MRI. The MRI premium of Dans, less the DBP
service fee of 10 percent, was credited by DBP to WHEREFORE, the decision of the Court of
the savings account of the DBP MRI Pool. On Appeals in CA G.R.-CV
September 1987, Dans died. The DBP MRI pool No. 26434 is MODIFIED and petitioner DBP is
then informed DBP that Dans was not eligible for ORDERED: (1) to REIMBURSE respondent Estate
the MRI coverage because the acceptance limit of Juan B. Dans the amount of P1,476.00 with legal
was 60 years at the time of application. Candida interest from the date of the filing of the complaint
Dans filed a case against DBP and the DBP MRI until fully paid; and (2) to PAY said Estate the
for the collection of sum of money with damages, amount of Fifty Thousand Pesos (P50,000.00) as
alleging that Dans became insured by the DBP moral damages and the amount of Ten Thousand
MRI Pool when DBP, with full knowledge of Dans' Pesos (P10,000.00) as attorney's fees. With costs
age at the time of application, required him to apply against petitioner.
for MRI, and later collected the insurance premium
thereon. The DBP was found to be liable. The DBP
MRI, on the other hand, was found by the trial court 3. GREAT PACIFIC LIFE ASSURANCE
to have no privity of contract between the it and the CO. VS. CA, 1979
deceased.
G.R. No. L-31845 April 30, 1979
ISSUE: Was DBP estopped for having led Dans
into applying for MRI and actually collecting the GREAT PACIFIC LIFE ASSURANCE
premium and the service fee, despite knowledge of COMPANY, petitioner,
his age ineligibility vs. HONORABLE COURT OF
APPEALS, respondents.
G.R. No. L-31878 April 30, 1979 LAPULAPU D.
HELD: In dealing with Dans, DBP was wearing two MONDRAGON, petitioner,
legal hats: the first as a lender, and the second as vs. HON. COURT OF APPEALS and NGO
an insurance agent. As an insurance agent, DBP HING, respondents.
made Dans go through the motion of applying for
said insurance, thereby leading him and his family The two above-entitled cases were ordered
to believe that they had already fulfilled all the consolidated by the Resolution of this Court dated
requirements for the MRI and that the issuance of April 29, 1970, (Rollo, No. L-31878, p. 58),
their policy was forthcoming. Apparently, DBP had because the petitioners in both cases seek similar
full knowledge that Dan's application was never relief, through these petitions for certiorari by way
going to be approved. The maximum age for MRI of appeal, from the amended decision of
acceptance is 60 years as clearly and specifically respondent Court of Appeals which affirmed in toto
provided in Article 1 of the Group Mortgage the decision of the Court of First Instance of Cebu,
Redemption Insurance Policy signed in 1984 by all ordering "the defendants (herein petitioners Great
the insurance companies concerned (Exh. "1- Pacific Life Assurance Company and Mondragon)
Pool"). Under Article 1987 of the Civil Code of the jointly and severally to pay plaintiff (herein private
Philippines, "the agent who acts as such is not respondent Ngo Hing) the amount of P50,000.00
personally liable to the party with whom he with interest at 6% from the date of the filing of the
contracts, unless he expressly binds himself or complaint, and the sum of P1,077.75, without
exceeds the limits of his authority without giving interest.
such party sufficient notice of his powers." The
DBP is not authorized to accept applications for
MRI when its clients are more than 60 years of age Facts: Ngo Hing filed an application with the Great
(Exh. "1-Pool"). Knowing all the while that Dans Pacific for a twenty-year endowment policy in the
was ineligible for MRI coverage because of his amount of P50,000.00 on the life of his one-year
advanced age, DBP exceeded the scope of its old daughter Helen. He supplied the essential data
authority when it accepted Dan's application for which petitioner Mondragon, the Branch Manager,
MRI by collecting the insurance premium, and wrote on the form. The latter paid the annual
premium the sum of P1,077.75 going over to the
13
Company, but he retained the amount of P1,317.00 Pacific Life disapproved the insurance application
as his commission for being a duly authorized of respondent Ngo Hing, the binding deposit receipt
agent of Pacific Life. in question had never become in force at any time.
The binding deposit receipt is conditional and does
Upon the payment of the insurance premium, the not insure outright. This was held in Lim v Sun.
binding deposit receipt was issued Ngo Hing.
Likewise, petitioner Mondragon handwrote at the The deposit paid by private respondent shall have
bottom of the back page of the application form his to be refunded by Pacific Life.
strong recommendation for the approval of the
insurance application. Then Mondragon received a 2.  Ngo Hing had deliberately concealed the state
letter from Pacific Life disapproving the insurance of health of his daughter Helen Go. When he
application. The letter stated that the said life supplied data, he was fully aware that his one-year
insurance application for 20-year endowment plan old daughter is typically a mongoloid child. He
is not available for minors below seven years old, withheld the fact material to the risk insured.
but Pacific Life can consider the same under the “The contract of insurance is one of perfect good
Juvenile Triple Action Plan, and advised that if the faith uberrima fides meaning good faith, absolute
offer is acceptable, the Juvenile Non-Medical and perfect candor or openness and honesty; the
Declaration be sent to the company. absence of any concealment or demotion, however
The non-acceptance of the insurance plan by slight.”
Pacific Life was allegedly not communicated by The concealment entitles the insurer to rescind the
petitioner Mondragon to private respondent Ngo contract of insurance.
Hing. Instead, on May 6, 1957, Mondragon wrote
back Pacific Life again strongly recommending the
approval of the 20-year endowment insurance plan III. INSURABLE INTEREST
to children, pointing out that since the customers
were asking for such coverage.
Helen Go died of influenza. Ngo Hing sought the 1. Spouses Cha vs CA (August 18, 1997) G.R.
payment of the proceeds of the insurance, but No. 124520  August 18, 1997
having failed in his effort, he filed the action for the
recovery before the Court of First Instance of Cebu,
which ruled against him. FACTS:
Issues:  Spouses Nilo Cha and Stella Uy-Cha
and CKS Development Corporation entered
1. Whether the binding deposit receipt constituted a
a 1 year lease contract with a stipulation not
temporary contract of the life insurance in question
to insure against fire the chattels,
2. Whether Ngo Hing concealed the state of health merchandise, textiles, goods and effects
and physical condition of Helen Go, which placed at any stall or store or space in the
rendered void the policy leased premises without first obtaining the
written consent and approval of the lessor.
Held:  No. Yes. Petition dismissed. But it insured against loss by fire their
Ratio: The receipt was intended to be merely a merchandise inside the leased premises for
provisional insurance contract. Its perfection was P500,000 with the United Insurance Co.,
subject to compliance of the following conditions: Inc. without the written consent of CKS
(1) that the company shall be satisfied that the  On the day the lease contract was to expire,
applicant was insurable on standard rates; (2) that fire broke out inside the leased premises
if the company does not accept the application and and CKS learning that the spouses procured
offers to issue a policy for a different plan, the an insurance wrote to United to have the
insurance contract shall not be binding until the proceeds be paid directly to them. But
applicant accepts the policy offered; otherwise, the United refused so CKS filed against
deposit shall be refunded; and (3) that if the Spouses Cha and United.
company disapproves the application, the
insurance applied for shall not be in force at any  RTC: United to pay CKS the amount of
time, and the premium paid shall be returned to the P335,063.11 and Spouses Cha to pay
applicant. P50,000 as exemplary damages, P20,000
as attorney’s fees and costs of suit
The receipt is merely an acknowledgment that the
latter's branch office had received from the  CA: deleted exemplary damages and
applicant the insurance premium and had accepted attorney’s fees
the application subject for processing by the
ISSUE: W/N the CKS has insurable interest
insurance company. There was still approval or
because the spouses Cha violated the stipulation
rejection the same on the basis of whether or not
the applicant is "insurable on standard rates." Since
14
HELD: NO. CA set aside. Awarding the proceeds trade, goods in process and/or inventories only
to spouses Cha. hereby insured, and unless notice be given and the
particulars of such insurance or insurances be
 Sec. 18.  No contract or policy of insurance stated therein or endorsed in this policy pursuant to
on property shall be enforceable except for Section 50 of the Insurance Code, by or on behalf
the benefit of some person having an of the Company before the occurrence of any loss
insurable interest in the property insured or damage, all benefits under this policy shall be
 A non-life insurance policy such as the fire deemed forfeited, provided however, that this
insurance policy taken by petitioner-spouses condition shall not apply when the total insurance
over their merchandise is primarily a or insurances in force at the time of the loss or
contract of indemnity.  Insurable interest in damage is not more than P200,000.00."
the property insured must exist a t the time The petitioners’ stocks were destroyed by fire. He
the insurance takes effect and at the time then filed a claim which was subsequently denied
the loss occurs.  The basis of such because the petitioner’s stocks were covered by
requirement of insurable interest in property two other fire insurance policies for Php 200,000
insured is based on sound public policy: to issued by PFIC. The basis of the private
prevent a person from taking out an respondent's denial was the petitioner's alleged
insurance policy on property upon which he violation of Condition 3 of the policy.
has no insurable interest and collecting the
proceeds of said policy in case of loss of the Geagonia then filed a complaint against the private
property.  In such a case, the contract of respondent in the Insurance Commission for the
insurance is a mere wager which is void recovery of P100,000.00 under fire insurance
under Section 25 of the Insurance Code. policy and damages. He claimed that he knew the
existence of the other two policies. But, he said that
 SECTION 25.  Every stipulation in a policy he had no knowledge of the provision in the private
of Insurance for the payment of loss, respondent's policy requiring him to inform it of the
whether the person insured has or has not prior policies and this requirement was not
any interest in the property insured, or that mentioned to him by the private respondent's
the policy shall be received as proof of such agent.
interest, and every policy executed by way
of gaming or wagering, is void The Insurance Commission found that the
petitioner did not violate Condition 3 as he had no
 Section 17.  The measure of an insurable knowledge of the existence of the two fire
interest in property is the extent to which the insurance policies obtained from the PFIC; that it
insured might be damnified by loss of injury was Cebu Tesing Textiles w/c procured the PFIC
thereof policies w/o informing him or securing his consent;
 The automatic assignment of the policy to and that Cebu Tesing Textile, as his creditor, had
CKS under the provision of the lease insurable interest on the stocks.
contract previously quoted is void for being The Insurance Commission then ordered the
contrary to law and/or public policy.  The respondent company to pay complainant the sum
proceeds of the fire insurance policy thus of P100,000.00 with interest and attorney’s fees.
rightfully belong to the spouses.  The liability
of the Cha spouses to CKS for violating their CA reversed the decision of the Insurance
lease contract in that Cha spouses obtained Commission because it found that the petitioner
a fire insurance policy over their own knew of the existence of the two other policies
merchandise, without the consent of CKS, is issued by the PFIC.
a separate and distinct issue which we do
not resolve in this case. The chief issues that crop up from the first and third
grounds are (a) whether the petitioner had prior
knowledge of the two insurance policies issued by
the PFIC when he obtained the fire insurance
2. Geagonia vs CA February 6, 1995 policy from the private respondent, thereby, for not
Facts: disclosing such fact, violating Condition 3 of the
policy, and (b) if he had, whether he is precluded
Geagonia, owner of a store, obtained from Country from recovering therefrom.
Bankers fire insurance policy for P100,000.00. The
1 year policy and covered thestock trading of dry Issues:
goods. 1. WON the petitioner had not disclosed the two
The policy noted the requirement that insurance policies when he obtained the fire
insurance and thereby violated Condition 3 of the
"3. The insured shall give notice to the Company policy.
of any insurance or insurances already effected, or
which may subsequently be effected, covering any 2. WON he is prohibited from recovering
of the property or properties consisting of stocks in
15
Held: Yes. No. Petition Granted former policies was not fatal to the petitioner's right
to recover on the private respondent's policy.
Ratio:
Furthermore, by stating within Condition 3 itself that
1. The court agreed with the CA that the petitioner such condition shall not apply if the total insurance
knew of the prior policies issued by the PFIC. His in force at the time of loss does not exceed
letter of 18 January 1991 to the private respondent P200,000.00, the private respondent was
conclusively proves this knowledge. His testimony amenable to assume a co-insurer's liability up to a
to the contrary before the Insurance Commissioner loss not exceeding P200,000.00. What it had in
and which the latter relied upon cannot prevail over mind was to discourage over-insurance. Indeed,
a written admission made ante litem motam. It was, the rationale behind the incorporation of "other
indeed, incredible that he did not know about the insurance" clause in fire policies is to prevent over-
prior policies since these policies were not new or insurance and thus avert the perpetration of fraud.
original. When a property owner obtains insurance policies
2. Stated differently, provisions, conditions or from two or more insurers in a total amount that
exceptions in policies which tend to work a exceeds the property's value, the insured may have
forfeiture of insurance policies should be construed an inducement to destroy the property for the
most strictly against those for whose benefits they purpose of collecting the insurance. The public as
are inserted, and most favorably toward those well as the insurer is interested in preventing a
against whom they are intended to operate. situation in which a fire would be profitable to the
insured.32
With these principles in mind, Condition 3 of the
subject policy is not totally free from ambiguity and WHEREFORE, the instant petition is hereby
must be meticulously analyzed. Such analysis GRANTED. The decision of the Court of Appeals in
leads us to conclude that (a) the prohibition applies CA-G.R. SP No. 31916 is SET ASIDE and the
only to double insurance, and (b) the nullity of the decision of the Insurance Commission in Case No.
policy shall only be to the extent exceeding 3340 is REINSTATED.
P200,000.00 of the total policies obtained.
Furthermore, by stating within Condition 3 itself that 3. RCBC vs CA 289 SCRA 292
such condition shall not apply if the total insurance
in force at the time of loss does not exceed FACTS:
P200,000.00, the private respondent was
amenable to assume a co-insurer's liability up to a
 RCBC Binondo Branch initially granted a
loss not exceeding P200,000.00. What it had in
credit facility of P30M to Goyu & Sons, Inc.
mind was to discourage over-insurance. Indeed,
GOYU’s applied again and through Binondo
the rationale behind the incorporation of "other
Branch key officer's Uy’s and Lao’s
insurance" clause in fire policies is to prevent over-
recommendation, RCBC’s executive
insurance and thus avert the perpetration of fraud.
committee increased its credit facility to
When a property owner obtains insurance policies
P50M to P90M and finally to P117M.
from two or more insurers in a total amount that
exceeds the property's value, the insured may have  As security, GOYU executed 2 real
an inducement to destroy the property for the estate mortgages and 2 chattel
purpose of collecting the insurance. The public as mortgages in favor of RCBC.  
well as the insurer is interested in preventing a
situation in which a fire would be profitable to the  GOYU obtained in its name 10
insured. insurance policy on the mortgaged
properties from Malayan Insurance
The first conclusion is supported by the portion of Company, Inc. (MICO). In February
the condition referring to other insurance "covering 1992, he was issued 8 insurance
any of the property or properties consisting of policies in favor of RCBC.
stocks in trade, goods in process and/or inventories
only hereby insured," and the portion regarding the  April 27, 1992: One of GOYU’s factory
insured's declaration on the subheading CO- buildings was burned so he claimed against
INSURANCE that the co-insurer is Mercantile MICO for the loss who denied contending
Insurance Co., Inc. in the sum of P50,000.00. A that the insurance policies were
double insurance exists where the same person is either attached pursuant to writs of
insured by several insurers separately in respect of attachments/garnishments or that creditors
the same subject and interest. As earlier stated, the are claiming to have a better right
insurable interests of a mortgagor and a mortgagee  GOYU filed a complaint for specific
on the mortgaged property are distinct and performance and damages at the RTC
separate. Since the two policies of the PFIC do not
cover the same interest as that covered by the  RCBC, one of GOYU’s creditors, also filed
policy of the private respondent, no double with MICO its formal claim over the
insurance exists. The non-disclosure then of the proceeds of the insurance policies, but said
16
claims were also denied for the same  The two courts below erred in failing to see
reasons that MICO denied GOYU’s claims that the promissory notes which they ruled
should be excluded for bearing dates which
 RTC: Confirmed GOYU’s other creditors are after that of  the fire, are mere renewals
(Urban Bank, Alfredo Sebastian, and of previous ones
Philippine Trust Company) obtained their
writs of attachment covering an aggregate  RCBC has the right to claim the insurance
amount of P14,938,080.23 and ordered that proceeds, in substitution of the property lost
10 insurance policies be deposited with the in the fire. Having assigned its rights, GOYU
court minus the said amount so MICO lost its standing as the beneficiary of the
deposited P50,505,594.60.   said insurance policies
 Another Garnishment of P8,696,838.75 was  insurance company to be held liable for
handed down unreasonably delaying and withholding
payment of insurance proceeds, the delay
 RTC: favored GOYU against MICO for the must be wanton, oppressive, or malevolent -
claim, RCBC for damages and to pay RCBC not shown
its loan
 Sebastian’s right as attaching creditor must
 CA: Modified by increasing the damages in yield to the preferential rights of RCBC over
favor of GOYU the Malayan insurance policies as first
 In G.R. No. 128834, RCBC seeks right to mortgagee.
intervene in the action between Alfredo C.
Sebastian (the creditor) and GOYU (the
debtor), where the subject insurance policies 4. Gaisano Cagayan, Inc vs Insurance
were attached in favor of Sebastian Company of North America GR No.
147839 June 8, 2006
 RTC and CA: endorsements do not bear the
signature of any officer of GOYU concluded
that the endorsements favoring RCBC as
defective. FACTS:
 Intercapitol Marketing Corporation (IMC) is
the maker of Wrangler Blue Jeans.
ISSUE: W/N RCBC as mortgagee, has any right while Levi Strauss (Phils.) Inc. (LSPI) is the
over the insurance policies taken by GOYU, the local distributor of products bearing
mortgagor, in case of the occurrence of loss trademarks owned by Levi Strauss & Co
 IMC and LSPI separately obtained from
Insurance Company of North America fire
HELD: YES. insurance policies for their book debt
 mortgagor and a mortgagee have separate endorsements related to their ready-made
and distinct insurable interests in the same clothing materials which have been sold or
mortgaged property, such that each one of delivered to various customers and dealers
them may insure the same property for his of the Insured anywhere in the Philippines
own sole benefit which are unpaid 45 days after the time of
the loss
 although it appears that GOYU obtained the
subject insurance policies naming itself as  February 25, 1991: Gaisano Superstore
the sole payee, the intentions of the parties Complex in Cagayan de Oro City, owned by
as shown by their contemporaneous acts, Gaisano Cagayan, Inc., containing the
must be given due consideration in order to ready-made clothing materials sold and
better serve the interest of justice and equity delivered by IMC and LSPI was consumed
by fire. 
 8 endorsement documents were
prepared by Alchester in favor of  February 4, 1992: Insurance Company of
RCBC North America filed a complaint for damages
against Gaisano Cagayan, Inc. alleges that
 MICO, a sister company of RCBC IMC and LSPI filed their claims under their
 GOYU continued to enjoy the benefits respective fire insurance policies which it
of the credit facilities extended to it by paid thus it was subrogated to their rights
RCBC.   Gaisano Cagayan, Inc: not be held
 GOYU is at the very least liable because it was destroyed due
estopped from assailing their to fortuities event or force majeure
operative effects. 

17
 RTC: IMC and LSPI retained ownership of  it is sufficient that the insured is so
the delivered goods until fully paid, it must situated with reference to the property
bear the loss (res perit domino) that he would be liable to loss should
it be injured or destroyed by the peril
 CA: Reversed - sales invoices is an against which it is insured
exception under Article 1504 (1) of the Civil
Code to res perit domino  an insurable interest in property does
not necessarily imply a property
ISSUE: W/N Insurance Company of North America interest in, or a lien upon, or
can claim against Gaisano Cagayan for the debt possession of, the subject 
that was isnured
 matter of the insurance, and neither
HELD: YES. petition is partly GRANTED. order to the title nor a beneficial interest is
pay P535,613 is DELETED requisite to the existence of such an
interest
 insurance policy is clear that the subject of
the insurance is the book debts and  insurance in this case is not for loss of
NOT goods sold and delivered to the goods by fire but for petitioner's accounts
customers and dealers of the insured with IMC and LSPI that remained unpaid 45
days after the fire - obligation is pecuniary in
 ART. 1504. Unless otherwise agreed, the nature
goods remain at the seller's risk until the
ownership therein is transferred to the  obligor should be held exempt from
buyer, but when the ownership therein is liability when the loss occurs thru a
transferred to the buyer the goods are at the fortuitous event only holds true when
buyer's risk whether actual delivery has the obligation consists in the delivery
been made or not, except that: of a determinate thing and there is no
stipulation holding him liable even in
case of fortuitous event
(1) Where delivery of the goods has been made to  Article 1263 of the Civil Code in an
the buyer or to a bailee for the buyer, in pursuance obligation to deliver a generic thing, the loss
of the contract and the ownership in the goods has or destruction of anything of the same kind
been retained by the seller merely to secure does not extinguish  the obligation (Genus
performance by the buyer of his obligations under nunquan perit)
the contract, the goods are at the buyer's risk from
the time of such delivery;  The subrogation receipt, by itself, is
sufficient to establish not only the
 IMC and LSPI did not lose complete interest relationship of respondent as insurer and
over the goods. They have an insurable IMC as the insured, but also the amount
interest until full payment of the value of the paid to settle the insurance claim
delivered goods. Unlike the civil law concept
of res perit domino, where ownership is the  Art. 2207. If the plaintiff's property has been
basis for consideration of who bears the risk insured, and he has received indemnity from
of loss, in property insurance, one's interest the insurance company for the injury or loss
is not determined by concept of title, but arising out of the wrong or breach of
whether insured has substantial economic contract complained of, the insurance
interest in the property company shall be subrogated to the rights of
the insured against the wrongdoer or the
 Section 13 of our Insurance Code defines person who has violated the contract. 
insurable interest as "every interest in
property, whether real or personal, or any  As to LSPI, no subrogation receipt was
relation thereto, or liability in respect thereof, offered in evidence. 
of such nature that a contemplated peril
might directly damnify the insured."  Failure to substantiate the claim of
Parenthetically, under Section 14 of the subrogation is fatal to petitioner's
same Code, an insurable interest in property case for recovery of the amount of
may consist in: (a) an existing interest; (b) P535,613
an inchoate interest founded on existing
interest; or (c) an expectancy, coupled with
an existing interest in that out of which the IV. DEVICES FOR ASCERTAINING AND
expectancy arises.  CONTROLLING RISK AND LOSS

 Anyone has an insurable interest in property


who derives a benefit from its existence or 1. GREAT PACIFIC LIFE ASSURANCE
would suffer loss from its destruction. COMPANY, petitioner, vs. HONORABLE

18
COURT OF APPEALS, respondents. was when things were in such state that on May
G.R. No. L-31845 April 30, 1979 28, 1957 Helen Go died of influenza with
complication of bronchopneumonia. Thereupon,
LAPULAPU D. MONDRAGON, petitioner, vs. Ngo Hing sought the payment of the proceeds of
HON. COURT OF APPEALS and NGO HING, the insurance, but having failed in his effort, he
respondents. G.R. No. L-31878 April 30, 1979 filed the action for the recovery of the same
J. DE CASTRO before the CFI of Cebu, which rendered the
adverse decision.
The Case:
Issues:
The two above-entitled cases were ordered
consolidated by the Resolution of SC, because (1) whether the binding deposit receipt
the petitioners in both cases sought similar constituted a temporary contract of the life
relief, through these petitions for certiorari by insurance in question; and
way of appeal, from the amended decision of (2) whether private respondent Ngo Hing
the CA which affirmed in toto the decision of the concealed the state of health and physical
CFI of Cebu, ordering Great Pacific Life condition of Helen Go, which rendered void the
Assurance Company and Mondragon jointly and aforesaid deposit receipt.
severally to pay Ngo Hing the amount of
P50,000.00 with interest at 6% from the date of
the filing of the complaint, and the sum of
P1,077.75, without interest. Held:

Facts: The provisions printed on the deposit receipt


provide that the binding deposit receipt is
Ngo Hing filed an application with the Great intended to be merely a provisional or temporary
Pacific Assurance Company (Grepalife) for a insurance contract and only upon compliance of
twenty-year endowment policy in the amount of the following conditions:
P50,000.00 on the life of his one-year old
daughter Helen Go. Said respondent supplied (1) that the company shall be satisfied that the
the essential data which petitioner Lapulapu D. applicant was insurable on standard rates; (2)
Mondragon, Branch Manager of the Grepalife in that if the company does not accept the
Cebu City wrote on the corresponding form in application and offers to issue a policy for a
his own handwriting. Mondragon finally type- different plan, the insurance contract shall be
wrote the data on the application form which binding until the applicant accepts the policy
was signed by private respondent Ngo Hing. offered; otherwise, the deposit shall be
The latter paid the annual premuim to the refunded; and (3) that if the applicant is not able
Company, but he retained a certain amount as according to the standard rates, and the
his commission for being a duly authorized company disapproves the application, the
agent of Grepalife. Upon the payment of the insurance applied for shall not be in force at any
insurance premium, the binding deposit receipt time, and the premium paid shall be returned to
was issued to private respondent Ngo Hing. the applicant. Clearly implied from the aforesaid
Likewise, petitioner Mondragon handwrote at conditions is that the binding deposit receipt in
the bottom of the back page of the application question is merely an acknowledgment, on
form his strong recommendation for the behalf of the company, that the latter's branch
approval of the insurance application. office had received from the applicant the
insurance premium and had accepted the
Then Mondragon received a letter from application subject for processing by the
Grepalife disapproving the insurance insurance company; and that the latter will either
application. The letter stated that the said life approve or reject the same on the basis of
insurance application for 20-year endowment whether or not the applicant is "insurable on
plan is not available for minors below seven standard rates." Since petitioner Grepalife
years old, but Grepalife can consider the same disapproved the insurance application of
under the Juvenile Triple Action Plan, and respondent Ngo Hing, the binding deposit
advised that if the offer is acceptable, the receipt in question had never become in force at
Juvenile Non-Medical Declaration be sent to the any time. Upon this premise, the binding deposit
company. receipt is, manifestly, merely conditional and
does not insure outright. As held by this Court,
The non-acceptance of the insurance plan by where an agreement is made between the
Grepalife was allegedly not communicated by applicant and the agent, no liability shall attach
Mondragon to Ngo Hing. Instead, Mondragon until the principal approves the risk and a receipt
wrote back Grepalife again strongly is given by the agent. The acceptance is merely
recommending the approval of the 20-year conditional and is subordinated to the act of the
endowment insurance plan to children, pointing company in approving or rejecting the
out that since 1954 the customers, especially application. Thus, in life insurance, a "binding
the Chinese, were asking for such coverage. It
19
slip" or "binding receipt" does not insure by itself The decision appealed from was SET ASIDE,
(De Lim vs. Sun Life Assurance Company of and in lieu thereof, one was entered which
Canada, 41 Phil. 264). absolved petitioners Lapulapu D. Mondragon
and Great Pacific Life Assurance Company from
As held in De Lim vs. Sun Life Assurance their civil liabilities as found by respondent Court
Company of Canada, supra, "a contract of and ordered the aforesaid insurance company to
insurance, like other contracts, must be reimburse the amount of P1,077.75, without
assented to by both parties either in person or interest, to private respondent, Ngo Hing. Costs
by their agents ... The contract, to be binding against private respondent.
from the date of the application, must have been
a completed contract, one that leaves nothing to
be done, nothing to be completed, nothing to be
passed upon, or determined, before it shall take
effect. There can be no contract of insurance 2. SUNLIFE ASSURANCE COMPANY OF
unless the minds of the parties have met in CANADA vs. COURT OF APPEALS
agreement." G.R. No. 105135, 22 June 1995

2. Relative to the second issue of alleged FACTS:


concealment, the Court is of the firm belief that
 
private respondent had deliberately concealed
Robert John Bacani procured a life insurance
the state of health and physical condition of his
daughter Helen Go. Where private respondent contract for himself from petitioner-company,
supplied the required essential data for the designating his mother Bernarda Bacani, herein
insurance application form, he was fully aware private respondent, as the beneficiary. He was
that his one-year old daughter is typically a issued a policy valued at P100,000.00 with
mongoloid child. Such a congenital physical double indemnity in case of accidental death.
defect could never be ensconced nor disguised. Sometime after, the insured died in a plane
Nonetheless, private respondent, in apparent crash. Bernarda filed a claim with petitioner,
bad faith, withheld the fact material to the risk to seeking the benefits of the insurance policy
be assumed by the insurance company. As an taken by her son. However, said insurance
insurance agent of Grepalife, he ought to know, company rejected the claim on the ground that
as he surely must have known. his duty and the insured did not disclose material facts
responsibility to such a material fact. Had he
relevant to the issuance of the policy, thus
diamond said significant fact in the insurance
application form Grepalife would have verified rendering the contract of insurance voidable.
the same and would have had no choice but to Petitioner discovered that two weeks prior to his
disapprove the application outright. application for insurance, the insured was
examined and confined at the Lung Center of
The contract of insurance is one of perfect good the Philippines, where he was diagnosed for
faith uberrima fides meaning good faith, renal failure. The RTC, as affirmed by the CA,
absolute and perfect candor or openness and
this fact was concealed, as alleged by the
honesty; the absence of any concealment or
demotion, however slight [Black's Law petitioner. But the fact that was concealed was
Dictionary, 2nd Edition], not for the alone but not the cause of death of the insured and that
equally so for the insurer (Field man's Insurance matters relating to the medical history of the
Co., Inc. vs. Vda de Songco, 25 SCRA 70). insured is deemed to be irrelevant since
Concealment is a neglect to communicate that petitioner waived the medical examination prior
which a party knows and ought to communicate to the approval and issuance of the insurance
(Section 25, Act No. 2427). Whether intentional policy.
or unintentional the concealment entitles the  
insurer to rescind the contract of insurance ISSUE: Whether or not the concealment of such
(Section 26, Id.: Yu Pang Cheng vs. Court of material fact, despite it not being the cause of
Appeals, et al, 105 Phil 930; Satumino vs. death of the insured, is sufficient to render the
Philippine American Life Insurance Company, 7
insurance contract voidable
SCRA 316). Private respondent appears guilty
 
thereof.
HELD:
The SC held that no insurance contract was  
perfected between the parties with the YES. Section 26 of the Insurance Code is
noncompliance of the conditions provided in the explicit in requiring a party to a contract of
binding receipt, and concealment, as legally insurance to communicate to the other, in good
defined, having been committed by herein faith, all facts within his knowledge which are
private respondent.
material to the contract and as to which he
Disposition: makes no warranty, and which the other has no

20
means of ascertaining. Anent the finding that the RTC: ruled in favor of Julita; ordered Philamcare
facts concealed had no bearing to the cause of to pay and reimburse the medical expenses +
death of the insured, it is well settled that the Moral Damages + exemplary damages.
insured need not die of the disease he had
CA: affirmed RTC decision in favor of Julita but
failed to disclose to the insurer. It is sufficient
removed the damages.
that his non-disclosure misled the insurer in
forming his estimates of the risks of the
proposed insurance policy or in making ISSUES:
inquiries. The SC, therefore, ruled that petitioner
(1) Whether or not a health care agreement is
properly exercised its right to rescind the
an insurance contract.
contract of insurance by reason of the
concealment employed by the insured. It must (2) Whether or not the alleged concealment
be emphasized that rescission was exercised avoided the agreement.
within the two-year contestability period as
recognized in Section 48 of The Insurance
Code. WHEREFORE, the petition is GRANTED HELD:
and the Decision of the Court of Appeals is (1) YES.
REVERSED and SET ASIDE.
Section 2 (1) of the Insurance Code defines a
contract of insurance as an agreement whereby
3. PHILAMCARE HEALTH SYSTEMS, one undertakes for a consideration to indemnify
INC., petitioner, another against loss, damage or liability arising
vs. COURT OF APPEALS and JULITA from an unknown or contingent event. An
TRINOS, respondents. G.R. No. 125678 insurance contract exists where the following
March 18, 2002 elements concur:

Ernani Trinos, deceased, husband of 1. The insured has an insurable interest;


respondent Julita Trinos applied for a health 2. The insured is subject to a risk of loss by
care coverage with petitioner Philamcare Health the happening of the designated peril;
Systems Inc. In the application form, he
answered “NO” to the question – “Have you or 3. The insurer assumes the risk;
any family member ever consulted or been 4. Such assumption of risk is part of a
treated for high blood pressure, heart trouble, general scheme to distribute actual
diabetes, cancer, liver disease, asthma or peptic losses among a large group of persons
ulcer?” bearing a similar risk; and
The application for was approved for a period of 5. In consideration of the insurers promise,
one year, from March 1, 1988 to March 1, 1989, the insured pays a premium
and he was issued Health Care Agreement,
wherein the husband Ernani was entitled to avail Section 3 of the Insurance Code states that any
of hospitalization benefits; out patient benefits contingent or unknown event, whether past or
such as annual physical examinations and future, which may damnify a person having an
preventive health cared. insurable interest against him, may be insured
against. Every person has an insurable interest
The agreement was extended with the coverage in the life and health of himself. Section 10
amount increasing to P75,000 per disability. provides:
During the period, Ernani suffered a heart attack
and was confined in Manila Medical Center Every person has an insurable interest in the life
(MMC) for one month. While he was at the and health:
hospital, Julita tried to claim the benefits under 1. of himself, of his spouse and of his
the health care agreement, but Philamcare children;
denied liability saying that such was void.
Apparently, Ernani concealed his medical 2. of any person on whom he depends
history since the Manila Medical doctors wholly or in part for education or support,
discovered that he was hypertensive, diabetic or in whom he has a pecuniary interest;
and asthmatic, which was contrary to his answer
3. of any person under a legal obligation to
in the application form. Julita paid the expenses
him for the payment of money, respecting
amounting to P76,000.
property or service, of which death or
illness might delay or prevent the
performance; and
21
4. of any person upon whose life any estate the agreement or whenever he avails of the
or interest vested in him depends. covered benefits which he has prepaid.
In the case at bar, the insurable interest of Under Section 27 of the Insurance Code, a
respondents husband in obtaining the health concealment entitles the injured party to rescind
care agreement was his own health. The health a contract of insurance. The right to rescind
care agreement was in the nature of non-life should be exercised previous to the
insurance, which is primarily a contract of commencement of an action on the contract. In
indemnity. Once the member incurs hospital, this case, no rescission was made. Besides,
medical or any other expense arising from the cancellation of health care agreements as in
sickness, injury or other stipulated contingent, insurance policies require the concurrence of
the health care provider must pay for the same the following conditions:
to the extent agreed upon under the contract.
 Prior notice of cancellation to insured;

(2) NO.  Notice must be based on the occurrence


after effective date of the policy of one or
The question relating to an applicant’s medical more of the grounds mentioned;
history largely depends on opinion, rather than
fact, especially coming from Ernani, who was  Must be in writing, mailed or delivered to
not a medical doctor. Where matters of opinion the insured at the address shown in the
are called for, answers made in good faith and policy;
without intent to deceive will not avoid a policy  Must state the grounds relied upon
even though they are untrue. provided in Section 64 of the Insurance
Thus, although false, a representation of the Code and upon request of insured, to
expectation, intention, belief, opinion, or furnish facts on which cancellation is
judgment of the insured will not avoid the policy based
if there is no actual fraud in inducing the None of the above pre-conditions was fulfilled in
acceptance of the risk, or its acceptance at a this case. When the terms of insurance contract
lower rate of premium, and this is likewise the contain limitations on liability, courts should
rule although the statement is material to the construe them in such a way as to preclude the
risk, if the statement is obviously of the insurer from non-compliance with his obligation.
foregoing character, since in such case the Being a contract of adhesion, the terms of an
insurer is not justified in relying upon such insurance contract are to be construed strictly
statement, but is obligated to make further against the party which prepared the contract
inquiry. There is a clear distinction between the insurer. By reason of the exclusive control of
such a case and one in which the insured is the insurance company over the terms and
fraudulently and intentionally states to be true, phraseology of the insurance contract, ambiguity
as a matter of expectation or belief, that which must be strictly interpreted against the insurer
he then knows, to be actually untrue, or the and liberally in favor of the insured, especially to
impossibility of which is shown by the facts avoid forfeiture. This is equally applicable to
within his knowledge, since in such case the Health Care Agreements. The phraseology used
intent to deceive the insurer is obvious and in medical or hospital service contracts, such as
amounts to actual fraud. the one at bar, must be liberally construed in
The fraudulent intent on the part of the insured favor of the subscriber, and if doubtful or
must be established to warrant rescission of the reasonably susceptible of two interpretations the
insurance contract. Concealment as a defense construction conferring coverage is to be
for the health care provider or insurer to avoid adopted, and exclusionary clauses of doubtful
liability is an affirmative defense and the duty to import should be strictly construed against the
establish such defense by satisfactory and provider
convincing evidence rests upon the provider or As to incontestability:
insurer. In any case, with or without the authority
to investigate, petitioner is liable for claims made Under the title Claim procedures of expenses,
under the contract. Having assumed a the Philamcare Health Systems Inc. had twelve
responsibility under the agreement, petitioner is months from the date of issuance of the
bound to answer the same to the extent agreed Agreement within which to contest the
upon. In the end, the liability of the health care membership of the patient if he had previous
provider attaches once the member is ailment of asthma, and six months from the
hospitalized for the disease or injury covered by issuance of the agreement if the patient was
22
sick of diabetes or hypertension. The periods Pacific did not depend upon the state of mind of
having expired, the defense of concealment or Jaime Canilang. A man’s state of mind or
misrepresentation no longer lie. subjective belief is not capable of proof in our
judicial process, except through proof of external
As to Julita being the proper claimant:
acts or failure to act from which inferences as to
Finally, petitioner alleges that respondent was his subjective belief may be reasonably drawn.
not the legal wife of the deceased member Neither does materiality depend upon the actual
considering that at the time of their marriage, the or physical events which ensure. Materiality
deceased was previously married to another relates rather to the “probable and reasonable
woman who was still alive. The health care influence of the facts” upon the party to whom
agreement is in the nature of a contract of the communication should have been made, in
indemnity. Hence, payment should be made to assessing the risk involved in making or omitting
the party who incurred the expenses. She paid to make further inquiries and in accepting the
all the hospital and medical expenses. She is application for insurance; that “probable and
therefore entitled to reimbursement. reasonable influence of the facts” concealed
must, of course, be determined objectively, by
the judge ultimately. WHEREFORE, the Petition
4. THELMA VDA. DE CANILANG vs. for Review is DENIED for lack of merit and the
COURT OF APPEALS G.R. No. 92492, Decision of the Court of Appeals dated 16
17 June 1993 October 1989 in C.A.-G.R. SP No. 08696 is
hereby AFFIRMED. No pronouncement as to
FACTS: the costs.

Jaime Canilang applied for a “non-medical”


insurance policy with respondent Great Pacific 5. TAN V. COURT OF APPEALS, G.R. NO.
Life Assurance Company naming his wife, L-48049, [JUNE 29, 1989], 256 PHIL
Thelma Canilang as his beneficiary. But he did 158-166)
not disclose the fact that he was diagnosed as
suffering from sinus tachycardia and that he has FACTS: Petitioners appeal from the Decision of
consulted a doctor twice. Jaime was issued an the Insurance Commissioner dismissing herein
ordinary life insurance policy with the face value petitioners’ complaint against respondent
of P19,700.00. Jaime died of “congestive heart Philippine American Life Insurance Company for
failure”, “anemia”, and “chronic anemia”. the recovery of the proceeds of Policy No.
Petitioner widow and beneficiary of the insured, 1082467 in the amount of P80,000.00.
filed a claim with Great Pacific which the insurer
“On September 23, 1973, Tan Lee Siong, father
denied upon the ground that the insured had
of herein petitioners, applied for life insurance in
concealed material information from it. Hence,
the amount of P80,000.00 with respondent
Thelma filed a complaint against Great Pacific
company. Said application was approved and
with the Insurance Commission for recovery of
Policy No. 1082467 was issued effective
the insurance proceeds.
November 6, 1973, with petitioners the
ISSUE: Whether or not the non-disclosure of beneficiaries thereof (Exhibit A).
certain facts about the insured’s previous health
“On April 26, 1975, Tan Lee Siong died of
conditions is material to warrant the denial of the
hepatoma (Exhibit B). Petitioners then filed with
claims of Thelma Canilang.
respondent company their claim for the
HELD: YES. The SC agreed with the Court of proceeds of the life insurance policy. However,
Appeals that the information which Jaime in a letter dated September 11, 1975,
Canilang failed to disclose was material to the respondent company denied petitioners’ claim
ability of Great Pacific to estimate the probable and rescinded the policy by reason of the
risk he presented as a subject of life insurance. alleged misrepresentation and concealment of
Had Canilang disclosed his visits to his doctor, material facts made by the deceased Tan Lee
the diagnosis made and medicines prescribed Siong in his application for insurance (Exhibit 3).
by such doctor, in the insurance application, it The premiums paid on the policy were
may be reasonably assumed that Great Pacific thereupon refunded.
would have made further inquiries and would
“Alleging that respondent company’s refusal to
have probably refused to issue a non-medical
pay them the proceeds of the policy was
insurance policy or, at the very least, required a
unjustified and unreasonable, petitioners filed on
higher premium for the same coverage. The
November 27, 1975, a complaint against the
materiality of the information withheld by Great
23
former with the Office of the Insurance designed in such a way as to conceal from the
Commissioner, docketed as I.C. Case No. 218. applicant their importance. If a warning in bold
red letters or a boxed warning similar to that
“After hearing the evidence of both parties, the
required for cigarette advertisements by the
Insurance Commissioner rendered judgment on
Surgeon General of the United States is
August 3, 1977, dismissing petitioners’
necessary, that is for Congress or the Insurance
complaint.”
Commission to provide as protection against
The petitioners contend that the respondent high pressure insurance salesmanship. We are
company no longer had the right to rescind the limited in this petition to ascertaining whether or
contract of insurance as rescission must not the respondent Court of Appeals committed
allegedly be done during the lifetime of the reversible error. It is the petitioners’ burden to
insured within two years and prior to the show that the factual findings of the respondent
commencement of action. court are not based on substantial evidence or
that its conclusions are contrary to applicable
ISSUE: WON THE INSURER IS LIABLE.  law and jurisprudence. They have failed to
HELD: NO. The so-called “incontestability discharge that burden.
clause” precludes the insurer from raising the
defenses of false representations or
concealment of material facts insofar as health 6. PRUDENTIAL GUARANTEE AND
and previous diseases are concerned if the ASSURANCE INC. v. TRANS-ASIA SHIPPING
insurance has been in force for at least two LINES, INC.
years during the insured’s lifetime. The phrase
G.R. NO. 151890; June 20, 2006
“during the lifetime” found in Section 48 simply
means that the policy is no longer considered in
force after the insured has died. The key phrase
FACTS:
in the second paragraph of Section 48 is “for a
period of two years.” TRANS-ASIA is the owner of the vessel M/V
Asia Korea. In consideration of payment of
The insurer has two years from the date of
premiums, PRUDENTIAL insured M/V Asia
issuance of the insurance contract or of its last
Korea for loss/damage of the hull and machinery
reinstatement within which to contest the policy,
arising from perils, inter alia, of fire and
whether or not, the insured still lives within such
explosion for the sum of P40 Million, beginning
period. After two years, the defenses of
on July 1, 1993 up to July 1, 1994. On October
concealment or misrepresentation, no matter
25, 1993, while the policy was in force, a fire
how patent or well founded, no longer lie.
broke out while M/V Asia Korea was undergoing
Congress felt this was a sufficient answer to the
repairs at the port of Cebu. On October 26, 1993
various tactics employed by insurance
plaintiff TRANS-ASIA filed its notice of claim for
companies to avoid liability. The petitioners’
damage sustained by the vessel. On August 13,
interpretation would give rise to the incongruous
1996, the adjuster, Richards Hogg International,
situation where the beneficiaries of an insured
Inc. completed is survey report determining the
who dies right after taking out and paying for a
amount of loss sustained by TRANS-ASIA.
life insurance policy, would be allowed to collect
PRUDENTIAL denied the claim of TRANS-ASIA
on the policy even if the insured fraudulently
for alleged breach of insurance policy
concealed material facts.b
conditions, that TRANS-ASIA violated an
The petitioners argue that no evidence was express and material warranty in the insurance
presented to show that the medical terms were contract – that M/V ASIA KOREA is required to
explained in a layman’s language to the insured. be CLASSED AND CLASS MAINTAINED.
They state that the insurer should have
ISSUE:
presented its two medical field examiners as
witnesses. Moreover, the petitioners allege that Whether or not TRANS-ASIA violated a material
the policy intends that the medical examination warranty in the insurance contract that the
must be conducted before its issuance vessel should be classed and class maintained.
otherwise the insurer “waives whatever
imperfection by ratification.” HELD:

There is no showing that the questions in the NO. PRUDENTIAL failed to establish that
application form for insurance regarding the TRANS-ASIA violated and breached the policy
insured’s medical history are in smaller print condition on WARRANTED VESSEL CLASSED
than the rest of the printed form or that they are
24
AND CLASS MAINTAINED, as contained in the for 853 pieces of logs equivalent to
subject insurance contract. 695,548 board feet. The total cargo insured
under the two marine policies consisted of 1,395
We are not unmindful of the clear language of
logs, or the equivalent of 1,195.498 bd. ft.
Sec. 74 of the Insurance Code which provides
that, "the violation of a material warranty, or After the issuance of the cover note, but before
other material provision of a policy on the part of the issuance of the two marine policies Nos. 53
either party thereto, entitles the other to HO 1032 and 53 HO 1033, some of the logs
rescind." It is generally accepted that "[a] intended to be exported were lost during loading
warranty is a statement or promise set forth in operations in the Diapitan Bay.
the policy, or by reference incorporated therein,
While the logs were alongside the vessel, bad
the untruth or non-fulfillment of which in any
weather developed resulting in 75 pieces of logs
respect, and without reference to whether the
which were rafted together co break loose from
insurer was in fact prejudiced by such untruth or
each other. 45 pieces of logs were salvaged, but
non-fulfillment, renders the policy voidable by
30 pieces were verified to have been lost or
the insurer." However, it is similarly indubitable
washed away as a result of the accident.
that for the breach of a warranty to avoid a
policy, the same must be duly shown by the Pacific Timber informed Workmen’s about the
party alleging the same. We cannot sustain an loss of 32 pieces of logs during loading of SS
allegation that is unfounded. Consequently, woodlock.
PRUDENTIAL, not having shown that TRANS-
ASIA breached the warranty condition, Although dated April 4, 1963, the letter was
CLASSED AND CLASS MAINTAINED, it received in the office of the defendant only on
remains that TRANS-ASIA must be allowed to April 15, 1963. The plaintiff claimed for
recover its rightful claims on the policy. insurance to the value of P19,286.79.
Woodmen’s requested an adjustment company
to assess the damage. It submitted its report,
V. POLICY where it found that the loss of 30 pieces of logs
is not covered by Policies Nos. 53 HO 1032 and
1. Pacific Timber Export Corporation vs
1033 but within the 1,250,000 bd. ft. covered by
CA (112 SCRA 199)
Cover Note 1010 insured for $70,000.00.
The adjustment
This petition seeks the review of the decision of company submitted a computation of the
the Court of Appeals reversing the decision of defendant's probable liability on the loss
the Court of First Instance of Manila in favor of sustained by the shipment, in the total amount of
petitioner and against private respondent which P11,042.04.
ordered the latter to pay the sum of Pll,042.04
Woodmen’s wrote the plaintiff denying the
with interest at the rate of 12% interest from
latter's claim on the ground they defendant's
receipt of notice of loss on April 15, 1963 up to
investigation revealed that the entire shipment of
the complete payment, the sum of P3,000.00 as
logs covered by the two marine policies were
attorney's fees and the costs 1 thereby
received in good order at their point
dismissing petitioner s complaint with costs.
of destination. It was further stated that the said
loss may be considered as covered under Cover
Note No. 1010 because the said Note had
Facts:
become null and void by virtue of the issuance
The plaintiff secured temporary insurance from of Marine Policy Nos. 53 HO 1032 and 1033.
the defendant for its exportation of
The denial of the claim by the defendant was
1,250,000 board feet of Philippine Lauan and
brought by the plaintiff to the attention of the
Apitong logs to be shipped from Quezon
Insurance Commissioner. The Insurance
Province to Okinawa and Tokyo, Japan.
Commissioner ruled in favor of indemnifying
Workmen’s Insurance issued a cover note Pacific Timber. The company added that the
insuring the cargo of the plaintiff subject to its cover note is null and void for lack of valuable
terms and conditions. consideration. The trial court ruled in petitioner’s
favor while the CA dismissed the case. Hence
The two marine policies bore the numbers 53 this appeal.
HO 1032 and 53 HO 1033. Policy No. 53 H0
1033 was for 542 pieces of logs equivalent to
499,950 board feet. Policy No. 53 H0 1033 was
25
Issues: whether delay would be a valid ground of
objection.
WON the cover note was null and void for lack
of valuable consideration There was enough time for insurer to determine
if petitioner was guilty of delay in communicating
WON the Insurance company was absolved
the loss to respondent company. It never did in
from responsibility due to unreasonable delay in
the Insurance Commission. Waiver can be
giving notice of loss.
raised against it under Section 84 of the
Insurance Act.

Held: No. No. Judgment reversed.


Section 84.—Delay in the presentation to an
insurer of notice or proof of loss is waived if
Ratio: caused by any act of his or if he omits to take
1. The fact that no separate premium was paid objection promptly and specifically upon that
on the Cover Note before the loss occurred ground.
does not militate against the validity of the From what has been said, We find duly
contention even if no such premium was paid. substantiated petitioner's assignments of error.
All Cover Notes do not contain particulars of the
shipment that would serve as basis for ACCORDINGLY, the appealed decision is set
the computation of the premiums. Also, no aside and the decision of the Court of First
separate premiums are required to be paid on a Instance is reinstated in toto with the affirmance
Cover Note. of this Court.

The petitioner paid in full all the premiums,


hence there was no account unpaid on the
2. Great Pacific Life Assurance Cor. vs
insurance coverage and the cover note. If the
CA (89 SCRA 543)
note is to be treated as a separate policy instead
of integrating it to the regular policies, the Facts:
purpose of the note would be meaningless. It is
Private respondent Ngo Hing filed an application
a contract, not a mere application for insurance.
with the Great Pacific Life Assurance Company
It may be true that the marine insurance policies for a twenty-year endowment policy on the life of
issued were for logs no longer including those his one-year old daughter Helen Go. Petitioner
which had been lost during loading operations. Mondragon, branch manager of Pacific Life
This had to be so because the risk insured handwrote at the bottom of the back page of the
against is for loss during transit, because the application form his strong recommendation for
logs were safely placed aboard. the approval of the insurance application.
Mondragon received a letter from Pacific Life
The non-payment of premium on the Cover Note
disapproving the insurance application. The
is, therefore, no cause for the petitioner to lose
letter stated that the said life insurance
what is due it as if there had been payment of
application for 20-year endowment plan is not
premium, for non-payment by it was not
available for minors below seven years old, but
chargeable against its fault. Had all the logs
Pacific Life can consider the same under the
been lost during the loading operations, but after
Juvenile Triple Action Plan, and advised that if
the issuance of the Cover Note, liability on the
the offer is acceptable, the Juvenile Non-
note would have already arisen even before
Medical Declaration be sent to the company.
payment of premium. Otherwise, the note would
serve no practical purpose in the realm of
commerce, and is supported by the doctrine that
The non-acceptance of the insurance plan by
where a policy is delivered without requiring
Pacific Life was allegedly not communicated by
payment of the premium, the presumption is that
petitioner Mondragon to private respondent Ngo
a credit was intended and policy is valid.
Hing. Mondragon wrote back Pacific Life again
2. The defense of delay can’t be sustained. The strongly recommending the approval of the 20-
facts show that instead of invoking the ground of year endowment insurance plan.
delay in objecting to petitioner's claim of
recovery on the cover note, the insurer never
had this in its mind. It has a duty to inquire when On May 28, 1957 Helen Go died of influenza
the loss took place, so that it could determine with complication of bronchopneumonia.
Thereupon, private respondent sought the
26
payment of the proceeds of the insurance, but 3. Philam Life and General Insurance Co. vs
having failed in his effort. Judge Valencia-Bagalacsa Regional
Trial Court of Libmanan, Camarines
Sur, Branch 56, and EDUARDO Z.
Issues: Whether or not there is a perfected LUMANIOG, CELSO Z. LUMANIOG and
insurance contract between the parties. RUBEN Z. LUMANIOG, respondents
(GR No 139776; August 1, 2002

Ruling:
Before us is a petition for review on certiorari
Pacific Life disapproved the insurance under Rule 45 of the Rules of Court. Petitioner
application in question on the ground that it is Philippine American Life and General Insurance
not offering the twenty-year endowment Company prays that the decision of the Court of
insurance policy to children less than seven Appeals promulgated on April 30, 1999 be
years of age. What it offered instead is another reversed and set aside and that the Complaint
plan known as the Juvenile Triple Action, which filed against it by private respondents Eduardo
private respondent failed to accept. In the Z. Lumaniog, Celso Z. Lumaniog and Ruben Z.
absence of a meeting of the minds between Lumaniog before the Regional Trial Court of
petitioner Pacific Life and private respondent Libmanan, Camarines Sur, docketed as Civil
Ngo Hing over the 20-year endowment life Case No. L-787 be ordered dismissed on
insurance in the amount of P50,000.00 in favor ground of prescription of action.
of the latter's one-year old daughter, and with
the non-compliance of the conditions stated in
the disputed binding deposit receipt, there could
Facts:
have been no insurance contract duly perfected
between then Accordingly, the deposit paid by On June 20, 1995, private respondents, as
private respondent shall have to be refunded by legitimate children and forced heirs of their late
Pacific Life. father, Faustino Lumaniog, filed with the
aforesaid RTC, a complaint for recovery of sum
of money against petitioner alleging that: their
A contract of insurance, like other contracts, father was insured by petitioner under Life
must be assented to by both parties either in Insurance Policy No. 1305486 with a face value
person or by their agents. The contract, to be of P50,000.00; their father died of "coronary
binding from the date of the application, must thrombosis" on November 25, 1980; on June 22,
have been a completed contract, one that 1981, they claimed and continuously claimed for
leaves nothing to be done, nothing to be all the proceeds and interests under the life
completed, nothing to be passed upon, or insurance policy in the amount of P641,000.00,
determined, before it shall take effect. There can despite repeated demands for payment and/or
be no contract of insurance unless the minds of settlement of the claim due from petitioner, the
the parties have met in agreement. last of which is on December 1, 1994, petitioner
finally refused or disallowed said claim on
We are thus constrained to hold that no February 14, 1995; and so, they filed their
insurance contract was perfected between the complaint on June 20, 1995.
parties with the noncompliance of the conditions
provided in the binding receipt, and
concealment, as legally defined, having been
Petitioner filed an Answer with Counterclaim
comraitted by herein private respondent.
and Motion to Dismiss, contending that: the
WHEREFORE, the decision appealed from is cause of action of private respondents had
hereby set aside, and in lieu thereof, one is prescribed and they are guilty of laches; it had
hereby entered absolving petitioners Lapulapu denied private respondents’ claim in a letter
D. Mondragon and Great Pacific Life Assurance dated March 12, 1982, signed by its then
Company from their civil liabilities as found by Assistant Vice President, Amado Dimalanta, on
respondent Court and ordering the aforesaid ground of concealment on the part of the
insurance company to reimburse the amount of deceased insured Faustino when he asserted in
P1,077.75, without interest, to private his application for insurance coverage that he
respondent, Ngo Hing had not been treated for indication of "chest
pain, palpitation, high blood pressure, rheumatic
fever, heart murmur, heart attack or other
disorder of the heart or blood vessel" when in
27
fact he was a known hypertensive since 1974; erroneous and arbitrary, thus, depriving
private respondents sent a letter dated May petitioner of due process, as discussed earlier.
25,1983 requesting for reconsideration of the
WHEREFORE, the petition is partly GRANTED.
denial; in a letter dated July 11, 1983, it
The assailed decision of the Court of Appeals
reiterated its decision to deny the claim for
dated April 30, 1999 insofar only as it upheld the
payment of the proceeds; more than ten (10)
Order dated December 12, 1997 is REVERSED
years later, or on December 1, 1994, it received
and SET ASIDE. A new judgment is entered
a letter from Jose C. Claro, a provincial board
reversing and setting aside the Order dated
member of the province of Camarines Sur,
December 12, 1997 of the Regional Trial Court
reiterating the early request for reconsideration
of Libmanan, Camarines Sur (Branch 56) and
which it denied in a letter dated February 14,
affirming its Order dated June 20, 1995. Said
1995.
RTC is directed to proceed with dispatch with
RTC ruled that the 10 year period has not Civil Case No. L-787.
prescribed.
Issue:
Makati Tuscany Condominium Corporation vs.
WON the 10 year prescriptive period had Court of Appeals
elapsed and no cause of action can further be
[GR 95546, 6 November 1992] First Division,
asserted.
Bellosillo (J): 3 concur, 1 on leave
Held:
Yes. The ruling of the RTC that the cause of
Facts:
action of private respondents had not
prescribed, is arbitrary and patently erroneous Sometime in early 1982, American Home
for not being founded on evidence on record, Assurance Co. (AHAC), represented by
and therefore, the same is void. American International Underwriters (Phils.),
Inc., (AIUI) issued in favor of Makati Tuscany
It based its finding on a mere explanation of the
Condominium Corporation (Tuscany) Insurance
private respondents’ counsel and not on
Policy AH-CPP-9210452 on the latter's building
evidence presented by the parties as to the date
and premises, for a period beginning 1 March
when to reckon the prescriptive period.
1982 and ending 1 March 1983, with a total
Ruling: premium of P466,103.05. The premium was
paid on installments on 12 March 1982, 20 May
It must be emphasized that petitioner had
1982, 21 June 1982 and 16 November 1982, all
specifically alleged in the Answer that it had
of which were accepted by AHAC. On 10
denied private respondents’ claim per its letter
February 1983, AHAC issued to Tuscany
dated July 11, 1983. Hence, due process
Insurance Policy No. AH-CPP-9210596, which
demands that it be given the opportunity to
replaced and renewed the previous policy, for a
prove that private respondents had received
term covering 1 March 1903 to 1 March 1984.
said letter, dated July 11, 1983. Said letter is
The premium in the amount of P466,103.05 was
crucial to petitioner’s defense that the filing of
again paid on installments on 13 April 1983, 13
the complaint for recovery of sum of money in
July 1983, 3 August 1983, 9 September 1983,
June, 1995 is beyond the 10-year prescriptive
and 21 November 1983. All payments were
period.
likewise accepted by AHAC. On 20 January
1984, the policy was again renewed and AHAC
issued to Tuscany Insurance Policy AH-
The appellate court should have granted the CPP9210651 for the period 1 March 1984 to 1
petition for certiorari assailing said Order of March 1985. On this renewed policy, Tuscany
December 12, 1997. Certiorari is an appropriate made two installment payments, both accepted
remedy to assail an interlocutory order (1) when by AHAC, the first on 6 February 1984 for
the tribunal issued such order without or in P52,000.00 and the second, on 6 June 1984 for
excess of jurisdiction or with grave abuse of P100,000.00. Thereafter, Tuscany refused to
discretion and (2) when the assailed pay the balance of the premium. Consequently,
interlocutory order is patently erroneous and the AHAC filed an action to recover the unpaid
remedy of appeal would not afford adequate and balance of P314,103.05 for Insurance Policy
expeditious relief.16 Said Order was issued with AHCPP-9210651. In its answer with
grave abuse of discretion for being patently counterclaim, Tuscany admitted the issuance of
Insurance Policy AH-CPP9210651. It explained
28
that it discontinued the payment of premiums Code in effect allows waiver by the insurer of the
because the policy did not contain a credit condition of prepayment by making an
clause in its favor and the receipts for the acknowledgment in the insurance policy of
installment payments covering the policy for receipt of premium as conclusive evidence of
1984-85, as well as the two (2) previous payment so far as to make the policy binding
policies, stated the following reservations: (2) despite the fact that premium is actually unpaid.
Acceptance of this payment shall not waive any Section 77 merely precludes the parties from
of the company rights to deny liability on any stipulating that the policy is valid even if
claim under the policy arising before such premiums are not paid, but does not expressly
payments or after the expiration of the credit prohibit an agreement granting credit extension,
clause of the policy; and (3) Subject to no loss and such an agreement is not contrary to
prior to premium payment. If there be any loss morals, good customs, public order or public
such is not covered. Tuscany further claimed policy. So is an understanding to allow insured
that the policy was never binding and valid, and to pay premiums in installments not so
no risk attached to the policy. It then pleaded a proscribed. At the very least, both parties should
counterclaim for P152,000.00 for the premiums be deemed in estoppel to question the
already paid for 1984-85, and in its answer with arrangement they have voluntarily accepted. It
amended counterclaim, sought the refund of appearing from the peculiar circumstances that
P924,206.10 representing the premium the parties actually intended to make the three
payments for 1982-85. After some incidents, (3) insurance contracts valid, effective and
Tuscany and AHAC moved for summary binding, Tuscany may not be allowed to renege
judgment. On 8 October 1987, the trial court on its obligation to pay the balance of the
dismissed the complaint and the counterclaim. premium after the expiration of the whole term of
Both parties appealed from the judgment of the the third policy (AHCPP-9210651) in March
trial court. Thereafter, the Court of Appeals 1985. Moreover, where the risk is entire and the
rendered a decision modifying that of the trial contract is indivisible, the insured is not entitled
court by ordering Tuscany to pay the balance of to a refund of the premiums paid if the insurer
the premiums due on Policy AHCPP-921-651, or was exposed to the risk insured for any period,
P314,103.05 plus legal interest until fully paid, however brief or momentary.
and affirming the denial of the counterclaim.
Tuscany filed the petition. Issue: Whether
payment by installment of the premiums due on UCPB General Insurance vs. Masagana
an insurance policy invalidates the contract of Telamart Inc. [GR 137172, 15 June 1999] First
insurance. Held: NO. The subject policies are Division, Pardo (J): 4 concur
valid even if the premiums were paid on
installments. The records clearly show that
Tuscany and AHAC intended subject insurance Facts: On 15 April 1991, UCPB General
policies to be binding and effective Insurance Co. Inc. (UCPBGen) issued 5
notwithstanding the staggered payment of the insurance policies covering Masagana Telamart,
premiums. The initial insurance contract entered Inc.'s various property described therein against
into in 1982 was renewed in 1983, then in 1984. fire, for the period from 22 May 1991 to 22 May
In those 3 years, the insurer accepted all the 1992. In March 1992, UCPBGen evaluated the
installment payments. Such acceptance of policies and decided not to renew them upon
payments speaks loudly of the insurer's intention expiration of their terms on 22 May 1992.
to honor the policies it issued to Tuscany. UCPBGen advised Masagana's broker, Zuellig
Certainly, basic principles of equity and fairness Insurance Brokers, Inc. of its intention not to
would not allow the insurer to continue collecting renew the policies. On 6 April 1992, UCPBGen
and accepting the premiums, although paid on gave written notice to Masagana of the non-
installments, and later deny liability on the lame renewal of the policies at the address stated in
excuse that the premiums were not prepaid in the policies. On 13 June 1992, fire razed
full. Thus, while the import of Section 77 is that Masagana's property covered by three of the
prepayment of premiums is strictly required as a insurance policies UCPBGen issued. On 13 July
condition to the validity of the contract, the Court 1992, Masagana presented to UCPBGen's
was not prepared to rule that the request to cashier at its head office 5 manager's checks in
make installment payments duly approved by the total amount of P225,753.95, representing
the insurer, would prevent the entire contract of premium for the renewal of the policies from 22
insurance from going into effect despite May 1992 to 22 May 1993. No notice of loss
payment and acceptance of the initial premium was filed by Masagana under the policies prior
or first installment. Section 78 of the Insurance to 14 July 1992. On 14 July 1992, Masagana
29
filed with UCPBGen its formal claim for that payment could be made later. UCPBGen
indemnification of the insured property razed by appealed.
fire. On the same day, 14 July 1992, UCPBGen
Issue: Whether the fire insurance policies issued
returned to Masagana the 5 manager's checks
by UCPBGen to the Masagana covering the
that it tendered, and at the same time rejected
period 22 May 1991 to 22 May 1992, had
Masagana's claim for the reasons (a) that the
expired on the latter date or had been extended
policies had expired and were not renewed, and
or renewed by an implied credit arrangement
(b) that the fire occurred on 13 June 1992,
though actual payment of premium was
before Masagana's tender of premium payment.
tendered on a latter date after the occurrence of
On 21 July 1992, Masagana filed with the
the risk (fire) insured against.
Regional Trial Court, Branch 58, Makati City, a
civil complaint against UCPBGen for recovery of Held: The answer is easily found in the
P18,645,000.00, representing the face value of Insurance Code. No, an insurance policy, other
the policies covering Masagana's insured than life, issued originally or on renewal, is not
property razed by fire, and for attorney's fees. valid and binding until actual payment of the
On 23 October 1992, after its motion to dismiss premium. Any agreement to the contrary is void.
had been denied, UCPBGen filed an answer to The parties may not agree expressly or impliedly
the complaint. It alleged that the complaint "fails on the extension of credit or time to pay the
to state a cause of action"; that UCPBGen was premium and consider the policy binding before
not liable to Masagana for insurance proceeds actual payment. The case of Malayan Insurance
under the policies because at the time of the Co., Inc. vs. Cruz-Arnaldo is not applicable. In
loss of Masagana's property due to fire, the that case, payment of the premium was in fact
policies had long expired and were not renewed. actually made on 24 December 1981, and the
After due trial, on 10 March 1993, the Regional fire occurred on 18 January 1982. Here, the
Trial Court, Branch 58, Makati, rendered payment of the premium for renewal of the
decision, (1) authorizing and allowing Masagana policies was tendered on 13 July 1992, a month
to consign/deposit with this Court the sum of after the fire occurred on 13 June 1992. The
P225,753.95 (refused by UCPBGen) as full assured did not even give the insurer a notice of
payment of the corresponding premiums for the loss within a reasonable time after occurrence of
replacement-renewal policies; (2) declaring the fire. Hence, the Supreme Court reversed
Masagana to have fully complied with its and set aside the decision of the Court of
obligation to pay the premium thereby rendering Appeals in CA-GR CV 42321. In lieu thereof, the
the replacement-renewal policy effective and Court rendered judgment dismissing
binding for the duration 22 May 1992 until 22 Masagana's complaint and UCPBGen's
May 1993; and, ordering UCPBGen to deliver counterclaims thereto filed with the Regional
forthwith to Masagana the said replacement- Trial Court, Branch 58, Makati City, in Civil Case
renewal policies; (3) declaring two of the policies 92-2023, without costs.
in force from 22 August 1991 up to 23 August
1992 and 9 August 1991 to 9 August 1992,
respectively; and (4) ordering UCPBGen to pay American Home Assurance Company vs. Chua
Masagana the sums of: (a) P18,645,000.00 [GR 130421, 28 June 1999] First Division,
representing the latter's claim for indemnity Davide Jr. (CJ): 4 concur
under three policies and/or its replacement-
renewal policies; (b) 25% of the total amount Facts: American Home Assurance Company
due as and for attorney's fees; (c) P25,000.00 (AHAC) is a domestic corporation engaged in
as necessary litigation expenses; and, (d) the the insurance business. Sometime in 1990,
costs of suit. In due time, UCPBGen appealed to Antonio Chua obtained from AHAC a fire
the Court of Appeals. On 7 September 1998, the insurance covering the stock-in-trade of his
Court of Appeals promulgated its decision business, Moonlight Enterprises, located at
affirming that of the Regional Trial Court with the Valencia, Bukidnon. The insurance was due to
modification that item 3 of the dispositive portion expire on 25 March 1990. On 5 April 1990 Chua
was deleted, and the award of attorney's fees issued PCIBank Check 352123 in the amount of
was reduced to 10% of the total amount due. P2,983.50 to AHAC’s agent, James Uy, as
The Court of Appeals held that following payment for the renewal of the policy. In turn,
previous practise, Masagana was allowed a 60 the latter delivered Renewal Certificate
to 90 day credit term for the renewal of its 00099047 to Chua. The check was drawn
policies, and that the acceptance of the late against a Manila bank and deposited in AHAC’s
premium payment suggested an understanding bank account in Cagayan de Oro City. The
corresponding official receipt was issued on 10
30
April. Subsequently, a new insurance policy, P100,000.00 as exemplary damages;
Policy 206-4234498-7, was issued, whereby P50,000.00 as attorney’s fees; and Cost of suit.
AHAC undertook to indemnify Chua for any On appeal, the assailed decision was affirmed in
damage or loss arising from fire up to P200,000 toto by the Court of Appeals. The Court of
for the period 25 March 1990 to 25 March 1991. Appeals found that Chua’s claim was
On 6 April 1990 Moonlight Enterprises was substantially proved and AHAC’s unjustified
completely razed by fire. Total loss was refusal to pay the claim entitled Chua to the
estimated between P4,000,000 and P5,000,000. award of damages. Its motion for
Chua filed an insurance claim with AHAC and reconsideration of the judgment having been
four other co-insurers, namely, Pioneer denied, AHAC filed the petition for review on
Insurance and Surety Corporation, Prudential certiorari.
Guarantee and Assurance, Inc., Filipino
Issue: Whether there was a valid payment of
Merchants Insurance Co. and Domestic
premium, considering that Chua’s check was
Insurance Company of the Philippines. AHAC
cashed after the occurrence of the fire.
refused to honor the claim notwithstanding
several demands by Chua, thus, the latter filed Held: YES. The general rule in insurance laws is
an action against AHAC before the trial court. In that unless the premium is paid the insurance
its defense, AHAC claimed there was no policy is not valid and binding. The only
existing insurance contract when the fire exceptions are life and industrial life insurance.
occurred since Chua did not pay the premium. It Whether payment was indeed made is a
also alleged that even assuming there was a question of fact which is best determined by the
contract, Chua violated several conditions of the trial court. The trial court found, as affirmed by
policy, particularly: (1) his submission of the Court of Appeals, that there was a valid
fraudulent income tax return and financial check payment by Chua to AHAC. Well-settled
statements; (2) his failure to establish the actual is the rule that the factual findings and
loss, which AHAC assessed at P70,000; and (3) conclusions of the trial court and the Court of
his failure to notify to AHAC of any insurance Appeals are entitled to great weight and respect,
already effected to cover the insured goods. and will not be disturbed on appeal in the
These violations, AHAC insisted, justified the absence of any clear showing that the trial court
denial of the claim. The trial court ruled in favor overlooked certain facts or circumstances which
of Chua. It found that Chua paid by way of would substantially affect the disposition of the
check a day before the fire occurred. The check, case. The Supreme Cpurt sees no reason to
which was deposited in AHAC’s bank account, depart from this ruling. According to the trial
was even acknowledged in the renewal court the renewal certificate issued to Chua
certificate issued by AHAC’s agent. It declared contained the acknowledgment that premium
that the alleged fraudulent documents were had been paid. It is not disputed that the check
limited to the disparity between the official drawn by Chua in favor of AHAC and delivered
receipts issued by the Bureau of Internal to its agent was honored when presented and
Revenue (BIR) and the income tax returns for AHAC forthwith issued its official receipt to Chua
the years 1987 to 1989. All the other documents on 10 April 1990. Section 306 of the Insurance
were found to be genuine. Nonetheless, it gave Code provides that any insurance company
credence to the BIR certification that Chua paid which delivers a policy or contract of insurance
the corresponding taxes due for the questioned to an insurance agent or insurance broker shall
years. As to Chua’s failure to notify AHAC of the be deemed to have authorized such agent or
other insurance contracts covering the same broker to receive on its behalf payment of any
goods, the trial court held that AHAC failed to premium which is due on such policy or contract
show that such omission was intentional and of insurance at the time of its issuance or
fraudulent. Finally, it noted that AHAC’s delivery or which becomes due thereon. Herein,
investigation of Chua's claim was done in the best evidence of such authority is the fact
collaboration with the representatives of other that AHAC accepted the check and issued the
insurance companies who found no irregularity official receipt for the payment. It is, as well,
therein. In fact, Pioneer Insurance and Surety bound by its agent’s acknowledgment of receipt
Corporation and Prudential Guarantee and of payment. Section 78 of the Insurance Code
Assurance, Inc. promptly paid the claims filed by explicitly provides that "An acknowledgment in a
Chua. The trial court ordered AHAC to pay Chua policy or contract of insurance of the receipt of
P200,000.00, representing the amount of the premium is conclusive evidence of its payment,
insurance, plus legal interest from the date of so far as to make the policy binding,
filing of the case; P200,000.00 as moral notwithstanding any stipulation therein that it
damages; P200,000.00 as loss of profit; shall not be binding until the premium is actually
31
paid." This Section establishes a legal fiction of damages, and attorney's fees equivalent to 20%
payment and should be interpreted as an of the total claim. On 19 July 1990 the trial court
exception to Section 77. ruled for Tibay, et al. and adjudged Fortune
liable for the total value of the insured building
and personal properties in the amount of
Tibay vs. Court of Appeals [GR 119655, 24 May P600,000.00 plus interest at the legal rate of 6%
1996] First Division, Bellosillo (J): 2 concur, 1 per annum from the filing of the complaint until
filed a separate opinion to which 1 joined full payment, and attorney's fees equivalent to
20% of the total amount claimed plus costs of
suit. On 24 March 1995 the Court of Appeals
Facts: On 22 January 1987, Fortune Life and reversed the court a quo by declaring Fortune
General Insurance Co., Inc. (Fortune) issued not to be liable to Tibay et al. but ordering
Fire Insurance Policy 136171 in favor of Violeta Fortune to return to the former the premium of
R. Tibay and/or Nicolas Roraldo on their two- P2,983.50 plus 12% interest from 10 March
storey residential building located at 5855 Zobel 1987 until full payment. Tibay, et al. filed the
Street, Makati City, together with all their petition for review.
personal effects therein. The insurance was for
P600,000.00 covering the period from 23
January 1987 to 23 January 1988. On 23 Issue: Whether a fire insurance policy be valid,
January 1987, of the total premium of binding and enforceable upon mere partial
P2,983.50, petitioner Violeta Tibay only paid payment of premium.
P600.00 thus leaving a considerable balance
unpaid. On 8 March 1987 the insured building
was completely destroyed by fire. Two days Held: NO. Insurance is a contract whereby one
later or on 10 March 1987 Violeta Tibay paid the undertakes for a consideration to indemnify
balance of the premium. On the same day, she another against loss, damage or liability arising
filed with Fortune a claim on the fire insurance from an unknown or contingent event. The
policy. Her claim was accordingly referred to its consideration is the premium, which must be
adjuster, Goodwill Adjustment Services, Inc. paid at the time and in the way and manner
(GASI), which immediately wrote Violeta specified in the policy, and if not so paid, the
requesting her to furnish it with the necessary policy will lapse and be forfeited by its own
documents for the investigation and processing terms. The Policy provides for payment of
of her claim. Petitioner forthwith complied. On premium in full. Accordingly, where the premium
28 March 1987 she signed a non-waiver has only been partially paid and the balance
agreement with GASI to the effect that any paid only after the peril insured against has
action taken by the companies or their occurred, the insurance contract did not take
representatives in investigating the claim made effect and the insured cannot collect at all on the
by the claimant for his loss which occurred at policy. This is fully supported by Section 77 of
5855 Zobel Roxas, Makati on 8 March 1987, or the Insurance Code which provides that "An
in the investigating or ascertainment of the insurer is entitled to payment of the premium as
amount of actual cash value and loss, shall not soon as the thing insured is exposed to the peril
waive or invalidate any condition of the policies insured against. Notwithstanding any agreement
of such companies held by said claimant, nor to the contrary, no policy or contract of
the rights of either or any of the parties to this insurance issued by an insurance company is
agreement, and such action shall not be, or be valid and binding unless and until the premium
claimed to be, an admission of liability on the thereof has been paid, except in the case of a
part of said companies or any of them. In a letter life or an industrial life policy whenever the
dated 11 June 1987 Fortune denied the claim of grace period provision applies." Apparently the
Violeta for violation of Policy Condition 2 and of crux of the controversy lies in the phrase "unless
Section 77 of the Insurance Code. Efforts to and until the premium thereof has been paid."
settle the case before the Insurance This leads us to the manner of payment
Commission proved futile. On 3 March 1988 envisioned by the law to make the insurance
Violeta and the other petitioners (Antonio Tibay, policy operative and binding. For whatever
Ofelia M. Roraldo, Victorina M. Roraldo, Virgilio judicial construction may be accorded the
M. Roraldo, Myrna M. Roraldo, and Rosabella disputed phrase must ultimately yield to the
M. Roraldo) sued Fortune for damages in the clear mandate of the law. The principle that
amount of P600,000.00 representing the total where the law does not distinguish the court
coverage of the fire insurance policy plus 12% should neither distinguish assumes that the
interest per annum, P100,000.00 moral legislature made no qualification on the use of a
32
general word or expression. It cannot be 1960 to 18 April 1961 or, say 271 days. On 6
disputed that premium is the elixir vitae of the July 1961, Phoenix demanded in writing for the
insurance business because by law the insurer payment of said amount. Woodworks Inc.,
must maintain a legal reserve fund to meet its through counsel, disclaimed any liability in its
contingent obligations to the public, hence, the reply-letter of 15 August 1961, contending, in
imperative need for its prompt payment and full essence, that it need not pay premium "because
satisfaction. It must be emphasized here that all the Insurer did not stand liable for any indemnity
actuarial calculations and various tabulations of during the period the premiums were not paid."
probabilities of losses under the risks insured On 30 January 1962, Phoenix commenced
against are based on the sound hypothesis of action in the Court of First Instance of Manila,
prompt payment of premiums. Upon this Branch IV (Civil Case 49468), to recover the
bedrock insurance firms are enabled to offer the amount of P7,483.11 as "earned premium."
assurance of security to the public at favorable Woodworks Inc. controverted basically on the
rates. But once payment of premium is left to the theory that its failure "to pay the premium after
whim and caprice of the insured, as when the the issuance of the policy put an end to the
courts tolerate the payment of a mere P600.00 insurance contract and rendered the policy
as partial undertaking out of the stipulated total unenforceable." On 13 September 1962,
premium of P2,983.50 and the balance to be judgment was rendered in Phoenix's favor
paid even after the risk insured against has "ordering Woodworks Inc. to pay Phoenix the
occurred, as Tibay et al. have done in this case, sum of P7,483.11, with interest thereon at the
on the principle that the strength of the vinculum rate of 6% per annum from 30 January 1962,
juris is not measured by any specific amount of until the principal shall have been fully paid, plus
premium payment, we will surely wreak havoc the sum of P700.00 as attorney's fees of the
on the business and set to naught what has Phoenix, and the costs of the suit." From this
taken actuarians centuries to devise to arrive at adverse Decision, Woodworks Inc. appealed to
a fair and equitable distribution of risks and the Court of Appeals which certified the case to
benefits between the insurer and the insured. the Supreme Court on a question of law.
Issue: Whether the Fire Insurance Policy was a
binding contract even if the premium stated in
Philippine Phoenix Surety & Insurance
the policy has not been paid.
Company vs. Woodworks Inc. [GR L-25317, 6
August 1979] First Division, Melencio-Herrera Held: Insurance is "a contract whereby one
(J): 4 concur, 1 abroad. undertakes for a consideration to indemnify
another against loss, damage or liability arising
from an unknown or contingent event." The
Facts: On 21 July 1960, upon Woodworks Inc.'s consideration is the "premium". "The premium
application, Philippine Phoenix Surety & must be paid at the time and in the way and
Insurance Company (Phoenix) issued in its favor manner specified in the policy and, if not so
Fire Insurance Policy 9749 for P500,000.00 paid, the policy will lapse and be forfeited by its
whereby Phoenix insured Woodworks Inc.'s own terms." The Policy provides for pre-
building, machinery and equipment for a term of payment of premium. Accordingly, "when the
one year from 21 July 1960 to 21 July 1961 policy is tendered the insured must pay the
against loss by fire. The premium and other premium unless credit is given or there is a
charges including the margin fee surcharge of waiver, or some agreement obviating the
P590.76 and the documentary stamps in the necessity for prepayment." To constitute an
amount of P156.60 affixed on the Policy, extension of credit there must be a clear and
amounted to P10,593.36. Woodworks Inc. did express agreement therefor. From the Policy
not pay the premium stipulated in the Policy provisions, there was no clear agreement that a
when it was issued nor at any time thereafter. credit extension was accorded Woodworks Inc.
On 19 April 1961, or before the expiration of the And even if it were to be presumed that Phoenix
one-year term, Phoenix notified Woodworks had extended credit from the circumstances of
Inc., through its Indorsement F-6963/61, of the the unconditional delivery of the Policy without
cancellation of the Policy allegedly upon request prepayment of the premium, yet it is obvious
of Woodworks Inc. The latter has denied having that Woodworks Inc. had not accepted the
made such a request. In said Indorsement, insurer's offer to extend credit, which is essential
Phoenix credited Woodworks Inc. with the for the validity of such agreement. An
amount of P3,110.25 for the unexpired period of acceptance of an offer to allow credit, if one was
94 days, and claimed the balance of P7,483.11 made, is as essential to make a valid agreement
representing "earned premium from 21 July for credit, to change a conditional delivery of an
33
insurance policy to an unconditional delivery, as
it is to make any other contract. Such an
Bonifacio Brothers Inc. vs. Mora [GR L-20853,
acceptance could not be merely a mental act or
29 May 1967] En Banc, Castro (J): 9 concur
state of mind, but would require a promise to
pay made known in some manner to
Woodworks Inc. In this respect, the present
case differs from that involving the same parties Facts: Enrique Mora is the owner of an
where recovery of the balance of the unpaid Oldsmobile sedan model 1956, bearing plate
premium was allowed inasmuch as in that case QC - 8088. He mortgaged the same to the H.S.
"there was not only a perfected contract of Reyes, Inc., with the condition that the former
insurance but a partially performed one as far as would insure the automobile, with the latter as
the payment of the agreed premium was beneficiary. The automobile was thereafter
concerned." This is not the situation obtaining insured on 23 June 1959 with the State Bonding
here where no partial payment of premiums has & Insurance Co. Inc., and motor car insurance
been made whatsoever. Since the premium had policy A-0615 was issued to Mora. During the
not been paid, the policy must be deemed to effectivity of an insurance contract, the car met
have lapsed. The nonpayment of premiums with an accident. The insurance company then
does not merely suspend but puts an end to an assigned the accident to the H.H. Bayne
insurance contract, since the time of the Adjustment Co. for investigation and appraisal of
payment is peculiarly of the essence of the the damage. Mora, without the knowledge and
contract. The rule is that under policy provisions consent of the H.S. Reyes, Inc., authorized the
that upon the failure to make a payment of a Bonifacio Bros. Inc. to furnish the labor and
premium or assessment at the time provided for, materials, some of which were supplied by the
the policy shall become void or forfeited, or the Ayala Auto Parts Co. For the cost of labor and
obligation of the insurer shall cease, or words to materials, Mora was billed at P2,102.73 through
like effect, because the contract so prescribes the H. H. Bayne Adjustment Co. The insurance
and because such a stipulation is a material and company, after claiming a franchise in the
essential part of the contract. This is true, for amount of P100, drew a check in the amount of
instance, in the case of life, health and accident, P2,002.73, as proceeds of the insurance policy,
fire and hail insurance policies. In fact, if the payable to the order of Mora or H.S. Reyes, Inc.,
peril insured against had occurred, Phoenix, as and entrusted the check to the H.H. Bayne
insurer, would have had a valid defense against Adjustment Co. for disposition and delivery to
recovery under the Policy it had issued. Explicit the proper party. In the meantime, the car was
in the Policy itself is Phoenix's agreement to delivered to Mora without the consent of the
indemnify Woodworks Inc. for loss by fire only H.S. Reyes, Inc., and without payment to the
"after payment of premium. Compliance by the Bonifacio Bros. Inc. and Ayala Auto Parts Co. of
insured with the terms of the contract is a the cost of repairs and materials. Upon the
condition precedent to the right of recovery. The theory that the insurance proceeds should be
burden is on an insured to keep a policy in force paid directly to them, the Bonifacio Bros. Inc.
by the payment of premiums, rather than on the and the Ayala Auto Parts Co. filed on 8 May
insurer to exert every effort to prevent the 1961 a complaint with the Municipal Court of
insured from allowing a policy to elapse through Manila against Mora and the State Bonding &
a failure to make premium payments. The Insurance Co. Inc. for the collection of the sum
continuance of the insurer's obligation is of P2,002.73. The insurance company filed its
conditional upon the payment of premiums, so answer with a counterclaim for interpleader,
that no recovery can be had upon a lapsed requiring the Bonifacio Bros. Inc. and the H.S.
policy, the contractual relation between the Reyes, Inc. to interplead in order to determine
parties having ceased. Moreover, an insurer who has a better right to the insurance proceeds
cannot treat a contract as valid for the purpose in question. Mora was declared in default for
of collecting premiums and invalid for the failure to appear at the hearing, and evidence
purpose of indemnity. The foregoing findings are against him was received ex parte. However,
buttressed by section 77 of the Insurance Code the counsel for the Bonifacio Bros. Inc., Ayala
(Presidential Decree No. 612, promulgated on Auto Parts Co. and State Bonding & Insurance
December 18, 1974), which now provides that Co. Inc. submitted a stipulation of facts, on the
no contract of insurance issued by an insurance basis of which the Municipal Court rendered a
company is valid and binding unless and until decision declaring the H.S. Reyes, Inc. as
the premium thereof has been paid, having a better right to the disputed amount, and
notwithstanding any agreement to the contrary. ordering the State Bonding & Insurance Co. Inc.
to pay to the H.S Reyes, Inc. the said sum of
34
P2,002.73. From this decision, Bonifacio Bros. person." Herein, no contract of trust, expressed
Inc. et al. elevated the case to the Court of First or implied exists. Thus, no cause of action exists
Instance of Manila before which the stipulation in favor of Bonifacio Bros. in so far as the
of facts was reproduced. On 19 October 1962 proceeds of insurance are concerned. Bonifacio
the latter court rendered a decision, affirming the Bros.' claim, if at all, is merely equitable in
decision of the Municipal Court. The Bonifacio nature and must be made effective through
Bros. Inc. and the Ayala Auto Parts Co. moved Enrique Mora who entered into a contract with
for reconsideration of the decision, but the trial the Bonifacio Bros Inc. This conclusion is
court denied the motion. Bonifacio Bros. Inc. et deducible not only from the principle governing
al. appealed. the operation and effect of insurance contracts
in general, but is clearly covered by the express
Issue: Whether Bonifacio Bros. has any cause
provisions of section 50 of the Insurance Act
of action to claim indemnity from the insurance
which read that "the insurance shall be applied
contract entered by State Bonding & Insurance
exclusively to the proper interest of the person in
Co. and Mora.
whose name it is made unless otherwise
Held: The insurance contract does not contain specified in the policy."
any words or clauses to disclose an intent to
The Insular Life Assurance Company Ltd. vs.
give any benefit to any repairmen or material
Ebrado [GR L-44059, 28 October 1977] First
men in case of repair of the car in question. The
Division, Martin (J): 5 concur
parties to the insurance contract omitted such
stipulation, which is a circumstance that
supports the said conclusion. On the other hand,
Facts: On 1 September 1968, Buenaventura
the "loss payable" clause of the insurance policy
Cristor Ebrado was issued by the Insular Life
stipulates that "Loss, if any, is payable to H.S.
Assurance Co., Ltd., Policy 009929 on a whole-
Reyes, Inc." indicating that it was only the H.S.
life plan for P5,882.00 with a rider for Accidental
Reyes, Inc. which they intended to benefit. It is
Death Benefits for the same amount.
likewise observed from the brief of the State
Buenaventura C. Ebrado designated Carponia
Bonding & Insurance Company that it has
T. Ebrado as the revocable beneficiary in his
vehemently opposed the assertion or pretension
policy. He referred to her as his wife. On 21
of Bonifacio Bros. that they are privy to the
October 1969, Buenventura C. Ebrado died as a
contract. If it were the intention of the Insurance
result of an accident when he was hit by a falling
Company to make itself liable to the repair shop
branch of a tree. As the insurance policy was in
or material men, it could have easily inserted in
force, Insular Life stands liable to pay the
the contract a stipulation to that effect. To hold
coverage of the policy in an amount of
now that the original parties to the insurance
P11,745.73, representing the face value of the
contract intended to confer upon Bonifacio Bros.
policy in the amount of P5,882.00 plus the
the benefit claimed by them would require as to
additional benefits for accidental death also in
ignore the indispensable requisite that a
the amount of P5,882.00 and the refund of
stipulation pour autrui must be clearly expressed
P18.00 paid for the premium due November,
by the parties, which the Court cannot do. As
1969, minus the unpaid premiums and interest
regards paragraph 4 of the insurance contract, a
thereon due for January and February, 1969, in
perusal thereof would show that instead of
the sum of P36.27. Carponia T. Ebrado filed
establishing privity between Bonifacio Bros. and
with the insurer a claim for the proceeds of the
the insurance company, such stipulation merely
policy as the designated beneficiary therein,
establishes the procedure that the insured has
although she admits that she and the insured
to follow in order to be entitled to indemnity for
Buenaventura C. Ebrado were merely living as
repair. This paragraph therefore should not be
husband and wife without the benefit of
construed as bringing into existence in favor of
marriage. Pascuala Vda. de Ebrado also filed
Bonifacio Bros. a right of action against the
her claim as the widow of the deceased insured.
insurance company as such intention can never
She asserts that she is the one entitled to the
be inferred therefrom. Another cogent reason for
insurance proceeds, not the common-law wife,
not recognizing a right of action by Bonifacio
Carponia T. Ebrado. In doubt as to whom the
Bros. against the insurance company is that "a
insurance proceeds shall be paid, the insurer
policy of insurance is a distinct and independent
commenced an action for Interpleader before
contract between the insured and insurer, and
the Court of First Instance of Rizal on 29 April
third persons have no right either in a court of
1970. On 25 September 1972, the trial court
equity, or in a court of law, to the proceeds of it,
rendered judgment declaring, among others,
unless there be some contract of trust,
Carponia T. Ebrado disqualified from becoming
expressed or implied, by the insured and third
35
beneficiary of the insured Buenaventura Cristor be proved by preponderance of evidence in the
Ebrado and directing the payment of the same action." In essence, a life insurance policy
insurance proceeds to the estate of the is no different from a civil donation insofar as the
deceased insured. From this judgment, beneficiary is concerned. Both are founded upon
Carponia T. Ebrado appealed to the Court of the same consideration: liberality. A beneficiary
Appeals, but on 11 July 1976, the Appellate is like a donee, because from the premiums of
Court certified the case to the Supreme Court as the policy which the insured pays out of
involving only questions of law. liberality, the beneficiary will receive the
proceeds or profits of said insurance. As a
Issue [1]: Whether a common-law wife named
consequence, the proscription in Article 739 of
as beneficiary in the life insurance policy of a
the new Civil Code should equally operate in life
legally married man can claim the proceeds
insurance contracts. The mandate of Article
thereof in case of death of the latter.
2012 cannot be laid aside: any person who
Held[1]: NO. It is quite unfortunate that the cannot receive a donation cannot be named as
Insurance Act (RA 2327, as amended) or even beneficiary in the life insurance policy of the
the new Insurance Code (PD 612, as amended) person who cannot make the donation. Under
does not contain any specific provision grossly American law, a policy of life insurance is
resolutory of the prime question at hand. Section considered as a testament and in construing it,
50 of the Insurance Act which provides that the courts will, so far as possible treat it as a will
"(t)he insurance shall be applied exclusively to and determine the effect of a clause designating
the proper interest of the person in whose name the beneficiary by rules under which wills are
it is made" cannot be validly seized upon to hold interpreted. Policy considerations and dictates of
that the same includes the beneficiary. The word morality rightly justify the institution of a barrier
"interest" highly suggests that the provision between common-law spouses in regard to
refers only to the insured and not to the property relations since such relationship
beneficiary, since a contract of insurance is ultimately encroaches upon the nuptial and filial
personal in character. Otherwise, the prohibitory rights of the legitimate family. There is every
laws against illicit relationships especially on reason to hold that the bar in donations between
property and descent will be rendered nugatory, legitimate spouses and those between
as the same could easily be circumvented by illegitimate ones should be enforced in life
modes of insurance. Rather, the general rules of insurance policies since the same are based on
civil law should be applied to resolve this void in similar consideration. As pointed out, a
the Insurance Law. Article 2011 of the New Civil beneficiary in a life insurance policy is no
Code states: "The contract of insurance is different from a donee. Both the recipients of
governed by special laws. Matters not expressly pure beneficence. So long as marriage remains
provided for in such special laws shall be the threshold of family laws, reason and morality
regulated by this Code." When not otherwise dictate that the impediments imposed upon
specifically provided for by the Insurance Law, married couple should likewise be imposed
the contract of life insurance is governed by the upon extra-marital relationship. If legitimate
general rules of the civil law regulating relationship is circumscribed by these legal
contracts. And under Article 2012 of the same disabilities, with more reason should an illicit
Code, "any person who is forbidden from relationship be restricted by these disabilities.
receiving any donation under Article 739 cannot
Issue [2]: Whether a conviction for adultery or
be named beneficiary of a life insurance policy
concubinage is exacted before the disabilities
by the person who cannot make a donation to
mentioned in Article 739 may effectuate.
him." Common-law spouses are, definitely,
barred from receiving donations from each Held [2]: NO. A conviction for adultery or
other. Article 739 of the new Civil Code provides concubinage is not exacted before the
that "the following donations shall be void: (1) disabilities mentioned in Article 739 may
Those made between persons who were guilty effectuate. More specifically, with regard to the
of adultery or concubinage at the time of disability on "persons who were guilty of adultery
donation; (2) Those made between persons or concubinage at the time of the donation,"
found guilty of the same criminal offense, in Article 739 itself provides that "In the case
consideration thereof; (3) Those made to a referred to in No. 1, the action for declaration of
public officer or his wife, descendants or nullity may be brought by the spouse of the
ascendants by reason of his office. In the case donor or donee; and the guilt of the donee may
referred to in No. 1, the action for declaration of be proved by preponderance of evidence in the
nullity may be brought by the spouse of the same action." The underscored clause neatly
donor or donee; and the guilt of the donee may conveys that no criminal conviction for the
36
disqualifying offense is a condition precedent. In P6,304.47 pursuant to Section 12(c) of
fact, it cannot even be gleaned from the Commonwealth Act 186 as amended by
aforequoted provision that a criminal Republic Acts 1616 and 3836. Consuegra did
prosecution is needed. On the contrary, the law not designate any beneficiary who would receive
plainly states that the guilt of the party may be the retirement insurance benefits due to him.
proved "in the same action" for declaration of Rosario Diaz, the widow by the first marriage,
nullity of donation. And, it would be sufficient if filed a claim with the GSIS asking that the
evidence preponderates upon the guilt of the retirement insurance benefits be paid to her as
consort for the offense indicated. The quantum the only legal heir of Consuegra, considering
of proof in criminal cases is not demanded. that the deceased did not designate any
Herein, the requisite proof of commonlaw beneficiary with respect to his retirement
relationship between the insured and the insurance benefits. Basilia Berdin and her
beneficiary has been conveniently supplied by children, likewise, filed a similar claim with the
the stipulations between the parties in the pre- GSIS, asserting that being the beneficiaries
trial conference of the case. It was agreed upon named in the life insurance policy of Consuegra,
and stipulated therein that the deceased insured they are the only ones entitled to receive the
Buenaventura C. Ebrado was married to retirement insurance benefits due the deceased
Pascuala Ebrado with whom she has six Consuegra. Resolving the conflicting claims, the
legitimate children; that during his lifetime, the GSIS ruled that the legal heirs of the late Jose
deceased insured was living with his common- Consuegra were Rosario Diaz, his widow by his
law wife, Carponia Ebrado, with whom he has first marriage who is entitled to one-half, or 8/16,
two children. These stipulations are nothing less of the retirement insurance benefits, on the one
than judicial admissions which, as a hand; and Basilia Berdin, his widow by the
consequence, no longer require proof and second marriage and their seven children, on
cannot be contradicted. A fortiori, on the basis of the other hand, who are entitled to the remaining
these admissions, a judgment may be validly one-half, or 8/16, each of them to receive an
rendered without going through the rigors of a equal share of 1/16. Dissatisfied with the
trial for the sole purpose of proving the illicit foregoing ruling and apportionment made by the
liaison between the insured and the beneficiary GSIS, Basilia Berdin and her children filed on 10
October 1966 a petition for mandamus with
Vda de Consuegra vs. Government Service
preliminary injunction in the Court of First
Insurance System [GR L-28093, 30 January
Instance of Surigao del Norte (Special
1971] En Banc, Zaldivar (J): 10 concur
Proceeding 1720) naming as respondents the
Facts: The late Jose Consuegra, at the time of GSIS, the Commissioner of Public Highways,
his death, was employed as a shop foreman of the Highway District Engineer of Surigao del
the office of the District Engineer in the province Norte, the Commissioner of Civil Service, and
of Surigao-del Norte. In his lifetime, Consuegra Rosario Diaz, praying that they (Basilia Berdin,
contracted two marriages, the first with Rosario et al.) be declared the legal heirs and exclusive
Diaz, solemnized in the parish church of San beneficiaries of the retirement insurance of the
Nicolas de Tolentino, Surigao, Surigao, on 15 late Jose Consuegra, and that writ of preliminary
July 1937, out of which marriage were born two injunction be issued restraining implementation
children, namely, Jose Consuegra, Jr. and of the adjudication made by the GSIS. On 7
Pedro Consuegra, but both predeceased their March 1967, the court of First Instance of
father; and the second, which was contracted in Surigao rendered judgment, holding that when
good faith while the first marriage was two women innocently and in good faith are
subsisting, with Basilia Berdin, on 1 May 1957 in legally united in holy matrimony to the same
the same parish and municipality, out of which man, they and their children, born of said
marriage were born seven children, namely, wedlock, will be regarded as legitimate children
Juliana, Pacita, Maria Lourdes, Jose, Rodrigo, and each family be entitled to one half of the
Lenida and Luz, * all surnamed Consuegra. estate. The court thus declared that Basilia
Being a member of the Government Service Berdin Vda. de Consuegra and Juliana, Pacita,
Insurance System (GSIS) when Consuegra died Maria Lourdes, Jose Jr., Rodrigo, Lenida and
on 26 September 1965, the proceeds of his life Luis, all surnamed Consuegra, beneficiary and
insurance under policy 601801 were paid by the entitled to 1/2 of the retirement benefit in the
GSIS to Basilia Berdin and her children who amount of P6,304.47) due to the deceased Jose
were the beneficiaries named in the policy. Consuegra from the GSIS or the amount of
Having been in the service of the government P3,152.235 to be divided equally among them in
for 22.5028 years, Consuegra was entitled to the proportional amount of 1/16 each. Likewise,
retirement insurance benefits in the sum of Rosario Diaz Vda. de Consuegra is hereby
37
declared beneficiary and entitled to the other the proceeds of the insurance will go to the
half of the retirement benefit of the late Jose estate of the insured. Retirement insurance is
Consuegra or the amount of P3,152.235. Basilia primarily intended for the benefit of the
Berdin and her children appealed (on purely employee — to provide for his old age, or
questions of law). incapacity, after rendering service in the
government for a required number of years. If
Issue [1]: Whether Basilia Berdin Vda. de
the employee reaches the age of retirement, he
Consuegra. who were the beneficiaries named
gets the retirement benefits even to the
in the life insurance should automatically be
exclusion of the beneficiary or beneficiaries
considered the beneficiaries to receive the
named in his application for retirement
retirement insurance benefits, to the exclusion of
insurance. The beneficiary of the retirement
Rosario Diaz, when the deceased Jose
insurance can only claim the proceeds of the
Consuegra failed to designate the beneficiaries
retirement insurance if the employee dies before
in his retirement insurance.
retirement. If the employee failed or overlooked
Held [1]: NO. If Consuegra had 22.5028 years of to state the beneficiary of his retirement
service in the government when he died on 26 insurance, the retirement benefits will accrue his
September 1965, it follows that he started in the estate and will be given to his legal heirs in
government service sometime during the early accordance with law, as in the case of a life
part of 1943, or before 1943. In 1943, insurance if no beneficiary is named in the
Commonwealth Act 186 was not yet amended, insurance policy. The GSIS, therefore, had
and the only benefits then provided for in said correctly acted when it ruled that the proceeds
Act were those that proceed from a life of the retirement insurance of the late Jose
insurance. Upon entering the government Consuegra should divided equally between his
service Consuegra became a compulsory first living wife Rosario on the one hand, and his
member of the GSIS, being automatically second wife Basilia Berdin his children by her,
insured on his life, pursuant to the provisions of on the other; and the lower court did not commit
CA 186 which was in force at the time. During error when it confirmed the action of the GSIS, it
1943 the operation of the GSIS was suspended being accepted as a fact that the second
because of the war, and the operation was marriage of Jose Consuegra to Basilia Berdin
resumed sometime in 1946. When Consuegra was contracted in good faith. The lower court
designated his beneficiaries in his life insurance has correctly applied the ruling of this Court in
he could not have intended those beneficiaries the case of Lao, et al. vs. Dee Tim, et al., 45
of his life insurance s also the beneficiaries of Phil. 739. In the recent case of Gomez vs.
his retirement insurance because the provisions Lipana, L-23214, June 30, 1970, the Court, in
on retirement insurance under the GSIS came construing the rights of two women who were
about only when CA 186 was amended by RA married to the same man — a situation more or
660 on 16 June 1951. Hence, it cannot be said less similar to the case of Basilia Berdin and
that cause Basilia Berdin et al. were designated Rosario Diaz — held "that since the defendant's
beneficiaries Consuegra's life insurance they first marriage has not been dissolved or
automatically became beneficiaries also of his declared void the conjugal partnership
retirement insurance. established by that marriage has not ceased.
Nor has the first wife lost or relinquished her
Issue [2]: Whether the GSIS and the trial court status as putative heir of her husband under the
are correct in ruling that each of the wives who new Civil Code, entitled to share in his estate
contracted marriage to the same man in good upon his death should she survive him.
faith are each entitled to half of the retirement Consequently, whether as conjugal partner in a
insurance benefits. still subsisting marriage or as such putative heir
Held [2]: YES. In the case of the proceeds of a she has an interest in the husband's share in the
life insurance, the same are paid to whoever is property here in dispute. " And with respect to
named the beneficiary in the life insurance the right of the second wife, this Court observed
policy. As in the case of a life insurance that although the second marriage can be
provided for in the Insurance Act (Act 2427, as presumed to be void ab initio as it was
amended), the beneficiary in a life insurance celebrated while the first marriage was still
under the GSIS may not necessarily be an heir subsisting, still there is need for judicial
of the insured. The insured in a life insurance declaration of such nullity. And inasmuch as the
may designate any person as beneficiary unless conjugal partnership formed by the second
disqualified to be so under the provisions of the marriage was dissolved before judicial
Civil Code. And in the absence of any declaration of its nullity, "[t]he only just and
beneficiary named in the life insurance policy, equitable solution in this case would be to
38
recognize the right of the second wife to her affirmed the appealed judgment in all of its
share of one-half in the property acquired by her parts, with costs against Go.
and her husband, and consider the other half as
pertaining to the conjugal partnership of the first
marriage." Country Bankers Insurance Corporation vs.
Lianga Bay and Community Multi-Purpose
Cooperative Inc. [GR 136914, 25 January 2002]
Go vs. Redfern [GR 47705, 25 April 1941] Second Division, De Leon Jr. (J): 4 concur
Second Division, Horrilleno (J): 4 concur
Facts: Country Bankers Insurance Corporation
Decision in Spanish [Rough translation,
(CBIC) is a domestic corporation principally
accuracy unverified]
engaged in the insurance business wherein it
Facts: In October 1937, Edward K. Redfern undertakes, for a consideration, to indemnify
obtained an insurance policy against accidents another against loss, damage or liability from an
from the International Assurance Co, Ltd. On 31 unknown or contingent event including fire while
August 1938, Redfern died from an accident. Lianga Baya and Community Multipurpose
The mother of the deceased, presenting the Cooperative Inc. (LBCMCI) is a duly registered
necessary evidence of the death of Redfern, cooperative judicially declared insolvent and
sought to claim the proceeds of the insurance represented by the elected assignee, Cornelio
policy from the assurance company. The Jamero. It appears that sometime in 1989, the
company, however, denied such claim, on the CBIC and LBCMCI entered into a contract of fire
ground that the insurance policy was amended insurance. Under Fire Insurance Policy F-1397,
on 22 November 1937 to include another CBIC insured LBCMCI's stocks-in-trade against
beneficiary, Concordia Go. Hence, an action fire loss, damage or liability during the period
was filed to determine who has the right to starting from 20 June 1989 at 4:00 p.m. to 20
collect the insurance proceeds of the deceased June 1990 at 4:00 p.m., for the sum of
Redfern. The mother claimed that the addition of P200,000.00. On 1 July 1989, at or about 12:40
the co-beneficiary is illegal. Go, on her part, a.m., LBCMCI's building located at Barangay
alleged the contrary. The trial court ruled in favor Diatagon, Lianga, Surigao del Sur was gutted by
of Angela Redfern, the mother. Go appealed. fire and reduced to ashes, resulting in the total
loss of LBCMCI's stocks-in-trade, pieces of
Issue: Whether the addition of Go’s name as co-
furniture and fixtures, equipments and records.
beneficiary can be allowed for her share in the
Due to the loss, LBCMCI filed an insurance
insurance proceeds.
claim with CBIC under its Fire Insurance Policy
Held: When designated in a policy, the F-1397, submitting: (a) the Spot Report of Pfc.
beneficiary acquires a right of which he cannot Arturo V. Juarbal, INP Investigator, dated 1 July
be deprived of without his consent, unless the 1989; (b) the Sworn Statement of Jose
right has been reserved specifically to the Lomocso; and (c) the Sworn Statement of
insured to modify the policy. The same doctrine Ernesto Urbiztondo. CBIC, however, denied the
was enunciated by the Court in the cases of insurance claim on the ground that, based on
Gercio vs. Sun Life Assurance Co. of Canada the submitted documents, the building was set
(48 Phil. 55) and Insular Life vs. Suva (34 Off. on fire by 2 NPA rebels who wanted to obtain
Gaz. 861). Thus, unless the insured has canned goods, rice and medicines as provisions
reserved specifically the right to change or to for their comrades in the forest, and that such
modify the policy, with respect to the beneficiary, loss was an excepted risk under paragraph 6 of
said policy constitutes an acquired right of the the policy conditions of Fire Insurance Policy
beneficiary, which cannot be modified except F1397, which provides that "This insurance does
with the consent of the latter. Herein, it is not cover any loss or damage occasioned by or
admitted that Redfern did not reserve expressly through or in consequence, directly or indirectly,
his right to change or modify the policy. Change of any of the following occurrences, namely: xxx
implies the idea of an alteration. The addition of (d) Mutiny, riot, military or popular uprising,
Go's name as one of the beneficiaries of the insurrection, rebellion, revolution, military or
policy constitutes change as all addition is an usurped power. Any loss or damage happening
alteration. The addition of Go's name changed during the existence of abnormal conditions
the policy inasmuch as there are two (whether physical or otherwise) which are
beneficiaries instead of one, and thus in effect occasioned by or through or in consequence,
the original beneficiary cannot recieve the full directly or indirectly, of any of said occurrences
amount of the policy. The Supreme Court shall be deemed to be loss or damage which is
not covered by this insurance, except to the
39
extent that the Insured shall prove that such loss CBIC failed to do so. CBIC relies on the Sworn
or damage happened independently of the Statements of Jose Lomocso and Ernesto
existence of such abnormal conditions." Finding Urbiztondo as well as on the Spot Report of Pfc.
the denial of its claim unacceptable, LBCMCI Arturo V. Juarbal dated 1 July 1989. The Sworn
then instituted in the trial court the complaint for Statements of Jose Lomocso and Ernesto
recovery of "loss, damage or liability" against Urbiztondo are inadmissible in evidence, for
CBIC. In due time, the trial court rendered its being hearsay, inasmuch as they did not take
Decision dated 26 December 1991 in favor of the witness stand and could not therefore be
LBCMCI, ordering CBIC to pay LBCMCI to fully cross-examined. CBIC's evidence to prove its
pay the insurance claim for the loss LBCMCI defense is sadly wanting and thus, gives rise to
sustained as a result of the fire under its Fire its liability to LBCMCI under Fire Insurance
Insurance Policy F-1397 in its full face value of Policy F-1397.
P200,000.00 with interest of 12% per annum
from date of filing of the complaint until the same
is fully paid; to pay as and in the concept of Roque vs. Intermediate Appellate Court [GR L-
actual or compensatory damages in the total 66935, 11 November 1985] First Division,
sum of P50,000.00; to pay as and in the concept Gutierrez (J): 5 concur, 1 on leave
of exemplary damages in the total sum of
P50,000.00; to pay in the concept of litigation Facts: On 19 February 1972, the Manila Bay
expenses the sum of P5,000.00; to pay by way Lighterage Corporation (MBLC) a common
of reimbursement the attorney's fees in the sum carrier, entered into a contract with Isabela
of P10,000.00; and to pay the costs of the suit. Roque (doing business under the name and
CBIC interposed an appeal to the Court of style of Isabela Roque Timber Enterprises) and
Appeals. On 29 December 1998, the appellate Ong Chiong whereby the former would load and
court affirmed the challenged decision of the trial carry on board its barge Mable 10 about 422.18
court in its entirety. CBIC filed the petition for cubic meters of logs from Malampaya Sound,
review on certiorari. Palawan to North Harbor, Manila. Roque and
Ong insured the logs against loss for
Issue: Whether the burden of proof of loss in this P100,000.00 with the Pioneer Insurance and
case is upon the insurer, and not the insured. Surety Corporation (Pioneer). On 29 February
1972, Roque and Ong loaded on the barge, 811
Held: YES. CBIC does not dispute that
pieces of logs at Malampaya Sound, Palawan
LBCMCI's stocks-in-trade were insured against
for carriage and delivery to North Harbor, Port of
fire loss, damage or liability under Fire
Manila, but the shipment never reached its
Insurance Policy F-1397 and that LBCMCI lost
destination because Mable 10 sank with the 811
its stocks-in-trade in a fire that occurred on 1
pieces of logs somewhere off Cabuli Point in
July 1989, within the duration of said fire
Palawan on its way to Manila. The barge where
insurance. CBIC, however, posits the view that
the logs were loaded was apparently not
the cause of the loss was an excepted risk
seaworthy such that it developed a leak. One of
under the terms of the fire insurance policy.
the hatches was left open causing water to enter
Where a risk is excepted by the terms of a policy
the barge and because the barge was not
which insures against other perils or hazards,
provided with the necessary cover or tarpaulin,
loss from such a risk constitutes a defense
the ordinary splash of sea waves brought more
which the insurer may urge, since it has not
water inside the barge. On 8 March 1972,
assumed that risk, and from this it follows that
Roque and Ong wrote a letter to MBLC
an insurer seeking to defeat a claim because of
demanding payment of P150,000.00 for the loss
an exception or limitation in the policy has the
of the shipment plus P100,000.00 as unrealized
burden of proving that the loss comes within the
profits but the latter ignored the demand.
purview of the exception or limitation set up. If a
Another letter was sent to Pioneer claiming the
proof is made of a loss apparently within a
full amount of P100,000.00 under the insurance
contract of insurance, the burden is upon the
policy but Pioneer refused to pay on the ground
insurer to prove that the loss arose from a cause
that its liability depended upon the "Total loss by
of loss which is excepted or for which it is not
Total Loss of Vessel only". Hence, Roque and
liable, or from a cause which limits its liability.
Ong commenced Civil Case 86599 against
Stated elsewise, since CBIC in this case is
MBLC and Pioneer Pioneer. During the initial
defending on the ground of non-coverage and
stages of the hearing, MBLC informed the trial
relying upon an exemption or exception clause
court that it had salvaged part of the logs. The
in the fire insurance policy, it has the burden of
court ordered them to be sold to the highest
proving the facts upon which such excepted risk
bidder with the funds to be deposited in a bank
is based, by a preponderance of evidence. But
40
in the name of Civil Case 86599. After hearing, Vessels, craft, aircraft, vehicles, goods, freights,
the trial court found in favor of Roque and Ong, cargoes, merchandise..." From the above-
condemning MBLC and Pioneer to pay Roque quoted provisions, there can be no mistaking the
and Ong, jointly and severally, the sum of fact that the term "cargo" can be the subject of
P100,000.00; sentencing MBLC to pay Roque marine insurance and that once it is so made,
and Ong, in addition, the sum of P50,000.00, the implied warranty of seaworthiness
plus P12,500.00, that the latter advanced to the immediately attaches to whoever is insuring the
former as down payment for transporting the cargo whether he be the shipowner or not. As
logs in question; ordering the counterclaim of ruled in the case of Go Tiaoco y Hermanos v.
Pioneer against Roque and Ong, dismissed, for Union Insurance Society of Canton (40 Phil. 40),
lack of merit, but as to its cross-claim against its "it is universally accepted that in every contract
MBLC, the latter is ordered to reimburse the of insurance upon anything which is the subject
former for whatever amount it may pay Roque of marine insurance, a warranty is implied that
and Ong as such surety; ordering the the ship shall be seaworthy at the time of the
counterclaim of MBLC against Roque and Ong, inception of the voyage. This rule is accepted in
dismissed for lack of merit; dismissing Roque's our own Insurance Law (Act No. 2427, sec.
and Ong's claim of not less than P100,000.00 106)." Moreover, the fact that the
and P75,000.00 as exemplary damages, for lack unseaworthiness of the ship was unknown to the
of merit; granting Roque's and Ong's claim for insured is immaterial in ordinary marine
attorney's fees in the sum of P10,000.00; insurance and may not be used by him as a
ordering MBLC and Pioneer to pay the costs; defense in order to recover on the marine
and holding that the sum of P150,000.00 award insurance policy. As was held in Richelieu and
to Roque and Ong, shall bear interest of 6% Ontario Nav. Co. v. Boston Marine, Inc., Co.
from 25 March 1975, until amount is fully paid. (136 U.S. 406), "the exception of losses
Pioneer appealed to the Intermediate Appellate occasioned by unseaworthiness was in effect a
Court. MBLC did not appeal, as allegedly, the warranty that a loss should not be so
transportation company is no longer doing occasioned, and whether the fact of
business and is without funds. On 30 January unseaworthiness were known or unknown would
1984, the appellate court modified the trial be immaterial." Since the law provides for an
court's decision and absolved Pioneer from implied warranty of seaworthiness in every
liability after finding that there was a breach of contract of ordinary marine insurance, it
implied warranty of seaworthiness on the part of becomes the obligation of a cargo owner to look
Roque and Ong and that the loss of the insured for a reliable common carrier which keeps its
cargo was caused by the "perils of the ship" and vessels in seaworthy condition. The shipper of
not by the "perils of the sea". It ruled that the cargo may have no control over the vessel but
loss is not covered by the marine insurance he has full control in the choice of the common
policy. After the appellate court denied their carrier that will transport his goods. Or the cargo
motion for reconsideration, Roque and Ong filed owner may enter into a contract of insurance
the petition for certiorari. which specifically provides that the insurer
answers not only for the perils of the sea but
Issue [1]: Whether there is a warranty of
also provides for coverage of perils of the ship.
seaworthiness by the cargo owner in cases of
The Court was constrained to apply Section 113
marine cargo insurance.
of the Insurance Code to the facts of this case.
Held [1]: YES. There is no dispute over the "In marine cases, the risks insured against are
liability of the common carrier MBLC. In fact, it 'perils of the sea' (Chute v. North River Ins. Co.,
did not bother to appeal the questioned Minn. 214 NW 472, 55 ALR 933). The purpose
decision. However, Roque and Ong state that of such insurance is protection against
MBLC has ceased operating as a firm and contingencies and against possible damages
nothing may be recovered from it. They are, and such a policy does not cover a loss or injury
therefore, trying to recover their losses from the which must inevitably take place in the ordinary
insurer. The liability of the insurance company is course of things. There is no doubt that the term
governed by law. Section 113 of the Insurance 'perils of the sea' extends only to losses caused
Code provides that "In every marine insurance by sea damage, or by the violence of the
upon a ship or freight, or freightage, or upon any elements, and does not embrace all losses
thing which is the subject of marine insurance, a happening at sea. They insure against losses
warranty is implied that the ship is seaworthy." from extraordinary occurrences only, such as
Section 99 of the same Code also provides in stress of weather, winds and waves, lightning,
part that "Marine insurance includes: (1) tempests, rocks and the like. These are
Insurance against loss of or damage to: (a) understood to be the 'perils of the sea' referred
41
in the policy, and not those ordinary perils which furthermore, that a loss which, in the ordinary
every vessel must encounter. 'Perils of the sea' course of events, results from the natural and
has been said to include only such losses as are inevitable action of the sea, from the ordinary
of extraordinary nature, or arise from some wear and tear of the ship, or from the negligent
overwhelming power, which cannot be guarded failure of the ship's owner to provide the vessel
against by the ordinary exertion of human skill with proper equipment to convey the cargo
and prudence. Damage done to a vessel by under ordinary conditions, is not a peril of the
perils of the sea includes every species of sea. Such a loss is rather due to what has been
damages done to a vessel at sea, as aptly called the 'peril of the ship.' The insurer
distinguished from the ordinary wear and tear of undertakes to insure against perils of the sea
the voyage and distinct from injuries suffered by and similar perils, not against perils of the ship.
the vessel in consequence of her not being As was well said by Lord Herschell in Wilson,
seaworthy at the outset of her voyage (as in this Sons & Co. v. Owners of Cargo per the Xantho
case). It is also the general rule that everything ([1887], 12 A. C., 503, 509), there must, in order
which happens thru the inherent vice of the to make the insurer liable, be 'some casualty,
thing, or by the act of the owners, master or something which could not be foreseen as one
shipper, shall not be reputed a peril, if not of the necessary incidents of the adventure. The
otherwise borne in the policy. (14 RCL on purpose of the policy is to secure an indemnity
'Insurance', Sec. 384, pp. 1203-1204; Cia. de against accidents which may happen, not
Navegacion v. Firemen's Fund Ins. Co., 277 US against events which must happen.”
66, 72 L. ed. 787, 48 S. Ct. 459)."
Issue [2]: Whether the loss of the cargo was due
La Razon Social "Go Tiaoco y Hermanos" vs.
to the perils of the ship rather than the perils of
Union Insurance Society of Canton Ltd. [GR
the sea.
13983, 1 September 1919] First Division, Street
Held [2]: PERILS OF THE SHIP. At the time
Mable 10 sank, there was no typhoon but
Facts:
ordinary strong wind and waves, a condition
which is natural and normal in the open sea. A cargo of rice belonging to the Go Tiaoco
The evidence shows that the sinking of Mable Brothers, was transported in the early days of
10 was due to improper loading of the logs on May, 1915, on the steamship Hondagua from
one side so that the barge was tilting on one the port of Saigon to Cebu. On discharging the
side and for that it did not navigate on even keel; rice from one of the compartments in the after
that it was no longer seaworthy that was why it hold, upon arrival at Cebu, it was discovered
developed leak; that the personnel of the that 1,473 sacks had been damaged by sea
tugboat and the barge committed a mistake water. The loss so resulting to the owners of
when it turned loose the barge from the tugboat rice, after proper deduction had been made for
east of Cabuli point where it was buffeted by the portion saved, was P3,875. The policy of
storm and waves, while the tugboat proceeded insurance, covering the shipment, was signed
to west of Cabuli point where it was protected by upon a form long in use among companies
the mountain side from the storm and waves engaged in maritime insurance. It purports to
coming from the east direction. In fact, in insure the cargo from the following among other
Roque's and Ong's complaint, it is alleged that risks: "Perils . . . of the seas, men, of war, fire,
the barge Mable 10 of MBLC developed a leak enemies, pirates, rovers, thieves, .jettisons, . . .
which allowed water to come in and that one of barratry of the master and mariners, and of all
the hatches of said barge was negligently left other perils, losses, and misfortunes that have
open by the person in charge thereof causing or shall come to the hurt, detriment, or damage
more water to come in", and that "he loss of of the said goods and merchandise or any part
their cargo was due to the fault, negligence, thereof." It was found out that the drain pipe
and/or lack of skill of MBLC and/or MBLC's which served as a discharge from the water
representatives on barge Mable 10. It is quite closet passed down through the compartment
unmistakable that the loss of the cargo was due where the rice in question was stowed and
to the perils of the ship rather than the perils of thence out to sea through the wall of the
the sea. The facts clearly negate Roque's and compartment, which was a part of the wall of the
Ong's claim under the insurance policy. In the ship. The joint or elbow where the pipe changed
case of Go Tiaoco y Hermanos v. Union Ins. its direction was of cast iron; and in course of
Society of Canton, the Court had occasion to time it had become corroded and abraded until a
elaborate on the term "perils of the ship" when it longitudinal opening had appeared in the pipe
ruled that "It must be considered to be settled, about one inch in length. This hole had been in
42
existence before the voyage was begun, and an from the natural and inevitable action of the sea,
attempt had been made to repair it by filling with from the ordinary wear and tear of the ship, or
cement and bolting over it a strip of iron. The from the negligent failure of the ship's owner to
effect of loading the boat was to submerge the provide the vessel with proper equipment to
vent, or orifice, of the pipe until it was about 18 convey the cargo under ordinary conditions, is
inches or 2 feet below the level of the sea. As a not a peril of the sea. Such a loss is rather due
consequence the sea water rose in the pipe. to what has been aptly called the "peril of the
Navigation under these conditions resulted in ship." The insurer undertakes to insure against
the washing out of the cement-filling from the perils of the sea and similar perils, not against
action of the sea water, thus permitting the perils of the ship. There must, in order to make
continued flow of the salt water into the the insurer liable, be "some casualty, something
compartment of rice. An action on a policy of which could not be foreseen as one of the
marine insurance issued by the Union Insurance necessary incidents of the adventure. The
Society of Canton, Ltd., upon the cargo of rice purpose of the policy is to secure an indemnity
belonging to the Go Tiaoco Brothers was filed. against accidents which may happen, not
The trial court found that the inflow of the sea against events which must happen." Herein, the
water during the voyage was due to a defect in entrance of the sea water into the ship's hold
one of the drain pipes of the ship and concluded through the defective pipe already described
that the loss was not covered by the policy of was not due to any accident which happened
insurance. Judgment was accordingly entered in during the voyage, but to the failure of the ship's
favor of Union Insurance and Go Tiaoco owner properly to repair a defect of the
Brothers appealed. existence of which he was apprised. The loss
was therefore more analogous to that which
Issue [1]: Whether perils of the sea includes
directly results from simple unseaworthiness
“entrance of water into the ship’s hold through a
than to that which results from perils of the sea.
defective pipe.”
Issue [2]: Whether there is an implied warranty
Held [1]: NO. It is determined that the words "all
on the seaworthy of the vessel in every marine
other perils, losses, and misfortunes" are to be
insurance contract.
interpreted as covering risks which are of like
kind (ejusdem generis) with the particular risks Held [2]: YES. It is universally accepted that in
which are enumerated in the preceding part of every contract of insurance upon anything which
the same clause of the contract. According to is the subject of marine insurance, a warranty is
the ordinary rules of construction these words implied that the ship shall be seaworthy at the
must be interpreted with reference to the words time of the inception of the voyage. This rule is
which immediately precede them. They were no accepted in our own Insurance Law (Act No.
doubt inserted in order to prevent disputes 2427, sec. 106). It is also well settled that a ship
founded on nice distinctions. Their office is to which is seaworthy for the purpose of insurance
cover in terms whatever may be within the spirit upon the ship may yet be unseaworthy for the
of the cases previously enumerated, and so they purpose of insurance upon the cargo (Act No.
have a greater or less effect as a narrower or 2427, sec. 106).
broader view is taken of those cases. For
example, if the expression "perils of the seas" is
given its widest sense the general words have Cathay Insurance Co. vs. Court of Appeals [GR
little or no effect as applied to that case. If on the 76145, 30 June 1987] Second Division, Paras
other hand that expression is to receive a limited (J): 3 concur, 2 took no part
construction and loss by perils of the seas is to
be confined to loss ex marine tempestatis Facts: A complaint was filed by Remington
discrimine, the general words become most Industrial Sales Corporation against Cathay
important. But still, when they first became the Insurance Co. seeking collection of the sum of
subject of judicial construction, they have always P868,339.15 representing Remington's losses
been held or assumed to be restricted to cases and damages incurred in a shipment of
"akin to" or "resembling" or "of the same kind as" seamless steel pipes under an insurance
those specially mentioned. I see no reason for contract in favor of Remington as the insured,
departing from this settled rule. In marine consignee or importer of aforesaid merchandise
insurance it is above all things necessary to while in transit from Japan to the Philippines on
abide by settled rules and to avoid anything like board vessel SS "Eastern Mariner." The total
novel refinements or a new departure. It must be value of the shipment was P2,894,463.83 at the
considered to be settled, furthermore, that a loss prevailing rate of P7.95 to a dollar in June and
which, in the ordinary course of events, results July 1984, when the shipment was made. The
43
trial court decided in favor of Remington by application of the policy") there is in fact an
ordering Cathay Insurance to pay it the sum of express exception to the application of the
P866,339.15 as its recoverable insured loss policy; (2) As adverted to in the Petition for
equivalent to 30% of the value of the seamless Review, Remington has admitted that the
steel pipes; ordering Cathay Insurance to pay questioned shipment is not covered by a
Remington interest on the aforecited amount at "square provision of the contract," but
the rate of 34% or double the ceiling prescribed Remington claims implied coverage from the
by the Monetary Board per annum from 3 phrase "perils of the sea" mentioned in the
February 1982 or 90 days from Remington's opening sentence of the policy; (3) The
submission of proof of loss to Cathay Insurance insistence of Remington that rusting is a peril of
until paid as provided in the settlement of claim the sea is erroneous; (4) Remington
provision of the policy; and ordering Cathay inaccurately invokes the rule of strict
Insurance to pay Remington certain amounts for construction against insurer under the guise of
marine surveyor's fee, attorney's fees and costs construction in order to impart a non-existing
of the suit. On appeal, the Court of Appeals ambiguity or doubt into the policy so as to
affirmed the decision of the Regional Trial Court resolve it against the insurer; (5) Remington
National Capital Region (NCR) Manila, Branch while impliedly admitting that a loss occasioned
38. Cathay Insurance moved for by an inherent defect or vice in the insured
reconsideration, but was denied. It thus filed the article is not within the terms of the policy,
petition for review. Remington, in its comment erroneously insists that rusting is not an inherent
on the petition, contends that (1) Coverage of vice or in the nature of steel pipes; (6) Rusting is
Remington's loss under the insurance policy not a risk insured against, since a risk to be
issued by Cathay Insurance is unmistakable; (2) insured against should be a casualty or some
Alleged contractual limitations contained in casualty, something which could not be
insurance policies are regarded with extreme foreseen as one of the necessary incidents of
caution by courts and are to be strictly construed adventure; (7) A fact capable of unquestionable
against the insurer; obscure phrases and demonstration or of public knowledge needs no
exceptions should not be allowed to defeat the evidence. This fact of unquestionable
very purpose for which the policy was procured; demonstration or of public knowledge is that
(3) Rust is not an inherent vice of the seamless heavy rusting of steel or iron pipes cannot occur
steel pipes without interference of external within a period of a 7 day voyage. Besides,
factors; (4) No matter how Cathay Insurance Cathay Insurance had introduced the clear
might want it otherwise, the 15-day clause of the cargo receipts or tally sheets indicating that
policy had been foreclosed in the pre-trial order there was no damage on the steel pipes during
and it was not even raised in Cathay Insurance's the voyage; and (8) The evidence of Remington
answer to Remington's complaint; (5) The betrays the fact that the account of P868,339.15
decision was correct in not holding that the awarded by the respondent Court is founded on
heavy rusting of the seamless steel pipes did speculation, surmises or conjectures and the
not occur during the voyage of 7 days from July amount of less has not been proven by
1 to July 7, 1981; (6) The alleged lack of competent, satisfactory and clear evidence.
supposed bad order survey from the arrastre
Issue: Whether the rusting of steel pipes in the
capitalized on by Cathay Insurance was more
course of a voyage is a "peril of the sea," and
than clarified by no less than 2 witnesses; (7)
whether rusting is a risk insured against.
The placing of notation "rusty" in the way bills is
not only Remington's right but a natural and Held: YES. There is no question that the rusting
spontaneous reaction of whoever received the of steel pipes in the course of a voyage is a
seamless steel pipes in a rusty condition at "peril of the sea" in view of the toll on the cargo
Remington's bodega; (8) The Court of Appeals of wind, water, and salt conditions. At any rate if
did not engage in any guesswork or speculation the insurer cannot be held accountable therefor,
in concluding a loss allowance of 30% in the the Court would fail to observe a cardinal rule in
amount of P868,339.15; and (9) The rate of 34% the interpretation of contracts, namely, that any
per annum double the ceiling prescribed by the ambiguity therein should be construed against
Monetary Board is the rate of interest fixed by the maker/issuer/drafter thereof, namely, the
the Insurance Policy itself and the Insurance insurer. Besides the precise purpose of insuring
Code. Cathay Insurance however maintains that cargo during a voyage would be rendered
(1) Remington does not dispute the fact that, fruitless.
contrary to the finding of the respondent Court
(that Cathay Insurance has failed "to present
any evidence of any viable exception to the
44
Filipino Merchants Insurance Co. Inc. vs. Court date of the filing of the complaint; and, on the
of Appeals [GR 85141, 28 November 1989] third party complaint, the third party defendant
Second Division, Regalado (J): 3 concur, 1 on Compagnie Maritime Des Chargeurs Reunis
leave and third party defendant E. Razon, Inc. are
ordered to pay FMICI jointly and severally
Facts: In December 1976, Choa Tiek Seng
reimbursement of the amounts paid by FMICI
insured said shipment with Filipino Merchants
with legal interest from the date of such payment
Insurance Company (FMICI) under cargo Policy
until the date of such reimbursement; without
M-2678 for the sum of P267,653.59 for the
pronouncement as to costs. On appeal, and on
goods described as 600 metric tons of fishmeal
18 July 1988, the Court of Appeals affirmed the
in new gunny bags of 90 kilos each from
decision of the lower court insofar as the award
Bangkok, Thailand to Manila against all risks
on the complaint is concerned and modified the
under warehouse to warehouse terms. Actually,
same with regard to the adjudication of the third-
what was imported was 59.940 metric tons not
party complaint. A motion for reconsideration of
600 tons at $395.42 a ton CNF Manila. The
the aforesaid decision was denied, hence FMICI
fishmeal in 666 new gunny bags were unloaded
filed the petition for review.
from the ship on 11 December 1976 at Manila
unto the arrastre contractor E. Razon, Inc. and Issue [1]: Whether an "all risks" marine policy
FMICI's surveyor ascertained and certified that has a technical meaning in insurance in that
in such discharge 105 bags were in bad order before a claim can be compensable it is
condition as jointly surveyed by the ship's agent essential that there must be "some fortuity,"
and the arrastre contractor. The condition of the "casualty" or "accidental cause" to which the
bad order was reflected in the turn over survey alleged loss is attributable.
report of Bad Order cargoes 120320 to 120322,
Held [1]: NO. The "all risks clause" of the
consisting of 3 pages. The cargo was also
Institute Cargo Clauses read as follows "5. This
surveyed by the arrastre contractor before
insurance is against all risks of logs or damage
delivery of the cargo to the consignee and the
to the subject-matter insured but shall in no case
condition of the cargo on such delivery was
be deemed to extend to cover loss, damage, or
reflected in E. Razon's Bad Order Certificates
expense proximately caused by delay or
14859, 14863 and 14869 covering a total of 227
inherent vice or nature of the subject-matter
bags in bad order condition. FMICI's surveyor
insured. Claims recoverable hereunder shall be
has conducted a final and detailed survey of the
payable irrespective of percentage." An "all risks
cargo in the warehouse for which he prepared a
policy" should be read literally as meaning all
survey report with the findings on the extent of
risks whatsoever and covering all losses by an
shortage or loss on the bad order bags totalling
accidental cause of any kind. The terms
227 bags amounting to 12,148 kilos. Based on
"accident" and "accidental", as used in
said computation, Choa made a formal claim
insurance contracts, have not acquired any
against FMICI for P51,568.62 the computation
technical meaning. They are construed by the
of which claim is contained therein. A formal
courts in their ordinary and common
claim statement was also presented by the
acceptance. Thus, the terms have been taken to
Choa against the vessel dated 21 December
mean that which happens by chance or
1976, but FMICI refused to pay the claim.
fortuitously, without intention and design, and
Consequently, an action was brought by the
which is unexpected, unusual and unforeseen.
consignee (Choa Tiek Seng) of the shipment of
An accident is an event that takes place without
fishmeal loaded on board the vessel SS
one's foresight or expectation; an event that
Bougainville and unloaded at the Port of Manila
proceeds from an unknown cause, or is an
on or about 11 December 1976 and seeks to
unusual effect of a known cause and, therefore,
recover from FMICI the amount of P51,568.62
not expected. The very nature of the term "all
representing damages to said shipment which
risks" must be given a broad and
has been insured by FMICI under Policy M-
comprehensive meaning as covering any loss
2678. FMICI brought a third party complaint
other than a wilful and fraudulent act of the
against third party defendants Compagnie
insured. This is pursuant to the very purpose of
Maritime Des Chargeurs Reunis and/or E.
an "all risks" insurance to give protection to the
Razon, Inc. seeking judgment against the third
insured in those cases where difficulties of
party defendants in case judgment is rendered
logical explanation or some mystery surround
against FMICI. The court below, after trial on the
the loss or damage to property. An "all risks"
merits, rendered judgment in favor of Choa,
policy has been evolved to grant greater
ordering FMICI to pay Choa the sum of
protection than that afforded by the "perils
P51,568.62 with interest at legal rate from the
clause," in order to assure that no loss can
45
happen through the incidence of a cause neither be inconsistent with the broad protective
insured against nor creating liability in the ship; it purpose of "all risks" insurance.
is written against all losses, that is, attributable
Issue [3]: Whether the insurer is liable
to external causes. The term "all risks" cannot
be given a strained technical meaning, the Issue [4]: There being no showing that the loss
language of the clause under the Institute Cargo was caused by any of the excepted perils, the
Clauses being unequivocal and clear, to the insurer is liable under the policy. It is believed
effect that it extends to all damages/losses that in the absence of any showing that the
suffered by the insured cargo except (a) loss or losses/damages were caused by an excepted
damage or expense proximately caused by peril, i.e. delay or the inherent vice or nature of
delay, and (b) loss or damage or expense the subject matter insured, and there is no such
proximately caused by the inherent vice or showing, the loss was covered by the policy.
nature of the subject matter insured. Herein, there is no evidence presented to show
that the condition of the gunny bags in which the
Issue [2]: Whether the failure of Choa to adduce
fishmeal was packed was such that they could
evidence, showing that the alleged loss to the
not hold their contents in the course of the
cargo in question was due to a fortuitous event,
necessary transit, much less any evidence that
precludes his right to recover from the insurance
the bags of cargo had burst as the result of the
policy.
weakness of the bags themselves. Had there
Held [2]: NO. Although generally, the burden of been such a showing that spillage would have
proof is upon the insured to show that a loss been a certainty, there may have been good
arose from a covered peril, under an "all risks" reason to plead that there was no risk covered
policy the burden is not on the insured to prove by the policy (See Berk vs. Style [1956] cited in
the precise cause of loss or damage for which it Marine Insurance Claims, p. 125). Under an “all
seeks compensation. The insured under an "all risks” policy, it was sufficient to show that there
risks insurance policy" has the initial burden of was damage occasioned by some accidental
proving that the cargo was in good condition cause of any kind, and there is no necessity to
when the policy attached and that the cargo was point to any particular cause. Contracts of
damaged when unloaded from the vessel; insurance are contracts of indemnity upon the
thereafter, the burden then shifts to the insurer terms and conditions specified in the policy. The
to show the exception to the coverage. As held agreement has the force of law between the
in Paris-Manila Perfumery Co. vs. Phoenix parties. The terms of the policy constitute the
Assurance Co., Ltd. the basic rule is that the measure of the insurer's liability. If such terms
insurance company has the burden of proving are clear and unambiguous, they must be taken
that the loss is caused by the risks excepted and and understood in their plain, ordinary and
for want of such proof, the company is liable. popular sense.
Coverage under an "all risks" provision of a
Issue [4]: Whether the consignee (Choa) has an
marine insurance policy creates a special type
insurable interest in said goods.
of insurance which extends coverage to risks
not usually contemplated and avoids putting Held [4]: Choa, as consignee of the goods in
upon the insured the burden of establishing that transit under an invoice containing the terms
the loss was due to the peril falling within the under "C & F Manila," has insurable interest in
policy's coverage; the insurer can avoid said goods. Section 13 of the Insurance Code
coverage upon demonstrating that a specific defines insurable interest in property as every
provision expressly excludes the loss from interest in property, whether real or personal, or
coverage. A marine insurance policy providing any relation thereto, or liability in respect
that the insurance was to be "against all risks" thereof, of such nature that a contemplated peril
must be construed as creating a special might directly damnify the insured. In principle,
insurance and extending to other risks than are anyone has an insurable interest in property
usually contemplated, and covers all losses who derives a benefit from its existence or
except such as arise from the fraud of the would suffer loss from its destruction whether he
insured. The burden of the insured, therefore, is has or has not any title in, or lien upon or
to prove merely that the goods he transported possession of the property. Insurable interest in
have been lost, destroyed or deteriorated. property may consist in (a) an existing interest;
Thereafter, the burden is shifted to the insurer to (b) an inchoate interest founded on an existing
prove that the loss was due to excepted perils. interest; or (c) an expectancy, coupled with an
To impose on the insured the burden of proving existing interest in that out of which the
the precise cause of the loss or damage would expectancy arises. As vendee/consignee of the
goods in transit has such existing interest
46
therein as may be the subject of a valid contract Policy OACM-86/002. The logs were loaded on
of insurance. His interest over the goods is 2 barges: (1) on barge PCT7000,610 pieces of
based on the perfected contract of sale. The logs with a volume f 1,000 cubic meters; and (2)
perfected contract of sale between him and the on Barge TPAC-1000, 598 pieces of logs, also
shipper of the goods operates to vest in him an with a volume of 1,000 cubic meters. On 28
equitable title even before delivery or before he January 1986, the two barges were towed by
performed the conditions of the sale. The one tugboat, the MT "Seminole." But, as fate
contract of shipment, whether under F.O.B., would have it, during the voyage, rough seas
C.I.F., or C. & F. as in the present case, is and strong winds caused damage to Barge
immaterial in the determination of whether the TPAC-1000 resulting in the loss of 497 pieces of
vendee has an insurable interest or not in the logs out of the 598 pieces loaded thereon.
goods in transit. The perfected contract of sale Panama demanded payment for the loss but
even without delivery vests in the vendee an Oriental Assurance refuse on the ground that its
equitable title, an existing interest over the contracted liability was for "TOTAL LOSS
goods sufficient to be the subject of insurance. ONLY." The rejection was upon the
Further, Article 1523 of the Civil Code provides recommendation of the Tan Gatue Adjustment
that where, in pursuance of a contract of sale, Company. Unable to convince Oriental
the seller is authorized or required to send the Assurance to pay its claim, Panama filed a
goods to the buyer, delivery of the goods to a Complaint for Damages against Ever Insurance
carrier, whether named by the buyer or not, for, Agency (allegedly, also liable), Benito Sy Lee
the purpose of transmission to the buyer is Yong and Oriental Assurance, before the
deemed to be a delivery of the goods to the Regional Trial Court, Kalookan, Branch 123
buyer, the exceptions to said rule not obtaining (Civil Case C-12601). After trial on the merit, the
in the present case. The Court has heretofore RTC rendered its Decision, ordering Oriental
ruled that the delivery of the goods on board the Assurance to pay Panama the amount of
carrying vessels partake of the nature of actual P415,000.00 as insurance indemnity with
delivery since, from that time, the foreign buyers interest at the rate of 12% per annum computed
assumed the risks of loss of the goods and paid from the date of the filing of the complaint;
the insurance premium covering them. C & F ordering Panama to pay Ever Insurance Agency
contracts are shipment contracts. The term or Antonio Sy Lee Yong, owner thereof (Ever
means that the price fixed includes in a lump being a single proprietorship) for the amount of
sum the cost of the goods and freight to the P20,000.00 as attorney's fee and another
named destination. It simply means that the amount of P20,000.00 as moral damages; and
seller must pay the costs and freight necessary dismissing the complaint against Benito Sy Lee
to bring the goods to the named destination but Yong. On appeal by both parties, the Appellate
the risk of loss or damage to the goods is Court affirmed the lower Court judgment in all
transferred from the seller to the buyer when the respects except for the rate of interest, which
goods pass the ship's rail in the port of was reduced from 12% to 6% per annum.
shipment. Oriental Assurance filed the petition for review
on certiorari.
Issue: Whether Oriental Assurance can be held
Oriental Assurance Corporation vs. Court of
liable under its marine insurance policy based
Appeals [GR 94052, 9 August 1991] Second
on the theory of a divisible contract of insurance
Division, Melencio-Herrera (J): 4 concur
and, consequently, a constructive total loss.
Facts: Sometime in January 1986, Panama
Held: NO. No liability attaches. The terms of the
Sawmill Co., Inc. (Panama) bought, in Palawan,
contract constitute the measure of the insurer's
1,208 pieces of apitong logs, with a total volume
liability and compliance therewith is a condition
of 2,000 cubic meters. It hired Transpacific
precedent to the insured's right to recovery from
Towage, Inc., to transport the logs by sea to
the insurer (Perla Compania de Seguros, Inc. v.
Manila and insured it against loss for PIM with
Court of Appeals, G.R. No. 78860, May 28,
Oriental Assurance Corporation (Oriental
1990, 185 SCRA 741). Whether a contract is
Assurance). There is a claim by Panama,
entire or severable is a question of intention to
however, that the insurance coverage should
be determined by the language employed by the
have been for P3M were it not for the fraudulent
parties. The policy in question shows that the
act of one Benito Sy Yee Long to whom it had
subject matter insured was the entire shipment
entrusted the amount of P6,000.00 for the
of 2,000 cubic meters of apitong logs. The fact
payment of the premium for a P3M policy.
that the logs were loaded on two different
Oriental Assurance issued Marine Insurance
barges did not make the contract several and
47
divisible as to the items insured. The logs on the Finman General Assurance Corporation vs.
two barges were not separately valued or Court of Appeals [GR 100970, 2 September
separately insured. Only one premium was paid 1992] Second Division, Nocon (J): 4 concur
for the entire shipment, making for only one
Facts: On 22 October 1986, deceased Carlie
cause or consideration. The insurance contract
Surposa was insured with Finman General
must, therefore, be considered indivisible. More
Assurance Corporation under Finman General
importantly, the insurer's liability was for "total
Teachers Protection Plan Master Policy 2005
loss only." A total loss may be either actual or
and Individual Policy 08924 with his parents,
constructive (Sec. 129, Insurance Code). An
spouses Julia and Carlos Surposa, and brothers
actual total loss is caused by: (a) A total
Christopher, Charles, Chester and Clifton, all
destruction of the thing insured; (b) The
surnamed Surposa, as beneficiaries. While said
irretrievable loss of the thing by sinking, or by
insurance policy was in full force and effect, the
being broken up; (c) Any damage to the thing
insured, Carlie Surposa, died on 18 October
which renders it valueless to the owner for the
1988 as a result of a stab wound inflicted by one
purpose for which he held it; or (d) Any other
of 3 unidentified men without provocation and
event which effectively deprives the owner of the
warning on the part of the former as he and his
possession, at the port of destination, of the
cousin, Winston Surposa, were waiting for a ride
thing insured." (Section 130, Insurance Code). A
on their way home along Rizal-Locsin Streets,
constructive total loss is one which gives to a
Bacolod City after attending the celebration of
person insured a right to abandon, under
the "Maskarra Annual Festival." Thereafter, Julia
Section 139 of the Insurance Code, which reads
Surposa and the other beneficiaries of said
"A person insured by a contract of marine
insurance policy filed a written notice of claim
insurance may abandon the thing insured, or
with Finman which denied said claim contending
any particular portion thereof separately valued
that murder and assault are not within the scope
by the policy, or otherwise separately insured,
of the coverage of the insurance policy. On 24
and recover for a total loss thereof, when the
February 1989, Surposa filed a complaint with
cause of the loss is a peril insured against. (a) If
the Insurance Commission which subsequently
more than threefourths thereof in value is
rendered a decision, ordering Finman liable to
actually lost, or would have to be expended to
pay Surposa the sum of P15,000.00
recover it from the peril; (b) If it is injured to such
representing the proceeds of the policy with
an extent as to reduce its value more than three-
interest from the date of the filing of the
fourths; xxx" The requirements for the
complaint until fully satisfied. As no evidence
application of Section 139 of the Insurance
was submitted to prove the claim for mortuary
Code, have not been met. The logs involved,
aid in the sum of P1,000.00, the same was not
although placed in two barges, were not
entertained. On 11 July 1991, the appellate
separately valued by the policy, nor separately
court affirmed said decision. Finman filed the
insured. Resultantly, the logs lost in barge
petition for certiorari.
TPAC-1000 in relation to the total number of
logs loaded on the same barge can not be made Issue: Whether the death was committed with
the basis for determining constructive total loss. deliberate intent which, by the very nature of a
The logs having been insured as one personal accident insurance policy, cannot be
inseparable unit, the correct basis for indemnified.
determining the existence of constructive total
loss is the totality of the shipment of logs. Of the Held: NO. The terms "accident" and
entirety of 1,208, pieces of logs, only 497 pieces "accidental," as used in insurance contracts
thereof were lost or 41.45% of the entire have not acquired any technical meaning, and
shipment. Since the cost of those 497 pieces are construed by the courts in their ordinary and
does not exceed 75% of the value of all 1,208 common acceptation. Thus, the terms have
pieces of logs, the shipment can not be said to been taken to mean that which happen by
have sustained a constructive total loss under chance or fortuitously, without intention and
Section 139(a) of the Insurance Code. In the design, and which is unexpected, unusual, and
absence of either actual or constructive total unforeseen. An accident is an event that takes
loss, there can be no recovery by the insured place without one's foresight or expectation —
Panama against the insurer, Oriental an event that proceeds from an unknown cause,
Assurance. or is an unusual effect of a known cause and,
therefore, not expected. The generally accepted
rule is that, death or injury does not result from
accident or accidental means within the terms of
an accident-policy if it is, the natural result of the
48
insured's voluntary act, unaccompanied by Sun Insurance Office Ltd. vs. Court of Appeals
anything unforeseen except the death or injury. [GR 92383, 17 July 1992] First Division, Cruz
There is no accident when a deliberate act is (J): 3 concur
performed unless some additional, unexpected,
Facts: Sun Insurance Office Ltd. issued
independent, and unforeseen happening occurs
Personal Accident Policy 05687 to Felix Lim, Jr.
which produces or brings about the result of
with a face value of P200,000.00. Two months
injury or death. In other words, where the death
later, he was dead with a bullet wound in his
or injury is not the natural or probable result of
head. As beneficiary, his wife Nerissa Lim
the insured's voluntary act, or if something
sought payment on the policy but her claim was
unforeseen occurs in the doing of the act which
rejected. Sun Insurance agreed that there was
produces the injury, the resulting death is within
no suicide. It argued, however, that there was
the protection of the policies insuring against
no accident either. Pilar Nalagon, Lim's
death or injury from accident. Herein, it cannot
secretary, was the only eyewitness to his death.
be pretended that Carlie Surposa died in the
It happened on 6 October 1982, at about 10
course of an assault or murder as a result of his
p.m., after his mother's birthday party. According
voluntary act considering the very nature of
to Nalagon, Lim was in a happy mood (but not
these crimes. In the first place, the insured and
drunk) and was playing with his handgun, from
his companion were on their way home from
which he had previously removed the magazine.
attending a festival. They were confronted by
As she watched the television, he stood in front
unidentified persons. The record is barren of any
of her and pointed the gun at her. She pushed it
circumstance showing how the stab wound was
aside and said it might be loaded. He assured
inflicted. Nor can it be pretended that the
her it was not and then pointed it to his temple.
malefactor aimed at the insured precisely
The next moment there was an explosion and
because the killer wanted to take his life. In any
Lim slumped to the floor. He was dead before
event, while the act may not exempt the
he fell. The widow sued Sun Insurance in the
unknown perpetrator from criminal liability, the
Regional Trial Court of Zamboanga City and
fact remains that the happening was a pure
was sustained. Sun Insurance was sentenced to
accident on the part of the victim. The insured
pay her P200,000.00, representing the face
died from an event that took place without his
value of the policy, with interest at the legal rate;
foresight or expectation, an event that
P10,000.00 as moral damages; P5,000.00 as
proceeded from an unusual effect of a known
exemplary damages; P50,000.00 as actual and
cause and, therefore, not expected. Neither can
compensatory damages; and P5,000.00 as
it be said that there was a capricious desire on
attorney's fees, plus the cost of the suit. This
the part of the accused to expose his life to
decision was affirmed on appeal, and the motion
danger considering that he was just going home
for reconsideration was denied. Sun Insurance
after attending a festival. Furthermore, the
then came to the Supreme Court.
personal accident insurance policy involved
specifically enumerated only 10 circumstances Issue: Whether the insured willfully exposed
wherein no liability attaches to Finamn for any himself to needless peril and thus removed
injury, disability or loss suffered by the insured himself from the coverage of the insurance
as a result of any of the stipulated causes. The policy.
principle of "expresso unius exclusio alterius" —
the mention of one thing implies the exclusion of Held: NO. An accident is an event which
another thing — is therefore applicable in the happens without any human agency or, if
present case since murder and assault, not happening through human agency, an event
having been expressly included in the which, under the circumstances, is unusual to
enumeration of the circumstances that would and not expected by the person to whom it
negate liability in said insurance policy cannot happens. It has also been defined as an injury
be considered by implication to discharge which happens by reason of some violence or
Finman from liability for any injury, disability or casualty to the insured without his design,
loss suffered by the insured. Thus, the failure of consent, or voluntary co-operation. Herein, the
Finman to include death resulting from murder incident that resulted in Lim's death was indeed
or assault among the prohibited risks leads an accident. On the other hand, the parties
inevitably to the conclusion that it did not intend agree that Lim did not commit suicide.
to limit or exempt itself from liability for such Nevertheless, Sun Insurance contends that the
death. insured willfully exposed himself to needless
peril and thus removed himself from the
coverage of the insurance policy. It should be
noted at the outset that suicide and willful
49
exposure to needless peril are in pari materia entered the house of the insured Juan S.
because they both signify a disregard for one's Biagtan.
life. The only difference is in degree, as suicide
Biagtan was killed as his house was being
imports a positive act of ending such life
robbed. The insurance company paid the basic
whereas the second act indicates a reckless
amount of P5,000 but refused to pay the
risking of it that is almost suicidal in intent. The
additional P5,000 under the accidental death
posture -- that by the mere act of pointing the
benefit clause, on the ground that his death was
gun to his temple, Lim had willfully exposed
the result of injuries intentionally inflicted by third
himself to needless peril and so came under the
parties and was not covered. The trial court
exception -- is arguable. But what is not is that
ruled that there was no proof that the robbers
Lim had removed the magazine from the gun
intended to kill Biagtan, or just to scare him
and believed it was no longer dangerous. He
away by thrusting at him with their knives.
expressed assured her that the gun was not
loaded. It is submitted that Lim did not willfully Issue: Whether or not, the wounds received by
expose himself to needless peril when he the insured at the hands of the robbers were
pointed the gun to his temple because the fact is inflicted intentionally?
that he thought it was not unsafe to do so. The
act was precisely intended to assure Nalagon Ruling: YES.
that the gun was indeed harmless. Lim was Unlike the ruling in the case of Calanoc vs.
unquestionably negligent and that negligence Court of Appeals, where the killing of the victim
cost him his own life. But it should not prevent was held as accidental and thus covered by the
his widow from recovering from the insurance insurance policy, the Supreme Court held that in
policy he obtained precisely against accident. the instant case, the insured was killed
There is nothing in the policy that relieves the intentionally. The term “intentional” implies the
insurer of the responsibility to pay the indemnity exercise of the reasoning faculties,
agreed upon if the insured is shown to have consciousness and volition.
contributed to his own accident. Indeed, most
accidents are caused by negligence. There are The Supreme Court held pointing out that there
only four exceptions expressly made in the were nine wounds in all. The exception in the
contract to relieve the insurer from liability, and accidental benefit clause does not speak of the
none of these exceptions is applicable in the purpose – whether homicidal or not – of a third
present case. It bears noting that insurance party in causing the injuries, but only of the fact
contracts are as a rule supposed to be that such injuries have been intentionally
interpreted liberally in favor of the assured. inflicted. Nine wounds inflicted with bladed
There is no reason to deviate from this rule, weapons at close range cannot be considered
especially in view of the circumstances of the innocent insofar as intent is concerned. The
case. manner of execution of the crime permits no
other conclusion.
Where a provision of the policy excludes
Biagtan vs Insular Life Assurance Co., Ltd., 44 intentional injury, it is the intention of the person
SCRA 58 inflicting the injury that is controlling. If the
Facts: injuries suffered by the insured clearly resulted
from the intentional act of a third party the
Juan S. Biagtan was insured with defendant insurer is relieved from liability.
Insular Life Assurance Company under Policy
No. 398075 for the sum of P5,000.00 and, under Under the circumstance, the insurance company
a supplementary contract denominated was correct in refusing to pay the additional sum
"Accidental Death Benefit Clause, for an of P2,000.00 under the accidental death benefit
additional sum of P5,000.00 if "the death of the clause which expressly provided that it would
Insured resulted directly from bodily injury not apply where death resulted from an injury
effected solely through external and violent "intentionally" inflicted by a third party.
means sustained in an accident and
independently of all other causes." The clause,
however, expressly provided that it would not XV. COMPULSORY MOTOR VEHICLE
apply where death resulted from an injury LIABILITY INSURANCE (INSURANCE)
"intentionally inflicted by another party."
On the night of May 20, 1964, or during the first 1. Vda. de Gabriel vs. Court of Appeals
hours of the following day a band of robbers
50
264 SCRA 137 (G.R. No. 103883) the Commissioner on the Courts within one year
from the denial of the claim, otherwise, the
November 14, 1996
claimant’s right of action shall prescribe.” The
notice of death was given to private respondent,
concededly, more than a year after the death of
Facts: petitioner’s husband. Private respondent in
The petitioner for review on certiorari seeks the invoking prescription, was not referring to the
reversal of the decision of the Court of Appeals one-year period from the denial of the claim
setting aside the judgment of the Regional Trial within which to file an action against an insurer
Court which ordered private respondent Fortune but obviously to the written notice of claim that
Insurance and Surety Company, Inc., to pay had to be submitted within six months from the
petitioner Jacqueline Jimenez Vda. de Gabriel, time of the accident.
the surviving spouse and beneficiary in an
accident (group) insurance of her dead
husband, the amount of ₱100,000.00, plus legal 2. VDA. De Maglana v. Hon. Francisco
interest. Consolacion G.R. No. 60506 August 6, 1992
Facts:

Marcelino Gabriel, the insured, was employed Lope Maglana was killed in a collision with a
by Emerald Construction & Development jeepney owned by Destrajo and insured with
Corporation (“ECDC”) at its construction project AFISCO. The heirs filed a civil case against the
in Iraq. He was covered by a personal accident owner and the insurer, and a criminal case
insurance in the amount of ₱100,000.00 under a against the driver. In the civil case, the lower
group policy procured from private respondent court held the owner Destrajo primarily liable for
by ECDC for its overseas workers. damages, while AFISCO was held liable only
secondarily (to reimburse Destrajo). Maglana’s
heirs now argue that AFISCO’s liability must be
On May 22, 1982, within the life of the policy, direct and solidary.
Gabriel died in Iraq. A year later, or on July 12, Issues:
1983, ECDC reported Gabriel’s death to private
respondent by telephone. Ultimately private 1. W/N the insurer, AFISCO, can be held directly
respondent denied the claim of ECDC on the liable by a third party.
ground of prescription. Petitioner went to court
2. W/N the insurer should be held solidarily
alleging that her husband died of electrocution
liable with Destrajo.
while working.
1. YES. Where an insurance policy insures
directly against liability, the insurer's liability
Issue: accrues immediately upon the occurrence of the
injury or even upon which the liability depends,
Whether or not the cause of action had and does not depend on the recovery of
prescribed. judgment by the injured party against the
insured. The underlying reason behind the third
party liability (TPL) of the Compulsory Motor
Held: Vehicle Liability Insurance is "to protect injured
Yes. On the issue of “prescription”, private persons against the insolvency of the insured
respondent correctly invoked Section 384 of the who causes such injury, and to give such injured
Insurance Code; viz: “Sec. 384. Any person person a certain beneficial interest in the
having any claim upon the policy issued proceeds of the policy.
pursuant to this chapter shall without any 2. NO. While it is true that where the insurance
unnecessary delay, present to the insurance contract provides for indemnity against liability to
company concerned a written notice of claim third persons, such third persons can directly
setting forth the nature, extent and duration of sue the insurer, however, the direct liability of
the injuries sustained as certified by a duly the insurer under indemnity contracts against
licensed physician. Notice of claim must be filed third party liability does not mean that the
within six months from date of the accident, insurer can be held solidarily liable with the
otherwise, the claim shall be deemed waived. insured and/or the other parties found at fault.
Action or suit for recovery of damage due to loss The liability of the insurer is based on contract;
or injury must be brought, in proper cases, with that of the insured is based on tort. • AFISCO,
51
which, under the insurance contract is liable only the highway; and that no early warning device
up to P20,000.00, can not be made solidarily was displayed.
liable with the insured for the entire obligation of
PPSII, for its part, admitted that it had an
P29,013.00 otherwise there would result "an
existing contract with petitioner Tiu, but averred
evident breach of the concept of solidary
that it had already attended to and settled the
obligation." • In fine, we conclude that the
claims of those who were injured during the
liability of AFISCO based on the insurance
incident. It could not accede to the claim of
contract is direct, but not solidary with that of
respondent Arriesgado, as such claim was way
Destrajo which is based on Article 2180 of the
beyond the scheduled indemnity as contained in
Civil Code.
the contract of insurance.
3. Tiu vs. Arriesgado Case Digest Tiu vs.
Issue:
Arriesgado G.R. No. 138060, September 1,
2004 What is the liability of PPSII as Insurer?

Facts:
A cargo truck marked "Condor Hollow Blocks Ruling:
and General Merchandise" bearing plate As can be gleaned from the Certificate of Cover,
number GBP-675 was loaded with firewood in such insurance contract was issued pursuant to
Bogo, Cebu and left for Cebu City. Just as the the Compulsory Motor Vehicle Liability
truck passed over a bridge, one of its rear tires Insurance Law. It was expressly provided
exploded. The driver, Sergio Pedrano, then therein that the limit of the insurer’s liability for
parked along the right side of the national each person was P12,000, while the limit per
highway and removed the damaged tire to have accident was pegged at P50,000. An insurer in
it vulcanized at a nearby shop. Pedrano left his an indemnity contract for third party liability is
helper, Jose Mitante, Jr. to keep watch over the directly liable to the injured party up to the extent
stalled vehicle. specified in the agreement but it cannot be held
solidarily liable beyond that amount. PPSII could
D Rough Riders passenger bus with plate not then just deny petitioner Tiu’s claim; it
number PBP-724 driven by Virgilio Te Laspiñas should have paid P12,000 for the death of
was cruising along the national highway of Sitio Felisa, and respondent Arriesgado’s
Aggies, Poblacion, Compostela, Cebu. Among hospitalization expenses of P1,113.80, which
its passengers were the Spouses Pedro A. the trial court found to have been duly supported
Arriesgado and Felisa Pepito Arriesgado, who by receipts. The total amount of the claims, even
were seated at the right side of the bus, about when added to that of the other injured
three (3) or four (4) places from the front seat. passengers which the respondent PPSII claimed
to have settled, would not exceed the P50,000
As the bus was approaching the bridge, limit under the insurance agreement.
Laspiñas saw the stalled truck, which was then
about 25 meters away. He applied the breaks The nature of Compulsory Motor Vehicle
and tried to swerve to the left to avoid hitting the Liability Insurance is such that it is primarily
truck. Too late; the bus rammed into the trucks intended to provide compensation for the death
left rear. Felisa Arriesgado, was brought to the or bodily injuries suffered by innocent third
Danao City Hospital but she died shortly parties or passengers as a result of the
thereafter. negligent operation and use of motor vehicles.
The victims and/or their dependents are assured
Respondent Pedro A. Arriesgado then filed a of immediate financial assistance, regardless of
complaint for breach of contract of carriage, the financial capacity of motor vehicle owners.
damages and attorneys fees against the Although the victim may proceed directly
petitioners, D Rough Riders bus operator against the insurer for indemnity, the third party
William Tiu and his driver, Virgilio Te Laspiñas. liability is only up to the extent of the insurance
Petitioner filed a Third-Party Complaint against policy and those required by law. While it is true
Phoenix Surety and Insurance, Inc (PPSII) as that where the insurance contract provides for
their insurer, the registered owner of the cargo indemnity against liability to third persons, and
truck and the driver of the truck. Petitioner such persons can directly sue the insurer, the
alleged that they were driving at a normal direct liability of the insurer under indemnity
speed; that the truck was parked in a slanted contracts against third party liability does not
manner as its rear was almost in the middle of mean that the insurer can be held liable in

52
solidum with the insured and/or the other parties Decree 116 (Usury Law) which raised the legal
found at fault. For the liability of the insurer is rate of interest from 6% to 12% cannot apply as
based on contract; that of the insured carrier or the adjusted rate mentioned in the circular refers
vehicle owner is based on tort. only to loans or forbearances of money, goods
or credits and court judgments thereon but not
to court judgments for damages arising from
XVI. CLAIMS SETTLEMENT (INSURANCE)
injury to persons and loss of property which
does not involve a loan. On the other hand, in
the case of Philippine Rabbit Bus Lines, Inc. vs.
1. Tio Khe Chio vs CA, G.R. No. 76101-02 Cruz, G.R. No. 71017, July 28, 1986, 143 SCRA
September 30, 1991 158, the Court declared that the legal rate of
Facts: interest is 6% per annum, and not 12%, where a
judgment award is based on an action for
On 18 December 1978, Tio Khe Chio imported damages for personal injury, not use or
1,000 bags of fishmeal valued at $36,000.30 forbearance of money, goods or credit. Clearly,
from Agro Impex, S.A. Dallas, Texas, U.S.A. the applicable law is Article 2209 of the Civil
The goods were insured with Eastern Assurance Code which reads "If the obligation consists in
and Surety Corporation (EASCO) and shipped the payment of a sum of money and the debtor
on board the M/V Peskov, a vessel owned by incurs in delay, the indemnity for damages,
Far Eastern Shipping Company. When the there being no stipulation to the contrary, shall
goods reached Manila on 28 January 1979, they be the payment of interest agreed upon, and in
were found to have been damaged by sea water the absence of stipulation, the legal interest
which rendered the fishmeal useless. Tio filed a which is six per cent per annum." And in the light
claim with EASCO and Far Eastern Shipping. of the fact that the contending parties did not
Both refused to pay. Whereupon, Tio sued them allege the rate of interest stipulated in the
before the then Court of First Instance of Cebu, insurance contract, the legal interest was
Branch II for damages. EASCO, as the insurer, properly pegged at 6%.
filed a counterclaim against the Tio for the
recovery of P18,387.86 representing the unpaid
insurance premiums. On 30 June 1982, the trial 2. Finman General Assurance Corp. v. Court of
court rendered judgment ordering EASCO and Appeals G.R. No. 138737 July 12, 2001
Far Eastern Shipping to pay Tio solidarily the
sum of P105,986.68 less the amount of Facts:
P18,387.86 for unpaid premiums with interest at
Usiphil, Inc. obtained a fire insurance policy from
the legal rate from the filing of the complaint, the
Finman General Assurance Corp. then doing
sum of P15,000.00 as attorney's fees and the
business in the name of Summa Insurance
costs. The judgment became final as to EASCO
Corp.). Sometime in 1982, Usiphil filed with
but the shipping company appealed to the Court
Finman an insurance claim for the loss of the
of Appeals and was absolved from liability by
insured properties due to fire. Despite several
the said court. The trial court, issued a writ of
demands by Usiphil, Finman refused to pay the
execution against EASCO. The sheriff enforcing
insurance claim on the grund that the same
the writ reportedly fixed the legal rate of interest
could not be allowed because Usiphil failed to
at 12%. EASCO moved to quash the writ
comply with a condition in the policy regarding
alleging that the legal interest to be computed
the submission of certain documents to prove
should be 6% per annum in accordance with
the loss. Thus, Usiphil was constraint to file a
Article 2209 of the Civil Code and not 12% as
complaint against Finman for the unpaid
insisted upon by Tio's counsel.
insurance claim. After trial, the trial court
rendered a decision in favor of Usiphil, and
ordered Finman to pay Usiphil the amount of the
Issue: insurance proceeds plus a 24% interest rate per
Whether the interest to be imposed on claims annum until the judgment proceeds is fully paid.
based on an insurance contract is 6% or 12%. Finman appealed the decision, arguing, among
others, that since there was no express finding
that it unreasonably withheld or denied the
Held: payment of the subject insurance claim, then the
award of 24% per annum is not proper. The
6%. The legal rate of interest is 6% per annum. Court of Appeals affirmed the decision of the
Circular 416 of the Central Bank which took trial court.
effect on 29 July 1974 pursuant to Presidential
53
Issue: Insurance Code. The RTC dismissed the
complaint for failure to state a cause of action. It
WON the 24% interest rate per annum awarded
interpreted the provision to mean that Trans-
by the lower courts is legal.
Asia is required to maintain the vessel at a
Held: certain class at all times pertinent during the life
of the policy. According to the court a quo,
Yes, it is, for the same is authorized by Secs. Trans-Asia failed to prove compliance of the
243 and 244 of the Insurance Code. Notably, terms of the warranty, the violation thereof
under Sec. 244, aprima facie evidence of entitled Prudential, the insured party, to rescind
unreasonable delay in payment of the claim is the contract. On appeal, said decision was
created by the failure of the insurer to pay the reversed by the CA.
claim within the time fixed in both Secs. 243 and
244. Further, Sec. 29 of the policy itself provides Issues:
that the claim shall be paid within 30 days after
Whether or not Trans-Asia breached a material
proof of loss is received by the company.
warranty that the vessel is classed and class
maintained.
3. Prudential Guarantee VS. Trans-Asia
Shipping Lines, Inc. G.R. NO. 151890 20 June
Ruling:
2006
In ruling in the negative, the Supreme Court held
Facts:
that Trans-Asia did not breach a material
Trans-Asia is the owner of the vessel M/V Asia warranty that the vessel is classed and class
Korea. In consideration of payment of maintained. Prudential Guaranty was not
premiums, defendant Prudential insured M/V successful in discharging the burden of
Asia Korea for loss/damage of the hull and evidence that Trans-Asia breached the subject
machinery arising from perils, inter alia, of fire policy condition on CLASSED AND CLASS
and explosion for the sum of P40 Million. While MAINTAINED. Foremost, Prudential, through
the policy was in force, a fire broke out while the Senior Manager of its Marine and Aviation
M/V Asia Korea was undergoing repairs at the Division, Lucio Fernandez, made a categorical
port of Cebu. Trans-Asia then filed its notice of admission that at the time of the procurement of
claim for damage sustained by the vessel. the insurance, Trans-Asia’s vessel, “M/V Asia
Trans-Asia reserved its right to subsequently Korea” was properly classed by Bureau Veritas,
notify Prudential as to the full amount of the a classification society recognized in the marine
claim upon final survey and determination by industry. As it is undisputed that Trans-Asia was
average adjuster Richard Hogg International properly classed at the time the contract of
(Phil.) of the damage sustained by reason of insurance was entered into, thus, it becomes
fire. Trans-Asia executed a Loan and Trust incumbent upon Prudential to show evidence
Receipt a portion of which states that it received that the status of Trans-Asia being properly
from Prudential P3 Million as a loan and without CLASSED by Bureau Veritas had shifted in
interest, repayable only in the event and to the violation of the warranty. Unfortunately,
extent that it recovers from any person or Prudential failed to support the allegation.
persons, corporation or corporations, or other
parties, on account of loss by any casualty for
which they may be liable occasioned by the fire
on board. Prudential, in a letter, denied Trans-
Asia’s claim from the fire incident due to the
latter’s breach of policy conditions one of which
is “WARRANTED VESSEL CLASSED AND
CLASS MAINTAINED”. This was followed by
another letter, requesting the return or payment
of the P3 Million within a period of ten 10 days
from receipt of said letter. Trans-Asia filed a
complaint for sum of money against Prudential
which sought the amount of P8,395,072.26 from
the latter, alleging that the same represents the
balance of the indemnity due upon the
insurance policy in the total amount of
P11,395,072.26. It similarly sought interest at
42% per annum citing Section 243 of the
54

You might also like